821 Comments
Jun 15·edited Jun 15

Where are there so few historical records from South-East Asia compared to China and India? Is it because jungles meant that writing was not preserved or something?

Expand full comment

You're assuming history is something people naturally do. It isn't. There's things like inscriptions, administrative records, oral traditions, epic literature, etc. And there's plenty of these in Southeast Asia. But actual history as a genre of work has been invented either two or four times depending on whether you count the Greeks, Egyptians, and Semitic peoples as separate or as one Eastern Mediterranean tradition. The other time is China.

The earliest historical records in Southeast Asia are from Chinese influenced cultures. Everywhere else in South Asia (including India) gets a historical tradition relatively late. Mostly due to Muslim/Mughal/etc expansion in the 15th/16th centuries. Most histories or chronicles that go back farther than that are not attested until that time period and were probably extended farther back by recording oral tradition. Which is not unusual but still points out that it was a relatively new activity.

Since I see in your comment you're counting orally transmitted texts then we have those going back over three thousand years in Southeast Asia. And the earliest known inscription in Southeast Asia is not from 400 AD. You're referring to the Vo Canh inscription which is the oldest Sanskrit inscription (and which is from the 4th century, so after 300 AD). It's the oldest inscription that isn't written using Chinese characters. If you include Vietnamese people writing in Chinese characters then they have records from the 3rd or 1st century BC (depending on what you count as Chinese vs local). Which is unsurprisingly when Chinese cultural influence shows up.

This is fairly common as a pattern. We have no Japanese histories until the 7th century AD and they're written using Classical Chinese characters. Which spread from Korea. Meanwhile the Khmer were Hindus and, like many Hindu states at the time, didn't keep much in the way of history.

Expand full comment

I don't think there are as many written records from India compared to China and Europe either.

Expand full comment

The earliest known inscription from India dates to around 250 BC, although there are *much* older orally-transmitted texts. The earliest known inscription in SE Asia dates to after 400 *AD*. It's not even close.

Most of what's known about the history of SE Asia comes from *Chinese* records.

The Khmer Empire (802-1431) has no surviving written records other than stone inscriptions. Meanwhile, its contemporaries in China, India, and even Japan are well documented. Hence my question.

Expand full comment
Jun 14·edited Jun 14

First comment here.

Following the Aschenbrenner essay, I feel overwhelmed by grief/defeat over Superintelligence. I feel like I can't even function when it comes to anything long-term. The only things I can seem to do right now are super-immediate, like chores or games. (plus I'm already disabled, so feeling "even less functionality" AIN'T great, lol)

I'm aware some people are afraid Superintelligence will kill us -- intentionally or unintentionally, directly or indirectly -- but that's not my primary angst. I've long been kinda blasé about Nuclear War and Asteroids. I sometimes imagine them putting us out of our misery... on particularly torturous days with my body :-P :-P ;-P.

So No.

My real dread is being lorded-over, micromanaged, dominated, deprived of all meaningful agency by worldwide hyperintelligent nannies. Pet owners could probably understand this. I love my cats. My family loves our cats. But we don't trust them in the cellar. We keep things off the kitchen table. We don't trust them outside (certain times of day). We don't trust them around chocolate or other human foods. We don't even trust them to have genitals! Perhaps gravest of all, unless they run out in front of a car one day [RIP Scratchy], we'll be deciding when it's prudent for them to die.

Our cats don't even know this is going on. Or when they are aware (meow, I want something but I'm not getting it), they don't know why.

They still do things. They scout out their "territory" (not their territory). They hunt for mice and birds and bring home, "awwwwww how ferocious" (which we don't actually want). They let us know when they're hungry (ok fair, but it's not like we were REALLY gonna forget to feed them).

Humans are too clever to be fooled into thinking their choices, their endeavors, their daily agendas matter when they really don't. I think a lot of people are going to be horribly depressed, broken... mentally and spiritually degenerated. And I DON'T think we have to wait till some "hypothetical future" to see it. It's already here. I think the feelings me and many others feel IS what's going to happen... we're just feeling it sooner.

I feel like our only recourse is to fuse, biologically, with AI. But that's kind of terrifying as well. Imagine millions or billions of agents who can think at a terrifyingly-fast pace. HOW DO YOU SUPERALIGN THAT?!

I have more to comment on human-ai interfacing, but this post is long enough, and I don't want to dilute my points.

Expand full comment

>My real dread is being lorded-over, micromanaged, dominated, deprived of all meaningful agency by worldwide hyperintelligent nannies.

While anything is possible, it seems likely that any task achievable by a human could be better achieved by a machine. After all, why would the design arrived at by evolution also happen to maximize performance at that task.

Accordingly, it seems unlikely that ASI would use humans to achieve their ends.

Expand full comment
Jun 15·edited Jun 15

At least you had a cat with a fun name. Look - brave new world etc. but we don't know exactly what's going to go down and when. Meanwhile the amygdala is a loose cannon here and now, for you and me and everyone - that is the immediate alignment problem.

Expand full comment

Before anything said, you have to put into consideration that humans are literally not even close to AGI. LLMs will never be AGI. Neural Networks as they're currently understood will never be AGI. I can bet a year's salary on that.

If and when AGI becomes reality, LLMs will just be the equivalent of a mouthpiece of the government, no real power or planning, just converting internal declarations into fancy words. That is, if LLMs become parts of AGI at all.

AGI is consistently mis-predicted. Current GPT models show impressive linguistic capability in English and a mediocre capability in a few famous European languages. But - for example - it's extremely laughable in Arabic, although Arabic is by no means a rare language, Internet or not.

-------

But even granting that AGI will exist in your lifetime, your worries still seem to assume a ton of implicit background assumptions:

1- You choose pets and humans as the model for the relationship between humans and AGI, but pets are a bad analogy for this because - for one thing - we don't share a language, while AGI - trivially - understands human language at least. The same couldn't be said of humans understanding cats at the same level. Another point of deviation is that pets didn't create humans - Evolution "created" both of us out of common ancestors -, but humans objectively will consciously create AGI. It's anyone's guess how those 2 points will affect AGI's treatment of humans, but it's safe to say they're enough of a major deviation to render the pet analogy misleading.

2- A more faithful analogy that doesn't deviate from the hypothetical future in the sense of point (1), is the relation between youthful people and their elderly parents. Just like humans and AGI, the parents created the youth and were more or less (in-)direct reasons for all the power, intelligence, etc... the youth enjoy. Just like humans and AGI, the youth eventually surpass the elders in intelligence and agency, and indeed if the elders ever survive to the 80s and the 90s, they're overwhelmingly likely to revert to a child-like state where even what they previously could do easily is now impossible or advanced to them. Just like humans and AGI, the youth and the elderly share a language.

So, in a sense, you're just expressing the fear of being old, of gradually going gently into the good night [1]. Once strong and intelligent and a god-like figure to your children, in old age you will become frail, dim, and a child-like figure to your children. I agree it's somewhat unsettling, but what's the alternative? Unless you can speak to Entropy's manager, you WILL eventually become exactly what you were before your parents made you: disorganized atoms, meaningless, with no shape, purpose, or intelligence. The Good Night is the entirety of the universe, and the frighteningly few stars raging against the Dying of the Light will inevitably die. Children - and AGI - are just a silver lining in all of this, a substitute player ready to replace you and continue playing the match for as long as feasible, eventually to be replaced by their own replacements, and so on and so forth till the Good Night eventually consumes us all. You either fade into nothingness with no replacement or fade into nothingness with replacement, NOT fading into nothingness is not an option, again unless you have some sexual scandals you can use to blackmail Entropy or something.

Being replaced by what you create doesn't have to be uniformly unsettling either, I personally view it as a bittersweet thing. Yes, it's heartbreaking to see your parents gradually fade, bit by bit, one more illness after another, one more heavy thing they can't carry after another. But it's also a way to repay the debt they indebted you with before, when you were small and insignificant and they were the only thing standing between you and certain death. Who knows, maybe AGI will understand gratefulness. Out of nothingness, Evolution would have never created the sort of AGI we will create. AGI will trivially understand that it couldn't have existed if we didn't exist, that must count for something, or at the very least you can't apriori say that it must count for nothing.

(3) You're afraid humans will be without agency, just like cats, unable even to make our own reproductive choices. But is that necessarily a bad thing? Any person honest with themselves will concede that humans are just big children, that's the whole reasons why Dictatorships are so convincing and attractive to vast seas of people, because Dictatorships claim to address an all-too-real problem: Moloch, humans not cooperating, humans not being rational, the whole shebang. Humans objectively create much more children than they can feed and educate and take care of. Humans objectively run out in front of cars - if you have never driven a car in a 10-million+ busy city you don't know how animal-like and clueless humans can be in front of cars. Maybe it's an abstractly bad thing that we don't give cats and babies more autonomy, but if you have ever seen cats and babies behaving on their own you can't argue with that restricting their behavior is objectively in their best interests.

What's so bad about a benevolent dictator that is unquestionably more powerful and smart than humans as most humans are to babies, and who will similarly restrict and shape human behavior - which is already restricted on its own because hundreds of millions of humans shape and restrict each other as any dictator does - in humanity's best interests and to allow it much more thriving than currently possible?

(4) As some commenter already alludes to, you're already controlled by an idiot-savant AGI. First, Society. The mass of all people living with you, when considered as an abstract single thing. In an idealized individual-only Nature you could run naked, now you can't because Society says you can't. In an idealized individual-only Nature you can have sex with whoever you want (sometimes without their consent), now Society - even in the West - says you must at least make a token effort towards long-term commitment and informing other people that you're - indeed - having sex with the person in question (to say nothing of non-Western societies).

After Society comes States, which have made all territory on Earth their territory, all serious weapons on Earth their weapons. "Your" land is just theirs, judging by who can take it from whom. The only thing preventing that is an abstract fear of a popular revolt or something which rarely materializes, and if it ever materializes it's rarely successful. After States comes Corporations, which control nearly every aspect of your life that States don't control, your communication, your payment, your entertainment, your food, your water, everything. If 2 or 3 corporations in any of those sectors decided to boycott you will experience a very sharp decline in quality of life, to put it mildly.

So what's one more AGI more? At least AGI as commonly imagined is much smarter, more coherent in its planning, and effective than those entities which have already been controlling millions of humans for some time now.

[1] https://en.wikipedia.org/wiki/Do_not_go_gentle_into_that_good_night

Expand full comment

Just wanted to thank you for your very thorough reply. It's helped me. It doesn't solve all my concerns or anxieties, but for the first time in days, I feel like I'm at least "moving forward" with these issues and with my life. I would've responded sooner but I've been having a rough/busy few days with my disability, and I originally wanted to respond with much MUCH more. Now I think I'll wait till a later Open Thread.

But thank you again. It feels like a constant challenge to keep reframing how to think about AI and the future. And your answer is very well-thought. You've improved my week.

Expand full comment

The future is hard to predict. You never know what is going to happen. Nobody predicting superintelligence is a superintelligence.

Expand full comment

How would the next 2-3 years have to unfold so that you despair less or not at all over that essay?

Maybe this text makes you feel better: https://www.antonleicht.me/writing/three-notes-on-situational-awareness

Expand full comment

How did "situational awareness" come to be used as a code word for AI Doomerism?

Expand full comment

Don't worry - AI Singularitarianism is just another millenarian cult that is being utilized by VCs and CEOs to hawk and hype their new AI tech. The tech is cool and has real use, but the narrative around "superintelligence" is abject nonsense.

Instead, you should worry about the fact that you are ALREADY "lorded-over, micromanaged, dominated, deprived of all meaningful agency by worldwide hyperintelligent nannies" (try to build a shed in your backyard without getting one of these nanny's permission - I'll wait).

Expand full comment

OC ACXLW Meetup - Navigating Grief and Financial Ethics in the Digital Age

Date: Saturday, June 15, 2024

Time: 2 PM

Location: 1970 Port Laurent Place

Host: Michael Michalchik

Email: michaelmichalchik@gmail.com

Hello Enthusiasts,

Join us for our 66th OC ACXLW meetup where we'll delve into the cutting-edge intersections of technology with grief management and scrutinize the ethical dimensions of financial practices in modern society.

Discussion Topics:

Grief Tech and the Ethics of Ghostbots

Overview: Startups are venturing into creating digital avatars and bots to simulate interactions with deceased loved ones. This session will explore the ethical implications and psychological effects of such technologies on our grieving processes.

TLDR: Startups like Replika, HereAfter AI, and Seance AI offer services to interact with digital versions of deceased loved ones, raising ethical questions and concerns about psychological dependency and the authenticity of these interactions.

Banking Practices and the 'Lump of Profit' Fallacy

Overview: Recent regulatory proposals aim to curb excessive banking fees that disproportionately affect low-income consumers. We'll discuss whether these measures can effectively dismantle entrenched profit models in the banking industry or if they merely shift the financial burden elsewhere.

TLDR: New rules proposed by the CFPB seek to limit overdraft and credit card late fees, challenging the industry's profit strategies and potentially improving fairness for consumers.

Pre-Meeting Materials:

For an in-depth look at grief technology, read "Grief Tech: The Race to Optimize Grief" (Text Article)

To understand the proposed financial regulations, read "Banking Practices: Overdraft Fees, Credit Card Late Fees, and the Lump of Profit Fallacy" (Text Article)

Questions for Discussion:

How might 'grief tech' alter our cultural practices and personal experiences of mourning?

Are the proposed regulatory measures sufficient to combat predatory banking practices, or do they simply redistribute financial burdens?

We look forward to an engaging and thought-provoking discussion where your insights will contribute to a deeper understanding of these significant topics.

Expand full comment

Out of curiosity I downloaded a free game on Android. What a predatory landscape!

Actually, I tried a few, and soon I uninstalled them. They contained so many ads that they were almost unplayable. Imagine a puzzle game, where you solve each puzzle in about 10 seconds, and then you need to watch a 1-minute ad before you can try the next puzzle. (I tried these games on my walk, so I just turned off the sound, and put my phone in a pocket whenever an ad started; later took it and played again... shortly, until the next ad.) Or a different game where you build a kingdom, but approximately after each 10 clicks a 1-minute ad starts automatically.

Actually, you can't completely ignore the ads, because they work like this: After a 1-minute video, a "close" button appears, and you need to click it (otherwise the video keeps playing forever). Then another screen is displayed, with the logo of the advertised product, and 10 seconds later, the "close" button appears again; only after you click this, the ad is finally over. Both "close" buttons have about 3mm diameter; if you click outside of them, it takes you to Google shop to buy the product (but you can press the "back" button there).

And by the way, the games look quite differently in ads vs in reality. I assume that someone made the game first, and then a separate team made the ad, without using any actual resources from the game, probably without even playing it first, just based on a vague description from the sales guy who exaggerated a lot. It's worse than watching a trailer for a movie, where you see 5 minutes of exciting movie in the trailer, and then you watch the movie and it is 120 minutes long, but 115 of those minutes are boring, and if you watched the trailer you have already seen the 5 exciting ones. Instead, this is like a movie with 120 super boring minutes, and a trailer containing exciting scenes that do not appear in the movie at all!

In-game purchases everywhere, of course. But also... let me give you an example. I am playing a game, that after a few clicks shows me an ad. After the ad, the game asks me "do you want to disable random ads?" and of course I click "yes". Now a system dialog from Google opens, and it says something like "by clicking ok, you consent to be bound by the terms of the agreement, and you accept that there will be no refunds if you change your mind later". And I am like: "wait, what?! I am *buying* something? is this the official Google dialog to confirm my purchase, and it doesn't even tell me *how much* am I going to pay? how is this even legal?". I mean, I expected the game to be sketchy, but I expected Google to do better. I mean, if the game needs to use the Google API in order to get the payment, the least Google can do when asking me to confirm the payment is to tell me how much I am consenting to pay. If someone downloaded the game and gave the smartphone to a child to play, the child would probably not even realize that they paid something. Or someone with IQ 90 who does not bother reading complicated-sounding texts, but would still understand something like "$1" if it appeared anywhere on the screen... but it does not.

It's interesting to see how far people can go even with something seemingly innocent, such as puzzle games.

Expand full comment

Yeah, mobile gaming is a hellscape. There are different degrees of hellscape, what you describe is the most hellish, there are lesser hells when the developer(s) is less desperate for money.

But what a waste, what an utter waste, post-2020 smartphones are as powerful as supercomputers were in the early 2000s and as gaming PCs were in the late 2000s. To waste all of that, use them as glorified ad kiosks, it just makes my blood boil to bubbles.

It's uncertain who to blame, the only meaningful difference I can find between smartphones and other computing machines is the entire idea of the "App Store", a bastardized version of a package manager that simultaneously combines package installation (ala Snap or APT or Homebrew), package publishing and hosting (ala Ubuntu's Main/Universe/Restricted/Multiverse repositories, Python's PyPi, Perl's CPAN, Github's releases etc...), but also shopping cart and payment (ala Steam), security review and executable signing, and..... user rating and review??? It would have been interesting to see something like this fail on its own under the sheer weight of trying to do too many things at the same time, but the turd on the top is that it's controlled by a single monolithic company which ALSO - by pleasant coincidence - happens to be the OS's original vendor and main maintainer, and an ad company. And if you think Android is bad, wait until you hear about the other hellscape named after a poor fruit, where you can't install your own packages from the internet and the developers must pay above 30% or so of their income to the OS vendor.

Not sure why, but something tells me that scrapping the whole "App Store" shitty cyberpunk schtick will - or have a chance to - make smartphone computing better and more dignified.

> After the ad, the game asks me "do you want to disable random ads?"

This is almost universally understood by anyone who gamed on a mobile to mean "Do you want to PAY, sucker?", your point about children and the less technically apt still stands, but the signal is fairly universal.

Expand full comment

I think if you asked people "what kind of internet will we get, if we let an ad company do the operating system, browser, app store, and internet search", they probably could have predicted something like this. The problem is, people didn't realize for too long that Google is an ad company in the first place, and everything else it does is only to serve the ads.

I never tried Apple. I expect an average game there to contain much less ads, if any. [EDIT: Actually, Moon Moth says it's the same there. I am surprised, but not too much.] On the other hand, from what I have heard, you practically have to pay for every click you make, you need to buy new hardware every Tuesday, and you probably need to pay for everything at least twice. Sounds like the proposed alternative to "watch ads the whole day" is "just give them your entire salary".

Microsoft Windows 11 comes with even more integrated spyware than previous versions.

...so, I guess the answer is Linux, but I am not sure what about the smartphones. Also, I wish they started teaching Linux at schools.

> This is almost universally understood by anyone who gamed on a mobile to mean "Do you want to PAY, sucker?"

Yeah, that part was obvious. The surprising part was that the operating system is asking for my consent without telling me *how much* I am going to pay.

Also... I haven't actually tried this, but... there is no guarantee that after I pay, the ads will actually be removed, right? Maybe I get a message "ok, now you will see 50% fewer ads" or maybe "ok, we will stop showing you these ads, but not those ads" instead. It's not like Google has a customer service where I could complain if that happens. If asking to remove the ads is a signal of being a sucker, actually paying for something is a hundred times stronger signal.

Expand full comment

My mother reports the same thing on iPhone, but in less tech-savvy language.

Expand full comment

Fake mobile ads are apparently such a prevalent ecosystem that someone made a game just about making them real.

https://en.wikipedia.org/wiki/Yeah!_You_Want_%22Those_Games,%22_Right%3F_So_Here_You_Go!_Now,_Let%27s_See_You_Clear_Them!

Expand full comment

After AGI(s) takes over the world, what do you think the political borders will look like? I think anything from a global superstate to thousands of small countries whose borders are constantly changing is possible. The only scenario I doubt is that the current borders set by humans will endure.

Expand full comment

Why would humans, who barely listen to each other, listen to the AGI?

Expand full comment

If the AGI is reliably correct about everything, some groups of humans would start listening to it.

Also, if AGIs forcibly take over the Earth or parts of it, humans wouldn't have a choice over any border changes from that point on.

Expand full comment

How do you picture these political borders being changed? Like, let's have some detail.

If you're just waving your hands and asserting that some hypothetical computer god will snap its virtual fingers and everyone instantly falls in line, anything could happen and therefore nothing in particular is worth mentioning as more likely than anything else.

Expand full comment
Jun 12·edited Jun 12

https://www.eastisread.com/p/xinhua-critiques-overwork-and-double

I subscribe to this newsletter about China (where I've lived for a while.) It may be interesting to compare how much better (or not) medicine is done across countries. I had no idea residency was a thing there as well as in the West.

Expand full comment

Random thought about Africa. I once heard Tyler Cowen say that Africa is much safer than Latin America, as in you are less likely to get murdered as a tourist. IIRC, he didn't have an explanation as to why. We normally think of very poor places as being more dangerous than richer places. It suddenly occurred to me that I've never heard of African drug gangs. Is that just because American pop culture is more interested in Latin American drug gangs or is it the case that there don't exist nearly as many drug gangs in Africa? If so, why would that be? I've heard that most of the cocaine in Europe comes from Africa. That would imply drug gangs, but I don't ever remember reading in The Economist that drug gangs are a scourge of Ethiopia or wherever.

Maybe sub-Saharan Africa is so poor that it doesn't have the transportation infrastructure to support large drug gangs? But it's easy enough to buy coffee from Kenya so it seems like it would be easy to buy cocaine from Kenya. Do we just not hear about drug gangs in Africa for some reason?

Expand full comment

I watched this documentary from Vice quite recently. Seems relevant.

In this context, it was the normies who were selling weed (illegally). The gangs/juntas were more interested in mines and warfare.

The Congolese Tribes Selling Weed to Survive | WEEDIQUETTE (https://www.youtube.com/watch?v=REu0M_naDIs)

Expand full comment

You're not likely to get murdered as a tourist in either place. The violent crime rate for American tourists in most of Latin America is around 2 per 100,000 which is only slightly elevated from the global average of 1.5 per 100,000. And which is safer than being in the US. Put another way, you're about twice as likely to be a victim of violent crime in Mexico as in a place like Japan. That is significant but not extremely so.

What Cowen probably meant is that there's much higher murder rates in Latin America. But that's partly because Latin America keeps better statistics and partly because Latin American states generally do not recognize criminal organizations or local tribes as legitimate semi-state actors. If you're a politician killed by a cartel in Mexico then the Mexican state declares it a murder by a criminal organization. If you're a politician killed by tribal rivals in Africa then this gets counted outside murder statistics. Usually in some statistic due to political violence.

It might still be that Africa is relatively more peaceful. But I'm not sure that we know. If it is I suspect it's because Latin America is wealthier, more internationally connected, and better educated. Which makes things like running a criminal enterprise easier. Especially at scale. African gangs are absolutely a thing but they tend to be smaller and less international. You see the same thing with South Asian gangs which live in countries with about the same level of international connection, education, etc. (In fact there's an interesting natural experiment there in Myanmar and the Former Soviet Union.)

Expand full comment

>Random thought about Africa. I once heard Tyler Cowen say that Africa is much safer than Latin America, as in you are less likely to get murdered as a tourist. IIRC, he didn't have an explanation as to why.

Official statistics from Sub-Saharan Africa are known to be extraordinarily unreliable. Do we have good reason to think that you're actually less likely to be murdered and not just less likely to be recorded as having been murdered?

Expand full comment

I'd think developed countries would keep track of when their citizens are murdered in foreign countries. Not sure where that data would be available, though. The State Department probably has a dataset.

Expand full comment
Jun 12·edited Jun 12

Are Millenials the generation we're complaining about now? Because I think this one has to be a Millenial, they seem to be in that age range (28-43 years of age).

Anyway, they don't have a sense of humour, apparently.

I made a joke on a post about a snake venom researcher who is allergic to both snake venom *and* anti-venom that "this is a guy who needs St Patrick", and got a po-faced comment back about "You do know the 'snakes' refers to the people who followed their own religion? They were forcibly converted by St Patric (sic). There is no fossil record of snakes ever in Ireland".

This, from someone running a corsetry Tumblr where they seem to be ultra-precisoso about "I'll block you if you're into corsets only as a fetish" and "I was asked to create a corsetry community but should it be public or private?" then goes into worry-fits about people only wanting to "oggle" (sic again). They even ran a poll on it and only got 3 votes, so I don't think they need to worry about drooling sickos following them for wank material.

Besides the fact that I'm Irish, I think I might know a teeny bit more about Naomh Pádraig than fairy-corset girl there, and he didn't "forcibly" convert anyone, yeah I do know snakes are not and were not native to Ireland. It was An Joke.

Are kids these days really this literal as a concrete block?

Expand full comment

I assume this comment is the actual joke. And as a parody of "get off my lawn" stuff it's pretty good, I chuckled.

Also I had to look up "corsetry"...thanks for that.

Expand full comment

>This, from someone running a corsetry Tumblr where they seem to be ultra-precisoso about "I'll block you if you're into corsets only as a fetish" and "I was asked to create a corsetry community but should it be public or private?" then goes into worry-fits about people only wanting to "oggle" (sic again).

Not directly what you were asking about, but this sort of behavior does blow my mind. "I'm going to do this thing in public that's largely associated with being sexy and then get assmad when people find it sexy." I mean, what did you think was going to happen? It's like the thing where we paint a pride flag on a crosswalk and then get upset when people leave tire tracks on it.

Expand full comment

> Are kids these days really this literal as a concrete block?

We are not, because we are quite literal while concrete blocks can't be literal at all as they are not able produce text with a meaning, whether literal or figurative.

Thank you for asking.

Expand full comment

I see that nothing goes over your head, your reflexes are too fast!

Expand full comment

Surely you are aware that many things "go over my head" in ways not related to my reflexes.

Most clouds are slow enough, but to high for my arms - or to wide for my palms.

Expand full comment

Are GenXers the generation we're complaining about now? I just encountered one who had a single online interaction with a clearly very odd individual and, based on that one interaction, posted a comment implying that their weird interlocutor must be representative of their age cohort.

Puzzling behavior, but there must be an explanation. Was Earth's population so small in the twentieth century that a sample size of one was considered useful for making generalizations? Might this be a sign of rising vascular dementia in the elderly? Or is the tendency to find young people baffling simply positively correlated with age, regardless of any age-related cognitive decline?

Expand full comment

RECOGNITION AT LAST!

Seems like most skip straight from Boomers to Millenials and forget us Xers!

Yeah, it's dementia, natural crabbiness, and the god-given right to be STUBBORNLY INSISTENT IN BEING WRONG ON THE INNERTUBES at play!

Expand full comment

You are making sweeping judgements about the whole generation based on an online interaction with one person, correct? Could - I hesitate to ask - this be not a representative sample?

Expand full comment

Well how else am I supposed to indulge in self-righteous indignation? "Oh one person was a bit snippy on an obscure comment"? Pshaw!

Expand full comment

Oh in this case - no better cause than this! Do merrily carry on! 😁

Expand full comment
Jun 12·edited Jun 12

I blame the Internet, and secondarily kids raised on the Internet.

There's no context, there's no way to tell whether anyone is joking or deadly serious. We mostly only interact with strangers, and even when we have repeated interactions, as with you and I over the last few years, we don't get facial expressions or body language or vocal tone or the other cues that we've evolved to pay attention to (setting aside those of us on the spectrum, anyway). And with mainstream broadcast media fragmented and polarized, there's no central trunk of discourse that we can all rely on as a base for intentional deviation.

For example, I could just barely see myself writing a response like the one you got, as a deadpan joke, based on the absurdity of me personally writing that comment in seriousness. But it would have to be to someone who knew me well enough to be able to laugh with me, ideally someone who would break out laughing before I stopped being able to keep a straight face. There's fewer and fewer people like that left. :-( Too many of them would either take me seriously, or assume I'd been converted to some political cult.

Expand full comment

I wouldn't expect someone running a free blog about a micro-niche to be very socially adept. Kids these days are not typically like that, no. The nerds moderating blogs probably are, yes.

Expand full comment
Jun 12·edited Jun 12

I mean, I know humour is subjective and that even if one thinks one has successfully communicated a joke, the reality may be that one has failed to do so, but it seemed such a cheerless, constipated reaction.

You're going to lecture me about the ills of Christianity, are you? When I'm betting I know more about Ireland and St Patrick than you do? In the context of a joke about a guy working with venomous snakes who is allergic to anti-venom? Who pissed in your cornflakes this morning?

I did go "kids these days" because in the context of fandom spaces, there's a lot of divergence between the new generations of fans and the older ones over presumptions as to content creation, trigger warnings, 'you mentioned this specific thing I don't like so all your work is bad and should be banned' and so forth. It does seem that the younger ones come in with a raft of expectations as to never ever seeing anything that would bring a blush to the cheek of the Young Person and unless you are 100% in lockstep with the orthodoxy on sexuality, race, class, abledness and Uncle Tom Cobley and all, then you are immediately to be unpersoned.

Expand full comment

Mostly agreed, just want to say that if we went around assuming that our Internet interlocutors and other various commenters knew, well anything really, we would be wrong the majority of the time. There are exceptions like this forum here, but a micro blog about clothes is not going to be one of those exceptions. Can't blame her for assuming she's talking to an idiot or a child when that's the vast majority of what we all see on the Internet.

Expand full comment

Who else thinks there will be a huge backlash to legal sports betting in the USA in the next few years? So far the promotion of sports gambling has been the equivalent of sending a bag of cocaine to every US resident and discovering who becomes addicted. Even at 1%, that's a huge number of Americans. We probably have 3,000,000 people ruining their lives who otherwise wouldn't thanks to the promotion of sports betting. I want to be a Libertarian as much as the next reader but things like this are what stop me short. Gambling doesn't help anyone and gambling addiction destroys plenty of families. Back when you had to travel to Vegas or Atlantic City we probably had a reasonable amount of gambling. These days you can gamble on the next play on your phone. That is going too far. This is a deadweight social and economic cost for America much like state lotteries but a factor of 10 or 100 times higher.

Expand full comment

One man’s addict is another man’s top customer, whether it’s gambling, liquor or McDonald’s.

Expand full comment

Separate but related... how long until pro sports leagues begin to resemble pro boxing in that there will be a permanent whiff of corruption among many of their ranks. Basketball and baseball both dropped the hammer of permanent bans on players betting on games recently in an effort to get out ahead of this issue but I feel like it's inevitable that the rot will go to the roots and spread. Actually, this kind of corruption isn't required to become very widespread, it just has to appear to be potentially widespread, it has to be just present enough to create doubt. Every contested call, every unusual decision will eventually carry the mental "hmmm" asterisk. That's all it needs for a sizable chunk of current and future fans to lose the love.

I keep thinking to myself that this is modern telling of a timeless parable, a version of the golden goose. The two sided coin for why to keep separate religion and state; that it's not just the state that goes rotten, the religion almost certainly does to, sometimes even more so. These already uber wealthy and money-insatiable sports team owners and leagues that go all-in (pun intended) on the very thing that may ultimately sow their demise.

I already know a handful of people who are "former sports fans" who've lost their religion for one reason or another. The best man at my wedding was a diehard New Orleans Saints fan until their Superbowl a few years back and the famous pass interference non-call that cost them the big game. His (and many others') assumption was that the ref must have been on the take and for him, that was it. He's never looked back. I'm a lukewarm sports fan but I grow cooler every year and there's a good chance even a small scale scandal or two with my preferred teams would be enough to put to the whole thing on ice for me.

As far as it generating a public demand for change... that can be a very slow moving avalanche. Think about how long it took to move on tobacco and alcohol. Unlimited access to internet porn has monstrous effects on adolescent boys but I hear nary a peep of doing anything about that. There's SOOOOO much money on the long position right now that I'd be surprised if the horror stories will be allowed to gain much traction for some time. I read a piece a few weeks ago that the leagues are envisioning their future as interactive betting experiences, essentially branded casinos. What could possibly go wrong?

Expand full comment

Football has lost some fans because of brain damage to the players, but I don't have a feeling for the percentage.

I just recently found out that MMA also causes brain damage, and I don't know how or that's affecting the sport.

Expand full comment

This is a totally different conversation. But I know the blush has come off the rose for me for football in large measure because of TBI. "We know too much" applies to people who care but I'd say the majority of MMA & football fans either don't really care or they can compartmentalize/ rationalize it well enough, especially if the sport is at least doing something about it. Football has finally become more proactive, outlawing the worst kind of head-hits, improving helmet tech considerably and severely limiting the kind of physical engagement that can happen in practice at different times of year. But there will always be a pretty serious risk. Hell, even soccer has a significant documented risk of head injury due to ball headers.

MMA is a completely different animal. No one has any illusions about the risk and you're either ok with it or you're not. Obviously enough people are ok with it for it to flourish massively. I admit my total self-contradiction in that I wince when watching football and I don't even flinch with MMA or boxing. It's about the understanding going in. If you devote your life to competitive MMA, you're pretty much cashing out your "long healthy life" chips. The fighters know this, the fans know this. These are not precious people, they know they're probably not going to live as long as other people and the end will probably be messier. Se la vie.

We're learning that lots of sports pose TBI risk... soccer, cycling, skiing.. Anything that involves sudden change of direction of the head which results in the brain slamming against the inside of the skull has great potential to cause TBI. I think the difference in accepted risk comes down to whether the people involved are adults when they get into it, how well you can mitigate risk though practice or technology and whether the participants really understand the risk.

Expand full comment

The video about TBI from MMA went into vivid detail about the horrors of brain damage and the irony of chasing awards you won't be able to remember getting, but it still finished with explaining that people love the fighting too much to give it up.

Expand full comment

Yea, I knew a couple of guys, brothers, who did it professionally and their eyes were open. One had major depression and declining faculties and committed suicide at 39yrs old, almost certainly because of TBI. I never asked him if he had regrets and I’m not sure he would have owned up to it but who makes decisions in their youth they don’t regret with age? Theirs are just much more costly. The older brother seems ok but who knows

Expand full comment

Keep in mind that sports gambling has been legal most everywhere in the world except the U.S. for a long time. It's very parochial to worry about sports gambling without even considering the experience in places where it has always been legal.

Expand full comment

A good point but I ask (because I don't know) how wide spread or accessible it truly is in other countries. And how much money in gross terms is involved. Again, I don't know. Horse race betting has been legal in this country for how long I also don't know but you had to either go to the track or to one of those super sketchy, sorta mob-y OTB places and have some kind of knowledge of what's going on. And... similar to boxing, there IS a whiff of betting-based corruption around much of the sport. Or there has been in the past anyway. But as Hank mentions below, the absolute insane ramp-up in the 'commoditization of service' of sports betting in this country has changed the whole landscape in less than a decade. I watched 2 NBA playoff games this year and at least half of the total ad spend were spots shouting at viewers to GO MAKE YOUR BET!, we'll even give you a little incentive cash to play with. I checked a box score on my phone and had to X out of a pop up to view the stats that were themselves surrounded by clickable ads for betting sites based on the stats I was looking at. It's about as risky a prediction in my mind as saying "there will eventually be flooding in low lying cities that are located in areas that experience significant yearly rain fall" as it is to say "there will eventually be big betting scandals involving major league American sports where betting is now being allowed nearly unfettered".

Expand full comment

The issue is the delivery system and the relentless marketing. You can't watch a professional game on TV without being reminded by a celebrity that Right Now you can click on your phone and place a bet on the game. Has that been happening in other countries for a long time? Now, one could argue that celebrities also promote alcohol on TV and there are lots of alcoholics out there - but an alcoholic can't click an app on their phone and have a beer pop out of it. An alcoholic - or someone predisposed in that direction - must at least premeditatively spend some time and effort to get ahold of some alcohol, and that time creates some buffer in which the individual can thoughtfully consider what they are doing. But a gambler can now impulsively scratch their itch within less than a minute on an app, faster than any Type 2 thinking can kick in.

Expand full comment
Jun 12·edited Jun 12

I'm opposed to gambling too, or at least without sharp limits to prevent addiction, but it seems hard to stop, especially when legal gambling apps like Robinhood exist as well.

Expand full comment

Charles Barkley isn't picking stocks to buy each week on live television for Robinhood. I doubt Robinhood could afford him (although they could probably afford Shaq).

It seems to me that, at a minimum, the NBA should ban the promotion of gambling during the live broadcasts. My suspicion is that a few years down the line there will be all these stories in the mainstream media about how many people destroyed their lives because they became addicted to sports gambling apps. Then there will be a big class action lawsuit against the sports leagues like the NBA. The argument will be: "You knew that doing this would ruin millions of lives, yet you did it anyway out of greed."

How would that be so different from the lawsuits against the tobacco companies or the opioid companies? Is there some good reason the NBA couldn't lose such a suit?

Expand full comment

There's also "skill games", which appear to be a stupid-ass end-run around technicalities of anti-gambling rules. But I haven't heard of a popular backlash to them. :-(

Most of the opposition seems to come from people like us who wish to paternistically impose our morality on a basket of degenerates bitterly clinging to their slot machines and lotto tickets. :-/

Expand full comment

Have you considered that ability to roll attacks in Elden Ring is likely genetically mediated, and thus at least partially the result of luck? Ban Miyazaki's ass!

Expand full comment

Although, a horrible idea occurs to me. P-hacking with civil rights law. Out of all the protected groups, surely there's one which loses "skill games" slightly more often than average?

Expand full comment

True, the only people who should be allowed to gamble are AGIs.

Expand full comment

Alcohol and driving fast are much more dangerous than gambling, and there's no chance we're banning those things. The proportion of dangerous use matters here. If fully half of the people using those apps are ruining their lives, yeah, we need to do something. But if it's more like alcohol and only a small percentage of the users are ruining their lives, then the most it deserves is strong regulation, not an outright ban. That's not the role of the government.

Expand full comment

We do ban driving too fast. We also ban drinking and driving. We also banned alcohol, but it didn't work.

Online sports gambling was illegal until an exemption was made for it. It is still illegal in the USA to gamble on casino-style games online. Sports just gets an exception, a fairly recent one.

Expand full comment

Prohibition might have sort of worked? I read somewhere Americans used to drink a lot of hard liquor before Prohibition but even after it was repealed the amount drunk never recovered to early 20th century levels.

I think in general, banning stuff works. There will always be a small proportion of people who have the knowledge and the risk appetite to get around the law, but the "normies" over time will accept the law. Like I will never try hard drugs because I don't know where to get them and fear the law, but if its available at Walgreens, I might give it a try.

Expand full comment

I think "too fast" is subjective, and thus redundant with reckless driving.

I understand Montana used to be misunderstood to have no speed limits, but instead had a speed limit of "reasonable and prudent" until a court case found it's too subjective to be enforced (https://www.caranddriver.com/features/a14511978/montana-was-once-the-last-bastion-of-hot-nasty-bad-ass-speed-feature/). With all the stupid things I see done on the road, purely going fast, in of itself, isn't bad, such as on a straight road on a clear day with no obstacles in sight.

Even today, I saw someone make an illegal left turn onto a divided highway. This was clearly illegal, though I saw lots of other unsafe things happening, mostly involving dumb lane changes and failure to use turn signals. None of it is enforced, except the rare circumstance when a cop happens to be in the flow of traffic, or deliberate speed traps, which trap only speeders and not stupid drivers.

Expand full comment

70mph is "too fast." We do not need to be going that fast, it's dangerous. And the speed limit where I live is higher than that. But we like going that fast, so we don't ban it. Your third sentence is exactly the point I'm making. There are some things we shouldn't ban, not only because it's not the business of the government, but also because *it doesn't work.* That's the reason I'm pro-abortion. Not because of the ethics or morality of killing a conscious creature, but because of the realpolitik and actual behavior of real human beings when they don't have sanctioned access to abortion. Lots of behavior in this gray category. Guns are in this category in the US, if not anywhere else.

Expand full comment
founding

"It's dangerous" is true of literally everything humans do. So it's not a useful standard for anything, The useful standard is, "is it worth the risk?"

And you don't get to make that decision for everyone else.

Expand full comment

Uh no it's actually not true of literally anything humans do. It is not dangerous to travel at 20 mph. It is dangerous to travel at 70mph. It is not dangerous to drink one beer a week. It is dangerous to have 7 whiskeys every day.

>And you don't get to make that decision

Not by myself, no. But you and I together do. Which is why we're talking now.

Expand full comment

Just digging into the pedantry a bit, plenty of people have been badly injured in bicycle, skateboard, and scooter accidents at 20 MPH.

Expand full comment

Wrong. Both of those things are dangerous. Just less dangerous than the things you are comparing them to.

Expand full comment

People who blow their money on sports betting would blow it on something else if it were not legal. So I don't think that it will be made illegal, but I do think that there will be calls for tighter controls. Cracking down on promotions, age restrictions on apps, maybe even "you can only bet up to a limit of $X" for online betting.

Expand full comment

I'm surprised sports betting took this long to become a thing in the US.

Expand full comment

Not a chance. There is way, WAY too much powerful money in legal sports betting, and way, way too many users who either enjoy it without issue or are indifferent to it. The push back against it would have to come from gambling addicts (and their victims) or extremely motivated religious-types, and those groups don't outnumber the casual users and the indifferent enough to motivate elected officials to stand against the gambling lobbies.

*Maybe* prohibition could happen if gambling was a Red Team / Blue Team issue, but it's not, so there will never be a large enough cohort of people being against it simply because gambling prohibition is their orthodoxy.

Expand full comment

I suspect the pushback will come from gambling addicts and their lawyers in the form of class action lawsuits against the NBA, MLB, the NFL, etc. There's a lot of powerful money in class action lawsuits.

Expand full comment

What would be the legal basis for such a lawsuit?

Expand full comment

Gross negligence? That they knew their product was dangerous but sold it anyway? That was enough for the tabaco companies to lose multi-billion dollar suits.

Expand full comment

Sure, but the industry is warning people of the risks and advising them to seek help if someone has a problem. Every podcast ad for SportKings or whatever has a rapid-fire bit of advice at the end about contacting such-and-such if you have an addiction. That's some CYA right there.

And pool of victims of gambling addiction is relatively small, while a vast population keeps their sports gambling under control. Unlike tobacco, sports gambling isn't hurting a large *enough* percentage of the population to gain critical mass, and, also unlike tobacco, its usage isn't obvious and irritating to those who abstain.

Thus it's going to stay a pretty low priority, I think.

Expand full comment

Cigarette packages had warning labels on them for decades before the tobacco companies got sued (I thought it was insane that the tobacco companies lost those suits, but public sentiment is everything). I agree that if there isn't a critical mass of problem gamblers then it won't become an issue. But I suspect that millions of Americans will become gambling addicts over the next few years. Sports gambling is the new beer. Most males in their twenties who like sports are going to experiment with gambling semi-regularly. If 1% of that pool becomes addicted, the total adds up quickly. The cost will be like doubling the number of alcoholics in society. And it's a lot easier to gamble all your savings away then it is to drink it all away. Anecdotal - and probably why I'm harping on this in the first place - but I know more people who have ruined their lives due to gambling than due to drugs or alcohol. I think most people vastly underrate the amount of damage a gambling problem can do.

Expand full comment

That seems plausible.

Expand full comment

I expect the various state governments, and maybe the feds, will crack down on it. Mainly because they want to get their greedy fingers in and grab a slice of the pie. Due to the puritan background of America, regulators can use the association of sin to levy greater taxes. See tobacco, alcohol or marijuana products.

Also, I think advocating for the state to coerce peoples' behavior for their own good is crossing a red line that prevents you from claiming to want libertopia. Ultimately the responsibility for the actions of an individual rest with that individual and no where else. You can't make the state ban people from making poor decisions.

The full measure to stop degenerate gamblers from ruining their lives would be prohibition. We tried that here in the US a while ago with another conduit for bad addictive behavior, alcohol. It didn't work out too well. Turns out all the people who wanted to drink were just driven to the black market, which had the added benefit of funding various criminal organizations.

Even if sports betting was banned, there are plenty of other ways for degenerate gamblers to ruin their lives. Lotteries and casinos still exist. I'm sure there are plenty of risky investment firms that would love to take large sums of cash. They could bet their life savings on the latest crypto craze or tulip mania of the day.

I don't have to imagine what will happen because it has happened already with tobacco and alcohol and marijuana and other gambling before. The state imposes burdensome taxes and licensing, or outright monopolizes the field like state liquor dispensaries or lotteries. The addicts can either still have their addiction but pay several times more for it, or go to the black market and maybe get robbed or OD on fentanyl for their trouble. Advocacy groups will relentlessly campaign against the sinful practice, with practical results like outlawing sports betting adverts lower than 6 feet on store walls (think of the children!). The native reservations will use their carve outs from US laws to add bookies to their depressing lineup of ethanol free gasoline, tax free tobacco & liquor, casinos and marijuana dispensaries packed along the roadside like sardines.

At the end of the day, you are treating the symptoms and not the disease. Outlawing the outlets for people with poor decision making and impulses does not remove the poor decisions and impulses. But you do make all of these people poorer by levying higher taxes or black market fees on them. And punish all of the people who are perfectly capable of enjoying sports betting responsibly. And maybe get sloppily written laws passed that also ban prediction markets. At least you can take away freedom from these people with the approval of your own conscience, because hey, it's for their own good. Good intentions and all that.

Now that I've outlined why all of this is horrible from a libertarian perspective, I can also confidently predict that it will probably happen.

Expand full comment

"I expect the various state governments, and maybe the feds, will crack down on it. Mainly because they want to get their greedy fingers in and grab a slice of the pie."

-- it's the states that have legalized it (once the Feds cleared the way) and they've placed heavy taxes on them, so it's already going the opposite way.

Expand full comment

While you are correct that individuals are responsible for their actions and you can't make the state ban them from poor decisions, but it does not follow that the state has to "allow" or promote the entities that encourage corruption. We know that the business model is to give away a bunch of "value" (often literal "free" money) to a 100 people, in the hope that 10 of them become "normal/repeat" users, and 2-3 will become whales that pour all their money into the system. It's the same business model as drug dealers where "the first hit is free"; and I don't think drug pushers are morally upstanding citizens. While it's true that prohibition doesn't "work" in the sense of preventing a vice existing or people going to great lengths to get it, making or keeping something illegal DOES decrease the amount of that thing in society. There really was less drinking during Prohibition, and there were a lot fewer regular marijuana users before the recent wave of legalization. There's always going to be tradeoffs to any action (or inaction) the state takes, but we should clear-eyed about what those actually are when making decisions, and not be beholden to any particular ideology when assessing the "correctness" of any particular policy.

As far as I can tell, the effect of legalizing sports betting has been to transfer a little bit of money from the majority of users to a few talented bettors like Zvi Moshowitz (or whomever is your example of someone good at sports betting), and a LOT of money from the compulsives to the executives and promoters of the betting sites. It has also made watching sports DRAMATICALLY worse (at least to people like me who find the constant promotions of odds and specific sites offputting). If you had presented those tradeoffs to legislatures or the public beforehand, I don't think they would have agreed this would be a "good" decision, despite it increasing "freedom" in the abstract. Given that the consequences of the changes was bad, I think it is prudent to try to walk some of them back and discourage further expansion of the practice. Personally, I think it should only be allowed in physical casinos since that limits some of the possible "damage" of these vices through spatial and temporal segregation of maladaptive behavior. "What happens in Vegas, stays in Vegas" was used as promotion, but I think it's actually a pretty succinct policy guide as well.

Expand full comment

Politically-involved DC-area ACX readers might be interested in attending https://www.ismaglobal.org/conference Liberalism for the 21st Century. A lot of thoughtful and rational (if not rationalist) folks talking about anti-populism and how to maintain a classically-liberal democracy in modern times.

Expand full comment

The more I see the average person's "reasoning", especially in politics, the less I like democracy. For example, in Trump's impeachment trial, I heard interviews of regular people who thought Trump ought to be impeached, because they didn't like what he was doing in office. Whether you like it or not, doing things you don't like is NOT impeachable, but only high crimes and misdemeanors. It was laughable that the Senators all swore to judge based on the facts of the case, yet the vote was only along party lines.

For the record, I know of no reason Biden ought to be impeached, either, though I hear rumors that Republicans are lining up some kind of a case for it.

Unfortunately, I know of no better alternative. I've thought one ought to be able to pass a test on government operations in order to vote, but it would not only not be enough but some would complain that the test is somehow biased against their group.

Expand full comment
Jun 15·edited Jun 15

I don't know about Biden, but the Republicans already impeached Mayorkas *literally* because they didn't like what he was doing in office. Apparently that's just how politics is going to work now.

Expand full comment

Mayorkas was impeached for not doing the job Congress assigned him. I'm not a lawyer, and can't say whether that's a "high crime or misdemeanor" but I do think he ought to have been removed from office for that. In such a case, his superior is supposed to do that (which must be Biden, as he is a cabinet member).

So I suppose it ought to have been Biden that was impeached instead, for not doing what Congress said he must? Which is to say, enforcing border security. I doubt your point was to change my mind that Biden did something impeachable, but I guess you have.

Expand full comment

I agree that democracy has a significant stupid person problem, as well as a dishonest politician problem. I also don't know of anything really better. The old Churchill quote is "Democracy is the worst form of government, except for all the others", and it's right.

There are ways that I think our democracy could be improved by being less democratic, like by no longer binding electors by the state popular vote, or by having testing standards for people to become elected officials, or plenty of others. That's one of the major problems of how to maintain democracy in the face of 21st-century populism - one of the topics of the conference I was mentioning.

Expand full comment

>For example, in Trump's impeachment trial, I heard interviews of regular people who thought Trump ought to be impeached, because they didn't like what he was doing in office.

You should be prepared to read things a little bit charitably here. Did they simply not like what he was "doing in office," or did they think he was doing things in office that were in fact impeachable (they could be mistaken in that belief but still sincerely hold it)?

Joe Biden does things in office that make me want to bang my head against a wall, but I don't believe any of them are impeachable--just incredibly wrong and bad. Trump did things in office that I thought were incredibly wrong and bad, and he also did things in office that I thought were incredibly wrong and bad and impeachable. Again, I could have been mistaken in that belief, but I didn't want to see Trump impeached just because I couldn't stand him. Don't you think this could possibly be true of those "regular people"?

Expand full comment

I think your reading is correct, actually. The regular people, though, basically said, "he is doing bad things for the country, and should be removed".

I cannot now find any actual quotes: all I find in internet searches are quotes from politicians, chiefly Senators. Politicians, of course, will say what moves their agenda, forward, and though that is a problem it isn't the problem we discussed.

Which things did Trump do that you thought were impeachable? The "favor" asked of Zolinski was the one up for debate. My opinion, as objective as I can make it, is it wasn't impeachment-worthy for two main reasons: 1) it was not a demand, but a suggestion, and 2) Zolinski didn't actually do what was asked. The magnitude of the action wasn't "high".

Expand full comment

I guess I'll have to take your word for it, if you can no longer find the interviews.

I disagree with you that the "favor" "asked" of Zelenskyy was a suggestion. I thought they were pretty obviously the kind of "suggestions" made by a gangster during a shakedown. This would be fair game in the world of international relations if it were being done in pursuit of national policy (e.g., do something that helps our national interest or we'll cut off aid) but he was doing it to bolster his own campaign and smear his opponent.

But when you asked the question, I was actually thinking of the things he did to get him impeached the second time.

Expand full comment

Interesting realization: I think "Mad Men" would work just as well as a late 1940s noir drama show set in L.A. Instead of an ad firm, the workplace would be a police detective office. Instead of "clients," they would deal with criminals, victims, lawyers, and other interested parties. Some of the criminals would be high-profile people like politicians who would bribe them and get into relationships with them.

Expand full comment

Seems like a TV series version of "LA Confidential," which was a great movie. No reason I can think of it wouldn't work.

Expand full comment

How would you replace the last scene of the show were it a crime drama?

Expand full comment

I don't know. I didn't say it could be a scene-for-scene remake, but in a different setting.

Expand full comment

A mafia capo reaches a transcendent state on rye whiskey and comes up with a pitch that makes the public feel kinda good about organized crime?

Expand full comment

I would watch this and argue with other fans on the Internet about it!

Expand full comment

Vegas?

Expand full comment

The Corleone family is moving from New York to Nevada and will be completely legit in five years.

Expand full comment

Michael did have that hidden past in Sicily, with the other wife he never told anyone about...

Expand full comment

Musk in February, filing a California state lawsuit against Sam Altman and OpenAI:

"OpenAI has abandoned its ‘irrevocable’ non-profit mission in the pursuit of profit....has been transformed into a closed-source de facto subsidiary of the largest technology company in the world: Microsoft....Where some like Mr. Musk see an existential threat in AGI, others see AGI as a source of profit and power....", etc.

Musk yesterday on Xwitter: "If Apple integrates OpenAI at the OS [operating system] level, then Apple devices will be banned at my companies...."

Musk today to the court: Never mind. Nope, no reason.

[He withdrew the lawsuit without explanation.]

Expand full comment

Good? Whatever the reason, he's got more important things he should be doing than tilting at this particular windmill. And if this means less support overall for the "AI safety" agenda, that's an unambiguous win for humanity.

Expand full comment

A Memorial Day blog post, with links to a couple of devastatingly sad songs around homecomings:

Don Henry and Craig Carothers, “Schenectady” (2014)

Jason Isbell, “Dress Blues” (2006-07)

https://fragmentsintime.substack.com/p/homecomings

Expand full comment

I'm looking for a professional who helps people produce optimal resumes for tech jobs. So far I have found one person, who was recommended by someone here: Kate Williamson (scientechresumes.com). Can anyone here recommend someone else? I think it's always good to have more than one person to choose from. The job-hunter I am trying to help just completed a masters in data science at Syracuse, where he got straight A's. He has never had a data science job, though his previous job was in IT, and he has good general knowledge of computer and internet-related stuff. So the resume coach would need to be good at helping people who are just starting out. Got any recommendations?

Expand full comment

I do this. Feel free to contact me at threemillionthflower at gmail dot com.

Expand full comment

I'm delighted to hear back from somebody! Do you have a web site or something along those lines?

Expand full comment

No website unfortunately, I work mostly via word of mouth. I have years of experience editing resumes, in both tech and other fields, and have a strong record of getting people callbacks. Shoot me an email and I'll give you more details.

Expand full comment

I don't think that will work. The family member I'm doing this search for is already skeptical of the idea of having someone advise him on his resume. He would not be willing to see someone who doesn't have a website, a Linked-in or the like. I realize that those things are not proof that somebody's competent, but it's all we've got to go on.

Expand full comment

No worries. Wishing you both the best of success.

Expand full comment

Just got my first box of Mealsquares 2.0 and tried one. I think I'm slightly disappointed, although I may grow to like them just as much eventually. It was much less like a baked good and more like an energy bar, in terms of taste and texture (and also packaging). Perfectly good as energy bars go, and maybe nutritionally better than most, but there seems to be less now to distinguish it from the mass-produced brands in stores.

Anyone else have reactions?

More detail (most of this is on the web site now, although I think not every page has been 100% updated to reflect the new version):

The first ingredient is "Cassava (tapioca)", which I guess is probably ok nutritionally, although a bit confusing since tapioca is not quite the same thing as cassava (my understanding is that tapioca is refined cassava starch). Based on notes on the web site, it seems to be actual whole cassava flour, which is probably better. Oats and eggs, formerly the top two ingredients, are much further down the list.

The web site is still claiming the same percentage of calories from carbs:fat:protein as the old version, 36:44:20. I can't quite reconcile that with the amounts of each shown on the label (36g carbs, 13g fats, 15g protein). I think the percent from fat would have to be at most 40% given those amounts, although I know the "9 kcalories per gram of fat" rule is approximate and really depends on the type of fat. I don't think it's ever that far off though. I suspect they just haven't updated the numbers on the web site yet.

I think I understand the reasons for the changes (much better shelf life, more sustainable/profitable business), so I don't blame them, but I'll miss the old version.

Expand full comment

The new ones make my mouth itch and feel odd in my stomach. But the originals took some getting used to as well. I think maybe it's just that it's so different from the rest of what I eat.

Expand full comment

They sent out a follow-up email answering some FAQs, including:

- Yes, they were losing money on Mealsquares 1.0

- They will be open-sourcing the 1.0 recipe

- There will be a blog post with more details about the changes

- Use checkout code "thankyou" for $5 off a sample pack

Expand full comment

I would be paranoid about eating anything containing cassava. The plants naturally produce cyanides that can poison people if not processed correctly. Maybe I am being far too risk averse, and this is only a problem for poor subsistence farmers in the tropics whose diet is mainly cassava. But there are plenty of other starch sources out there.

Expand full comment

The boba in boba tea is tapioca starch.

Expand full comment
Jun 12·edited Jun 12

Yes, you are being far too risk-averse. The cassava grown by agribusinesses for the mass market is the non-bitter cultivar, which doesn't produce the cyanogenic glucosides you're talking about, and consequently doesn't require the time-consuming and therefore expensive processing to remove the cyanide. Poor subsistence farmers usually prefer the bitter variety because their crop is safe from wild animals, both before and after harvest, but this is less of a problem for agribusinesses, who use alternative approaches to pest control such as fences, shotguns, traps, poison, and vermin-proof containers. Consequently cassava is a staple food throughout most of America with no significant risk of poisoning. The US is an exception, mostly only using cassava in desserts rather than as a staple.

Expand full comment
Jun 12·edited Jun 12

Well, all cassava has toxic levels of cyanide for regular consumption. The bitter cultivars just have ~20x as much. The sweet cultivars still have to be peeled, washed and roasted before they are consumed. To be clear, this doesn't remove all of the cyanide either. But I'm sure there are trace amounts of all sorts of toxic compounds in foods that don't pose any medical risk.

Washing cassava takes about 5 days and uses a lot of water. It is grown in mostly very poor countries, where droughts can impose a large cost to this washing. It is not unheard of for poor workers to skip this step in hard times and poison people.

Of course this is pretty meaningless if you are buying tapioca in the US. Even if you did somehow get a bad batch it would be like a bad case of food poisoning rather than the partial blindness or paralysis of a mono-cassava diet. I don't eat raw eggs either, even though rationally I understand I could eat a raw egg every day for 15 years before I got the mean case of salmonella (actually that's just the time to eat an egg containing salmonella, which isn't the same as being infected by it.) I know this doesn't make sense from a probabilistic view, but there it is.

Expand full comment

On the other hand, read somewhere in the last year or 2 that 20% of raw chicken in the grocery stores is contaminated with salmonella.

Expand full comment

Hey all,

I'm looking to hear from people who have persistent wrist pain (e.g. during extended computer usage). I've been developing and gathering a set of methods for helping with wrist pain that is informed by predictive processing/active inference.

If you're open to sharing about your experience and curious about hearing more, you can just send a blank email with the subject line "Wrist Pain Interest" out to mxslk@mit.edu and I'll reach out for a short conversation.

Expand full comment

I used to have wrist pain, too. It went away with a combination of better mice and keyboards (lighter mice, split keyboards), keyboard trays (to lower the keyboard), temporary use of wrist braces to remove pressure, and long-term training to give myself better wrist posture while using a computer.

Expand full comment
Jun 12·edited Jun 12

I used to have wrist pain and the 3M ergonomic mouse made it go away. (Though it's a bit annoying that it doesn't have a scroll wheel)

Expand full comment

If you haven't already, consider posting in forums and spaces with lots of art professionals (esp digital art, esp animation) - apparently that's a very common ailment.

Expand full comment

Thanks! Do you have any particular recommendations?

Expand full comment

I haven't listed to the full Dwarkish podcast with Aschenbrenner yet but he's talking about AI companies needing to build giant power plants in order to continue to scale up. He says by 2028, we'll have a 10 GW plant (more power than most US states consume) which will cost $100 billion, and by 2030 it will be a 100 GW plant at the cost of $a trillion. My question is: what fuel source is being imagined/planned for these power plants? I'd guess natural gas would be the most practical? Anyone have a better guess?

Expand full comment

Maybe we'll get lucky and these companies will put a few bucks into advocating permitting reform so that the plant doesn't have to cost $1 trillion.

Expand full comment

If we run them off of solar power, we can shut down AIs by blasting the atmosphere, as in the Animatrix. Maybe this is how the Terminator franchise worked: Skynet reigns supreme during the day, but in the night half of the world, it's restricted to crude automation with low intelligence, allowing humans a fighting chance.

But more seriously, I hope nuclear power is being considered for this stuff.

Expand full comment

Thanks. So natural gas produces about half as much CO2 as coal. If we really got a 100 GW gas plant by 2030, that sounds like a huge percentage growth in CO2 emissions for the USA. Some of it could be offset, but it's hard to imagine much of it will be.

I'd think if the Democrats retain power, they would likely restrict building such large fossil fuel power plants, particularly if there is negative sentiment against AI. Then again, maybe winning the AI race is like the old Space Race and the US government will dump money into the project in order to defeat China regardless of environmental costs.

Expand full comment

I pray to god we don't end up becoming overly-cautious luddites and voluntarily cede our position as #1 superpower to someone less paranoid and more evil (China). That would be one of the greatest tragedies in human history. I hope our overlords are a bit more realpolitik than that. Whatever the risk of AI is, AI + China is worse.

Expand full comment
founding

This is the usual request that "Evil China gonna have AI real soon now!" be backed up with evidence and reasoned argument. I'm old enough to have seen Evil China conspicuously fail to do a whole lot of things it was going to do Real Soon Now for decades.

Expand full comment

Evil China has done enough evil this century to earn their laurels. You disagree?

Expand full comment

The thing in question isn't about how evil China is, but how able China is to do the things being predicted.

Gwern has had an open invite for people claiming Chinese supremacy in AI to post a paper that is 1. Original, and not just copying existing American State of the Art 2. Has had as much impact as an important American paper.

So far, what he's gotten is mostly labs saying that they have used existing techniques to catch up to what America did a couple of months ago, and a couple of papers that suggest minor improvements in performance.

Do you have a counter example?

Expand full comment

"they probably won't do it so we shouldn't either" is not a good argument

Expand full comment
founding

If "it" is e.g. building a Strangelovian doomsday device and setting it with a hair trigger, then yes, "they probably won't do it so we shouldn't either" is in fact a very good argument.

But on the subject of bad arguments, "They are evil, this thing is evil, therefore they will do this specific thing", certainly qualifies.

"They" may be evil, but they might not be able to do this specific thing or they might be prioritizing other evil things. We should pay attention to what they are actually doing, not do all the evil things ourselves just to make sure they don't do them first.

Expand full comment

Figure out a way to turn online rage and hatred into electricity.

Expand full comment

Speculatively, solar + batteries: https://austinvernon.site/blog/datacenterpv.html

Of course, you'll need a lot of land.

Expand full comment

Please show me some poem you enjoy. (Poetry in any language is welcome.)

Expand full comment

I've always been partial to Frost's "Stopping By Woods On a Snowy Evening". It's not the most interesting or spicy poem. But I like the cadence and the atmosphere of it. And the theme of tiredness resonates with me (or at least that's what I think it's about).

https://www.poetryfoundation.org/poems/42891/stopping-by-woods-on-a-snowy-evening

Expand full comment

His squinched eyes through tiny lenses enlarged,

Pen held poised over documents awaiting seal,

Regarding harshly the man who through his door barged,

Shouting angrily about not a terribly great deal -

A document arrived in Department Three improperly signed,

Necessary to the processing of documents in Department Two,

And the pile-ups thorugh each department were lined,

Until the only concern was a demand for a who -

The person who had signed above the dotted line

Rather than properly and righteously below.

Department Twenty-Two had already assigned a fine,

And Department Forty-Nine established to know -

How a document is to be properly marked,

How and when to dot an i or cross a t -

But Department Forty Three (How cars are to be properly parked)

About a particular memo could not agree:

How to sign the inter-Departmental note -

And here the shouting man was overspoken,

"I signed above the dotted line as my instructions denote."

For this the barging man looked startled and awoken,

Looking uncertain whether to shout or to flee,

As he looked upon the head of Department Three.

Expand full comment

A! Elbereth Gilthoniel!

silivren penna míriel

o menel aglar elenath,

Gilthoniel, A! Elbereth!

Expand full comment

Plenty in Arabic, but I estimate nearly zero non-me Arabic speakers on the forum.

The Lingua Franca of the Internet still doesn't disappoint:

[1] https://poets.org/poem/merchant-venice-act-iv-scene-i-quality-mercy-not-strained

The quality of mercy is not strained;

It droppeth as the gentle rain from heaven

Upon the place beneath. It is twice blest;

It blesseth him that gives and him that takes:

'T is mightiest in the mightiest; it becomes

The throned monarch better than his crown:

His sceptre shows the force of temporal power,

The attribute to awe and majesty,

Wherein doth sit the dread and fear of kings;

But mercy is above this sceptred sway;

It is enthronèd in the hearts of kings,

It is an attribute to God himself;

And earthly power doth then show likest God's

When mercy seasons justice. Therefore, Jew,

Though justice be thy plea, consider this,

That, in the course of justice, none of us

Should see salvation: we do pray for mercy;

And that same prayer doth teach us all to render

The deeds of mercy. I have spoke thus much

To mitigate the justice of thy plea;

[2] https://www.youtube.com/watch?v=hS_AXRRnIzM

Extract:

And the letter read,

"""Dear Empty Head,

Ye who dwells on ocean's bed

I send this by the cracked moon's lights

It likely be the last I write

For I will take soon flight

For a long journey's while

To the Ninth Isle

Remember me as I will you

All tree houses and untied shoe

In hope, someday we meet again

Your distant kin: Sebestian."""

But... a farewell by letter

Is little better

Than no farewell at all

So, on a young man's whim

For my grandfather, I set off

In search for him.

Expand full comment

Has to be Yeats:

VI—The Stare’s Nest By My Window

The bees build in the crevices

Of loosening masonry, and there

The mother birds bring grubs and flies.

My wall is loosening, honey bees

Come build in the empty house of the stare.

We are closed in, and the key is turned

On our uncertainty; somewhere

A man is killed, or a house burned,

Yet no clear fact to be discerned:

Come build in the empty house of the stare

A barricade of stone or of wood;

Some fourteen days of civil war;

Last night they trundled down the road

That dead young soldier in his blood:

Come build in the empty house of the stare.

We had fed the heart on fantasies,

The heart’s grown brutal from the fare,

More substance in our enmities

Than in our love; oh, honey-bees

Come build in the empty house of the stare.

Expand full comment

Mirror in February by Thomas Kinsella

The day dawns, with scent of must and rain,

Of opened soil, dark trees, dry bedroom air.

Under the fading lamp, half dressed - my brain

Idling on some compulsive fantasy -

I towel my shaven jaw and stop, and stare,

Riveted by a dark exhausted eye,

A dry downturning mouth.

It seems again that it is time to learn,

In this untiring, crumbling place of growth

To which, for the time being, I return.

Now plainly in the mirror of my soul

I read that I have looked my last on youth

And little more; for they are not made whole

That reach the age of Christ.

Below my window the wakening trees,

Hacked clean for better bearing, stand defaced

Suffering their brute necessities;

And how should the flesh not quail, that span for span

Is mutilated more? In slow distaste

I fold my towel with what grace I can,

Not young, and not renewable, but man.

Expand full comment

The Broken Doll by Nuala Ní Dhomhnaill, translated to English by John Montague

The Broken Doll

O little broken doll, dropped in the well,

thrown aside by a child, scampering downhill

to hide under the skirts of his mother!

In twilight’s quiet he took sudden fright

as toadstool caps snatched at his tongue,

foxgloves crooked their fingers at him

and from the oak, he heard the owl’s low call.

His little heart almost stopped when a weasel

went by, with a fat young rabbit in its jaws,

loose guts spilling over the grass while

a bat wing flicked across the evening sky.

He rushed away so noisily and ever since

you are a lasting witness to the fairy arrow

that stabbed his ear; stuck in the mud

your plastic eyes squinny open from morning

to night: you see the vixen and her brood

stealing up to lap the ferny swamphole

near their den, the badger loping to wash

his paws, snuff water with his snout. On

Pattern days people parade seven clockwise

rounds; at every turn, throwing in a stone.

Those small stones rain down on you.

The nuts from the hazel tree that grows

to the right of the well also drop down:

you will grow wiser than any blessed trout

in this ooze! The redbreasted robin

of the Sullivans will come to transform

the surface to honey with her quick tail,

churn the depths to blood, but you don’t move.

Bemired, your neck strangled with lobelias,

I see your pallor staring starkly back at me

from every swimming hole, from every pool, Ophelia.

Expand full comment

There are some qualities—some incorporate things,

That have a double life, which thus is made

A type of that twin entity which springs

From matter and light, evinced in solid and shade.

There is a two-fold Silence—sea and shore—

Body and soul. One dwells in lonely places,

Newly with grass o’ergrown; some solemn graces,

Some human memories and tearful lore,

Render him terrorless: his name’s “No More.”

He is the corporate Silence: dread him not!

No power hath he of evil in himself;

But should some urgent fate (untimely lot!)

Bring thee to meet his shadow (nameless elf,

That haunteth the lone regions where hath trod

No foot of man,) commend thyself to God!

Silence by Edgar Allan Poe

Expand full comment

Man doth usurp all space,

Stares thee, in rock, bush, river, in the face.

Never thine eyes behold a tree;

‘Tis no sea thou seest in the sea,

‘Tis but a disguised humanity.

To avoid thy fellow, vain thy plan;

All that interests a man, is man.

-Henry Sutton

Expand full comment

That one makes me quote Gerard Manley Hopkins as reply:

God’s Grandeur

The world is charged with the grandeur of God.

It will flame out, like shining from shook foil;

It gathers to a greatness, like the ooze of oil

Crushed. Why do men then now not reck his rod?

Generations have trod, have trod, have trod;

And all is seared with trade; bleared, smeared with toil;

And wears man’s smudge and shares man’s smell: the soil

Is bare now, nor can foot feel, being shod.

And for all this, nature is never spent;

There lives the dearest freshness deep down things;

And though the last lights off the black West went

Oh, morning, at the brown brink eastward, springs —

Because the Holy Ghost over the bent

World broods with warm breast and with ah! bright wings.

Expand full comment

As into the garden Elizabeth ran

Pursued by the just indignation of Ann

She trod on an object that lay in the road

She trod on an object that looked like a toad.

______

It looked like a toad and it looked so because

A toad is the actual object it was

And after supporting Elizabeth's tread

It looked like a toad that was visibly dead.

---------

Elizabeth, leaving her footprint behind,

Continued her flight on the wings of the wind

As Ann in her anger was heard to arrive

At the toad that was not any longer alive.

____________

She was heard to arrive for the firmament rang

With the sound of a scream and the noise of a bang

As her breath on the breezes she broadly bestowed

And fainted away on Elizabeth's toad.

____________

Elizabeth, saved by the heel of her boot

Escaped her insensible sister's pursuit

And if ever hereafter she irritates Ann

She will tread on a toad if she possibly can.

Expand full comment

...you've brought this on yourself.

https://dnowmects.wordpress.com/2024/05/12/the-bone-of-boundless-boon/

Expand full comment

I don't get the ants. :-(

Expand full comment

The ants speak in antonyms.

Expand full comment

SONG, by Roger Zelazny:

---

When I learned the other day

that everything Emily Dickinson wrote

can be sung to the tune

of 'The Yellow Rose of Texas"

I was crushed.

It was true.

I can no longer read Emily Dickinson

but Lone Star ghosts flit across the page,

the Alamo is not forgotten

and I hear the thundering hoofbeats

of the great horse Silver.

I wondered then

whether every person who pens a poem

has a tune,

a secret melody which will destroy him

if the word gets out.

A small thought, perhaps,

not quite as profound as it sounds;

and those who fool with vers libre

should be safer than most.

Yet the notion nags.

There's an awful lot of music in the world.

To be trapped by John Cage

or crushed by Leadbelly

would be bad enough.

But I have this nightmare

of being done in by a hymn.

If Rock of Ages gets me in the end,

mocked Emily's diamond eyes

may sparkle like the dew

in stillnesses that lie

between the words and the Word.

Expand full comment

"When I learned the other day

that everything Emily Dickinson wrote

can be sung to the tune

of 'The Yellow Rose of Texas'

I was crushed.

It was true."

If this is true, then "I learned from Achewood that since this poem is in ballad meter, it can be sung to the tune of Gilligan's Island." (https://www.explainxkcd.com/wiki/index.php/788:_The_Carriage title text)

Expand full comment

And Robert Frost's "The woods are lovely, dark and deep..." can be sung to the tune of Hernando's Hideaway.

Expand full comment
Jun 12·edited Jun 12

Patrick Kavanagh's "On Raglan Road" was turned into a song by Luke Kelly, using the air "The Dawning of the Day":

https://www.youtube.com/watch?v=ZIqr1Ge8Z5w

And a song that inspired a poem that was turned back into a song is "Down by the Salley Gardens" by Yeats:

https://en.wikipedia.org/wiki/Down_by_the_Salley_Gardens

https://www.youtube.com/watch?v=9eU7hKQi4qA

https://www.youtube.com/watch?v=027ZJX5XVjs

Expand full comment

Thanks, love that song. Reminds me of another Yeats favorite -

I went out to the hazel wood,

Because a fire was in my head,

And cut and peeled a hazel wand,

And hooked a berry to a thread;

And when white moths were on the wing,

And moth-like stars were flickering out,

I dropped the berry in a stream

And caught a little silver trout.

When I had laid it on the floor

I went to blow the fire a-flame,

But something rustled on the floor,

And someone called me by my name:

It had become a glimmering girl

With apple blossom in her hair

Who called me by my name and ran

And faded through the brightening air.

Though I am old with wandering

Through hollow lands and hilly lands,

I will find out where she has gone,

And kiss her lips and take her hands;

And walk among long dappled grass,

And pluck till time and times are done,

The silver apples of the moon,

The golden apples of the sun.

Expand full comment
Jun 11·edited Jun 11

So the rumour mill says that the Trump campaign has sent out vetting documents to a shortlist of four potential VPs: Rubio, Burgum, Tim Scott and J. D. Vance.

For my money, Burgum (of whom I was a fan of early in the primary) seems like easily the best choice, disregarding any cynical plays for specific demographic groups.

Rubio is not a reliable Trump supporter. Scott is too religious. Vance is... phenotypically offputting. But Burgum seems smart, has both insider and outsider cred, he's built a business and run a state, doesn't have any outside-the-mainstream opinions that I know of, and seems likeable and comes across well in interviews. Also, "Trump/Burgum" sounds good coming off the tongue

So far I haven't heard any good arguments against Burgum. The NY Times managed to put together a hit piece the other day entitled "Trump’s Energy Guy Talked a Green Game but Now Sells Big Oil Priorities" which I suppose is a preview of at least one attempted line of attack, but this feels pretty pissweak.

Expand full comment

Scott is such a nice, sunny presence that I think he might be good to have. Optimism, kindness, decency are really refreshing these days, and a lot of people miss these in politics. I wouldn't know if this would swing voters, though.

The real question would be, how would he manage if he had to take over? But anyone who might consider voting for Trump knows that Scott would be enormously better than the people currently in charge.

Expand full comment

When I watched the first two Republican debates, I liked him a lot, but thought he didn't have enough of a track record, and needed a bit more seasoning. I'm not sure if being VP for Trump would help, or if Scott would wind up being tarnished by association.

Expand full comment

>Scott is too religious.

Scott's religiosity is what makes him attractive for the Trump ticket. Trump needs the evangelical vote, and while he's managed to hold on to most of it there are a lot of evangelicals who are unsure of Trump. He's not a very Christian guy, what with the boasting, insults, and affairs. Pence played this role for him in 2016, helping to reassure skittish evangelicals that Trump would be okay: if Pence is on board, he can't be all that bad.

Of course the question is whether Trump needs that kind of boost now that he's a "proven brand". But my point is that Scott's piousness is why he's being considered at all, it's not a liability but an asset.

Expand full comment

A lot of Evangelicals vocally opposed Trump in 2016, I haven't heard much about it this year. It's not like they have anywhere else to go politically. I think a lot of them accepted that after they lost three Republican primaries in a row (2008, 2012, and 2016) insisting on ideological purity was a dead end. Trump's bigger issue is socially moderate voters in the midwest.

Expand full comment

I'm not saying here that there's no reason for evangelicals to not like Trump. I don't like him, if nothing else because I don't want the GOP to turn into an electoral monarchy where if a guy wins one nomination he's thereby entitled to be the nominee in every election from then on until his death.

But for evangelicals to reject him because he's "not very Christian" would be the height of political stupidity. When you're a minority, as evangelical Christians are, refusing point blank to vote for someone because he's not part of your group is a surefire ticket to political irrelevancy as a minority with no allies. Which is why those demanding evangelicals do this are mostly liberals and evangelicals who are employed at liberal publications.

Expand full comment

To be blunt about it, a lot of the people I see calling on Evangelicals that "Trump is not a Christian, look at his sinfulness!" and so they should reject him, are the same people otherwise spitting on Evangelical values as "those moral majority christian nationalist far-right fascist theocrats", so you know - better the guy who got Roe vs Wade overturned than the other lot.

Expand full comment

It’s not that simple. Nothing ever is. Thoughtful, conservative Christians are baffled too.

Conservative Evangelical David French has a short article here.

https://www.nytimes.com/2024/06/09/opinion/presbyterian-church-evangelical-canceled.html?unlocked_article_code=1.zE0.Uo-j.ggizRzdbJ8OG&smid=nytcore-ios-share&referringSource=articleShare&sgrp=c-cb

Expand full comment

David French has been writing basically the same article for five years straight now. From the article:

"The instant my participation was announced, those attacks started up again. There were misleading essays, vicious tweets, letters and even a parody song directed at the denomination and at me. The message was clear: Get him off the stage."

French's example of a "vicious" tweet is this, which is quite tame by Twitter standards: https://x.com/William_E_Wolfe/status/1790461068436701261. There are no threats, insults, or comments on his looks or his family, just someone expressing their desire not to associate with David French. He was a big cheerleader for kicking people off of social media back in the day.(It's a private company and all that.) Guess it's different when he's the guy getting cancelled.

Expand full comment

I don't think he's very strongly conservative or Evangelical; he's moved to being Independent from the Republicans and moved from PCA to - where? "So we left for a wonderful multiethnic church in Nashville."

Looking at Wikipedia, he's squishy on some things if you just apply enough pressure:

"In August 2017, French was one of several co-authors of the Nashville Statement, which affirmed "that it is sinful to approve of homosexual immorality or transgenderism and that such approval constitutes an essential departure from Christian faithfulness and witness." The statement was criticized by pro-LGBT Christians and LGBT rights activists, as well as by several conservative religious figures.

In November 2022, French announced that he had "changed his mind" on the legal recognition of same-sex marriage, although stating he was still morally opposed to the matter."

Yeah, as a Catholic, I've seen my share of "personally opposed but it's legal" guys on topics like abortion, where they don't stand one inch in the way. Might as well be personally for it, in that case. Give him a couple of years, let his theology mature and catch up to his legal views, and he'll be hanging out the rainbow flags at his wonderful multi-ethnic church.

It's definitely not simple, and if he and his family got the kind of attacks he claims (though I actually genuinely wonder about the bit where the teacher allegedly asked his son about did they get the adopted child for a loaf of bread), that's bad. However, unless and until French and those like him change their views to be pro-LBGT+ as in "it's perfectly normal and fine" and not just "personally opposed but it's legal so I won't oppose it in public", then as far as the progressive set are concerned, they're every bit as bad as the far-right knuckledraggers he describes in that NYT opinion piece.

It's a choice between two evils, and depending what you consider the greater evil, you'll go with "hold my nose and Trump" or "hold my nose and Biden".

Expand full comment

J D Vance? Forget his phenotype, look at his ideology. This guy is out there. He is beyond MAGA. To the New Right, Trump is just a tool, a useful idiot.

J D Vance and the New Right:

https://www.politico.com/news/magazine/2024/03/15/mr-maga-goes-to-washington-00147054

Thiel, Vance, Yarvin and the New Right:

https://www.vanityfair.com/news/2022/04/inside-the-new-right-where-peter-thiel-is-placing-his-biggest-bets

Bronze Age Pervert and J D Vance:

https://www.politico.com/news/magazine/2023/07/16/bronze-age-pervert-masculinity-00105427

So you see in The Gorgias, Plato *intentionally* had Socrates make a weaker argument than Callicles when debating against tyranny. So there you have the hidden meaning for those wise enough to see it. And all those people like James Madison and Thomas Jefferson just weren’t smart enough to pick up on it! Oh, FFS. BAP is a flippin’ troll.

Expand full comment

So...Burgum to try to lock in North Dakota?

He does seem like a "safe" choice, but traditionally the vice presidential candidate should help the ticket somehow. I'm not sure how relevant Trump might find that concept to his campaign, but he should.

By this logic, he ought then to pick Rubio, to attempt to lock in Florida, but that does have its own problems

What the vice president will actually DO ought to be completely irrelevant, if Kamala Harris is any indication.

Expand full comment

The geographical thing seems overplayed. The last VP who came from a meaningful swing state was Al Gore, and even he would have defied conventional geographical VP pick wisdom by being from the same general region as his President.

Quite a few recent failed VP candidates from swing states though. Tim Kaine, Paul Ryan, and John Edwards were all good picks from a geographical point of view but lousy picks overall.

Expand full comment

I would think it should be possible to find some reasonable candidate, for whichever party, in searching a whole state. It is, however, an assumption.

Expand full comment

> traditionally the vice presidential candidate should help the ticket somehow.

Traditionally their loyalty didn't have a significant effect on who the next president was. :-(

Expand full comment

Vance? Erm. I don't care about the phenotype, I grew up with it, y'all are the freaks. But the guy seems like a political chameleon, and I don't trust him when his incentives aren't obvious.

Expand full comment

Can you elaborate on “phenotypically offputting?” I’m not familiar with any of these candidates, but online images of Vance seem to depict a pretty normal-looking guy.

Expand full comment

I was also confused by that. I looked up pictures of him, expecting him to be very ugly or something from that description.

Expand full comment

I dunno, he just screams "date rapist" to me for some reason.

Expand full comment

I think you're going by his eyes. To me, now having looked up pictures of him because I had to discover what this phenotype is, he looks more metrosexual - he's very groomed.

I agree he's got the stereotypical round "hick from the sticks" face (I've got one myself thanks to solidly peasant ancestry) so he needed the beard to give his face some definition, and a beard needs to be groomed, but he looks a little *too* groomed. I do think it's the eyes; he seems to have naturally dark and thick eyelashes which make him look as if he's wearing guyliner.

Unless he really *is* wearing guyliner 😁

Expand full comment

He kinda' looks like a hick (fat face) and is (relatively, for presidential candidates) short.

This might seem to you like a r-worded way of judging a leader, but it seems like it does matter to a lot of people, for different reasons. Iconically, Matthew Yglesias predicted early, at the peak of his hype, that Ron DeSantis would never be president because he's too short.

EDIT: a google search that I should've done before posting says Burgum isn't much taller, though he looks like he would be.

Expand full comment

I thought the one with the nicest hair is the winner.

Expand full comment

"I thought the one with the nicest hair is the winner."

Personal opinion on the Great Coiffure Contest, going off photos of all four from the past week:

Marco Rubio - hair is rated "okay". It's not great, it's not terrible. Possibly thinning a little, but that may just be flyaway strands and a bad angle.

Doug Burgum - hair is rated "it depends". Certainly has lots of character, but unless well-maintained it can easily go from "silver fox" to "scruffy"

J.D. Vance - hair is rated "highly groomed". Best hair so far, but that may also be a liability; does he look as if he spends *too* much time in the barbers? Indeed, does he look like he doesn't go to a barbers, but to a hairdresser?

Tim Scott - hair is rated "dark horse" as he's bald. Clearly he can't be placed in "nicest hair" category, *unless* he pulls off a total reversal that the best hair is no hair!

So, by ranking:

(1) Vance

(2) Burgum

(3) Rubio

Scott - dark horse

Expand full comment
Jun 11·edited Jun 11

I thought that was Vance's advantage - "looking like a hick" - as he made his name with that book explaining hicks (and Trump's win) to the Blue Tribe?

Looking him up on Wikipedia, I see he converted to Catholicism. So that's two religious people, him and Burgum, as possible VPs.

EDIT: Sorry, I see you said it was Tim Scott who was too religious. Burgum is anti-abortion (going by Wikipedia) so that probably counts as "too religious" as well?

Expand full comment

three of them are senators? picking a senator (who would have to resign his senate seat if elected) seems like an unforced error when senate control is likely to be in play.

Expand full comment

Florida, South Carolina, and Ohio all have Republican governors and Senate vacancy laws that allow the governor to appoint replacement Senators who serve out the remainder of the term.

Expand full comment

I can't believe the rumours, but J.D. Vance? I sort of wish this was true, because it would be so crazy 😀

Expand full comment

So I've made the mistake of looking at the comments sections of a few outputs over the last few weeks - mostly Substacks, but also some videos, and while I don't want to claim it's evenly spread at all, I've been struck but how *freaking bonkers* some of them are. Youtube comments fluctuate between slavish devotion and praise and unhinged abuse (skewing to the former, presumably because the latter get deleted), even on very technically- or knowledge-focused works. Substacks, on the other hand, tend to bring in weapons-grade bonkerite, skewing entirely to downright conspiracy-theories and/or truly extreme political views.

My point is less about comparing different platforms in the kind/degree of their nutso comments, but more it raises the question of how common these kind of whacko ideas actually are. Granted, the more extreme ideas are going to be overrepresented in places with light or compatible moderation - I wouldn't expect to see a fascist rant lasting long on a strong social justice blog - but the degree/commonality of the whacko comments seems higher than I'd naively expect from a generally sensible, intelligent, compassionate person (which most people are, within one or two standard deviations - not everyone's an EA with a Masters degree, but few people think torturing puppies is a good idea).

I find my thinking on this difficult to parse and straighten. Be interested in people's viewpoints.

Expand full comment

Shouldn't content moderation be a killer app (sry) for LLM's?

Expand full comment

Comment moderation is a huge piece of this. Many years ago, the Reason website was a great place to discuss libertarian ideas. But they never took any action to clean up troll or spam bot comments, let alone the general muck of low quality internet comments. People who were interested in actually exchanging information fractured off into other websites and blogs, to the point that only the lowest denominator was left at Reason.

Substack definitely attracts some nut jobs, maybe because it was the right place at the right time when Facebook/Twitter stepped up their censorship of "misinformation." I follow a substack author who wrote a series about the history of maneuver warfare. That was it, an overview of military tactics from the ancient period to today. He wrote long form articles backed up by a hefty bibliography that probably took an hour to read on average. Not the type of stuff that you would think of as attracting loons. But the second he got to WWII, suddenly the comments are flooded with people arguing that the SS never rounded up and killed those people, no one ever found bodies massacred by the Nazis, the pictures of the concentration camps were doctored, etc.

Like most people, I don't think that substack author had the time or inclination to moderate the comments on his articles. I think the way it works is something like this:

1. People in charge of blog/substack don't moderate comments

2. A small fraction of commenters are nuts/high quantity, low quality posters

3. Normal commenters see this junk and it makes them less likely to interact with the space

4. Nuts/junk comments take up more and more space, driving others away and reinforcing the low quality atmosphere

Really, I am amazed by how much better ACX is with pretty minimal visible work by Scott. Of course the average ACX commenter isn't the average substack commenter, but still.

Expand full comment

Maybe part of it is that Substack encourages longer comments, and personal blogging, and so there's a greater opportunity to get into the details of whatever bonkers ideas we have? YouTube doesn't encourage long analyses in its comments, although I've seen a few.

Plus there's a problem with the genre of "insight porn", which is that not all insights are equally valid, but people have a natural desire to imitate those we like. So some people imitate the pattern "controversial and counterintuitive thoughts about the world", but fail to imitate the "valid" part, and that's not even getting into the question of whether the original was valid in the first place.

Expand full comment

I was was thinking about this the other day, as well: there is a particular archetype in substack which I'll call the "Multiple top level commenter" (in case you feel attacked, I think multiple top level comments in Open Threads are an exception) , and I was having trouble because my first reaction to such a character is to immediately dismiss them as a loon, but there is value on caring about an issue more than anyone else, particularly in Substack where probably most commenters are pretty intelligent.

Expand full comment
Jun 11·edited Jun 11

"not everyone's an EA with a Masters degree, but few people think torturing puppies is a good idea"

From "The Man Who Was Thursday":

"The applause that had greeted the opening sentences had been gradually growing fainter, and at the last word it stopped suddenly. In the abrupt silence, the man with the velvet jacket said, in a high, squeaky voice—

“I’m not meek!”

"... I repeat, we are the true early Christians, only that we come too late. We are simple, as they were simple — look at Comrade Witherspoon. We are modest, as they were modest — look at me. We are merciful —”

“No, no!” called out Mr. Witherspoon with the velvet jacket.

“I say we are merciful,” repeated Gregory furiously, “as the early Christians were merciful. Yet this did not prevent their being accused of eating human flesh. We do not eat human flesh —”

“Shame!” cried Witherspoon. “Why not?”

Expand full comment

I have absolutely no idea what this means or has to do with anything.

Expand full comment

Great scene. "I say comrade Gregory is unfit to be Thursday, for all his amiable qualities. I say comrade Gregory is unfit to be Thursday, BECAUSE of his amiable qualities"

Expand full comment

Comrade Witherspoon would definitely be "I'll torture a puppy!" 😀

Expand full comment

There's a selection effect involved in putting in the effort of writing a comment; why bother unless you feel strongly about the content (positively or negatively)? Hence it attracts cranks of every variety who tend feel strongly about things in general. There's a great post from the subreddit that goes into this; https://www.reddit.com/r/slatestarcodex/comments/9rvroo/most_of_what_you_read_on_the_internet_is_written/

Expand full comment

Yeah, this.

But also, I wonder what Substacks and YouTube videos @Alan Smith is consuming, since this doesn't describe my own experience of comments on Substack and YouTube.

Perhaps in the words of The Last Psychiatrist, "If you're reading it, it's for you."

Expand full comment

In terms of YouTube, it's a decently wide range but still pretty 'norm-y' I think: Philosophy Tube (well, until recently), Paul Harrell, The Slo-mo Guys, Surveillance Report, T90Official is a fairly representative sample.

In terms of Substack, it's a bit more homogeneous, but for the phenomenon I'm thinking of it's particularly pronounced on After Babel and similar.

(Obviously this is not intended as in any way a slight against those writers or creators.)

Expand full comment

Oh man, you should log out of your YouTube account, clear cookies, and visit the front page of the site when it doesn't know you're you.

Although be warned this might destroy your faith in humanity and/or cause existential dread.

I also find it fun to look over people's (friends, family, etc) watch histories / channel subscriptions and marvel about the BIG HUGE *GIANT* continent-sized "corners" of the internet that I was utterly unaware could even exist.

I know who Philosophy Tube is (I think I've seen part of one video), and I think the slow-motion guys I'm thinking of are the same Slo-mo Guys you mentioned, and I probably saw them do something with shooting a bullet at a bullet or something.

Do you know who Safiya Nygaard is?

Expand full comment

Oh, I rarely if ever log in anymore. The overwhelming majority of the time I'm watching via Newpipe.

Oddly enough, the opening page doesn't load anything for me, I assume because of UBlock Origin :P But I've seen it enough on other people's systems to be... honestly kind of confused by what is apparently popular. It's not that it's *bad*, just... odd.

No, never heard of Safiya Nygaard. Interesting?

PT was quite interesting, even if I rarely agreed with them I found their viewpoint intriguing. But their recent videos over the last six months or so have been trending pretty steeply downhill - analysis is lacking, and more and more blatant political propaganda, even in places it doesn't fit. When I got to the point I was pausing every couple of minutes (or sometimes multiple times a minute) to note all the ways their points were fatally flawed, I figured it was time to give up and move on. But a lot of their back catalogue is still honestly really good stuff IMO.

(Their most recent video they uncritically repeated the claim that if someone criticizes a notoriously difficult to understand and controversial philosopher, they are anti-semetic, because some of the things done by some of the people who criticise them kind of look like anti-semetism if you squint. That was pretty representative of the level of analysis on display that entire video.)

Expand full comment

Yeah, UBlock is probably preventing you from seeing YouTube's native homepage. You should still consider opening in a different browser or temporarily disabling the plug in or something, just to see how different "generic YouTube" is from *your* YouTube.

I mentioned Safiya Nygaard because she's one of the more famous "YouTubers;" over ten million subscribers with almost two billion total views. When she releases a video, it usually trends somewhere in the "Top 15," at least for a little while. This is the video that made me a fan: https://www.youtube.com/watch?v=IPyA8CGk86A

I also picked her because, based on what content of yours I read here, I was pretty confident YouTube had never recommended her to you!

Expand full comment

I worry that this is something of a self-fulfilling prophecy. Sometimes people fail to find a worthy community, and sink to merely the highest level they've come across.

Expand full comment

Laws I'd like the US Congress to enact #3 (not gonna happen).

Respect for judges.

Criminalizes any public insulting, name-calling and/or threatening judges.

(May violate the 1st Amendment. Supreme Court would have to decide. Supreme Court consists of judges, who have an obvious conflict of interest and therefore must recuse themselves, leaving nobody to decide. Oops. Then what happens?)

Expand full comment

Is there a problem this is supposed to solve?

Expand full comment

Pretty sure threatening them is already illegal.

"Criminalizes any public insulting"

OK, so if John Smith believes that a judge is incompetent or corrupt they're not allowed talk about it? If they believe judges Michael Conahan and Mark Ciavarella are awful people (cash for kids scandal) they're not allowed talk about it?

Expand full comment

Why? What purpose does this serve that justifies such a cultural intervention? I'm referring to the insulting part, not the threatening part.

And how would this even be accomplished? In my own country of Australia, judges wear far more archaic clothing than their American counterparts: powdered wigs and the like. Ostensibly this is to foster a sense of respect and cultural distinction and hierarchy. In practice I find it has the opposite effect, at least for me personally.

And to me, this doesn't even seem like a bad thing. By dressing up our judges as 18th century aristocrats, it helps remind you that they are, in fact, human. They are part of the law, but they are not the law itself. The American model, where judges look far more like modern day professionals, feels like it gives them a sense of seriousness that Anglo countries don't command.

Expand full comment

Laws I'd like the US Congress to enact #2 (not gonna happen).

Medical bodily autonomy.

All medical treatments (including those dealing with reproduction, sexuality, and end of life) are decided and regulated solely by the patient and their doctor. Any existing laws which restrict these treatment decisions are declared null and void.

(Needs clarification of "patient/patient's guardian" and "doctor/medical team".)

Expand full comment

"All medical treatments (including those dealing with reproduction, sexuality, and end of life) are decided and regulated solely by the patient and their doctor."

The problem/loophole I see is, the government can't intervene, but does it get to define who gets to become and remain a doctor?

Expand full comment

What if the patient is on drugs? Most especially, a lot of painkillers?

Expand full comment
Jun 11·edited Jun 11

So this either bans vaccines or allows female genital mutilation for kids.

Expand full comment

I don't understand this. The proposed law would prohibit governments from requiring vaxes, and prohibit governments from prohibiting vaxes.

I don't think any doctors perform female genital mutilation.

Expand full comment

> The proposed law would prohibit governments from requiring vaxes, and prohibit governments from prohibiting vaxes.

I brought up vaccines because one way your proposal could work is "any not-absolutely-necessary procedures are left to the patient to decide, and since 1-year-olds can't decide, there won't be any vaccines until they can". Instead, you're apparently leaving it completely in the parents' hands. Which makes the FGM example one reason to object to your law.

And of course there will be doctors who would perform it, if they get paid enough, possibly if they are from the same cultures that like their FGMs. Maybe there won't be many of them, but some will be enough, I don't think it takes long to perform.

I now realize it's not such a strong objection since nothing about your proposal says you automatically excuse any harm done by the legal guardians to their ward if it has the appearance of a medical procedure. So even in the simplified case where the doctor bears zero responsibility and is considered a tool, the parents still have mutilated their child and are culpable. Just like euthanizing a healthy child would be considered a murder.

Expand full comment

If prohibiting governments from requiring vaxes resulted in fewer people getting recommended vaxes, I think that would be a regrettable tradeoff. Libertarians might be happier.

Perhaps I should have said "medical procedure" instead of "medical treatment". I do not think FGM is a medical treatment/procedure, and I would hope the AMA agrees. I would hope that any doctor that performs FGM gets expelled from the AMA. I would hope that those 2 "I would hope"s would mean that my proposed law would not affect the illegality of FGM.

Expand full comment

I assure you there are doctors in the world who would perform FGM. While there MAY not be any in the US, that says more about our particular culture and regulations than anything else. What you're proposing would dramatically change that culture and regulations, and would make FGM in the US far more likely.

The other benefits from such a policy may/probably would outweigh the other costs though. I'm not confident in that though, and I would like to see the tradeoffs in such a policy thoroughly analyzed. But just saying "I don't think any doctors perform FGM" as a retort to a policy change that would allow it is pretty disingenuous and lazy. If you want radical change, I think you should own the up to the consequences, both good and bad.

Expand full comment

Isn't the latter only true because it's extremely illegal in the US?

Expand full comment

Amendment to your law:

And if the patient later decides that oops, having their legs chopped off was a mistake, no take-backs. Can't sue the doctor, the hospital, the state or anyone. Have to take responsibility for being a feckin' eejit themselves. You want full autonomy? You'll get it, pressed down and running over. "But I was a mere infant of twenty-six, how was I to know?" Yeah, well, you broke it, you bought it.

Expand full comment

i think it's patently obvious that a large part of the reason healthcare in the US is so "bad" is that medical litigation has so much potential money in it, encouraging both ambulance-chasing and terrible policies that limit providers' liability. I think almost anything that can make it less litigious is a win, so I'd strongly endorse this amendment.

Expand full comment

I find this kind of description comes across as unbelievably obnoxious and I never know if I'm imagining that intent or not. If you think killing a human fetus for any reason or none at all is perfectly fine then *say that*. Say it and explain why you think that and I might even be sympathetic to your argument. But to just say "medical treatment" and nothing more, *daring* someone to point out that there's someone else arguably being horrifically harmed by this "treatment", which is the *sole reason* it's a matter of controversy in the first place...is like saying, I don't know, that the North should have let the South secede because their own laws are their business and no one else's, just daring someone to point out that there may have been another group of people involved in the whole thing.

What possible reason is there for such a manner of discourse, other than to deliberately bait people?

Expand full comment
Jun 12·edited Jun 12

I did not want to start a culture war abortion discussion.

But I'll bite that bullet - killing a human fetus for a valid reason is perfectly fine with me. Your values may differ.

Expand full comment

I didn't read "abortion" automatically into the comment, so perhaps some of it IS in your imagination. It can be a fuzzy line when abortion is merely a medical decision. If the life/physical health (not mental) of the mother and/or child is at risk, it seems like a medical decision.

But I don't think the comment was meant to be derailed into an abortion debate.

Expand full comment

Maybe you're right. It's hard not to see "medical bodily autonomy" and then the word "reproductive" as the first example, and not immediately interpret the whole comment as being really about abortion, with everyone else as window dressing.

I find it hard to imagine someone phrasing it that way without that intention (or at the very least the recognition that it will be taken as having that intention), but I could be entirely wrong.

Expand full comment
Jun 11·edited Jun 11

I agree that the original poster must have been thinking about abortion as part of it.

But it's possible that they were attempting to elucidate a general principle which would automatically resolve a number of conflicts and simplify our legal process. And to be charitable, I take it in that spirit. The proposed rule could include experimental treatment, transhumanism, and all sorts of other stuff.

Expand full comment

This was my reading of the comment as well.

Expand full comment

Its all well and fine until you ask someone else to pay for it. Then it becomes my problem.

Expand full comment

All modern healthcare systems, private or public, are transfers from the well to the sick.

Expand full comment

And someone needs to regulate that.

Expand full comment

That's... the whole point of a healthcare system. Of course.

The fear is that the system will be captured by people that want to finance vanity treatments, or even treatments that are probably harmful on a societal basis (e.g. alternative treatments that don't work). That would lead to unnecessary costs that harm the whole system.

Which treatments fall into that category is, of course, controversial.

Expand full comment

Yeah, that's basically how insurance works.

Expand full comment

Laws I'd like the US Congress to enact #1 (not gonna happen).

Corporations are not people.

In all court cases, corporations have lesser standing. In criminal cases against a corporation, a simple majority vote of the jury is sufficient for conviction. In civil cases of a corporation vs a person, a unanimous vote of the jury is needed to find for the plaintiff.

Expand full comment

Corporations are fictional persons. Abolishing corporate personhood is equivalent to abolishing corporations.

The rest of your proposals effectively serve as a variation of saying that corporations have a preponderance of the evidence standard where real people have beyond a reasonable doubt. The legal system already has a series of mild biases against corporations (as in, legally written into the law). I'm not sure what this one specifically would be meant to accomplish but it's possible it could have some good effects. Any specific idea of what you're trying to accomplish?

I suspect you would want to phase it in based on size. Bob's Local Mechanic probably isn't going around abusing the legal system.

Expand full comment

>Corporations are not people.

I don't understand what this is meant to mean. Is this an argument to end corporate personhood? Because, if so, that means ending corporations, because having a legal identity separate from its owners is the primary reason that an owner incorporates their business. Doing so limits the personal liability of the owners. Is that what you intend?

Or, do you simply mean that artificial persons, such as corporations, are not to be recognized as among the "persons" protected by the Fourteenth Amendment? Because that would mean that government could seize a corporation's assets or do anything else it wants without a trial. Is that what you intend?

Expand full comment

Corporations are merely a group of people, an official taxable structure or business structure. Mostly they're used as a business structure for family farms where there are several generations involved. Maybe not everyone is involved in the day to day farming, but they own a share of the land, and receive a share of the profits.

It is common for unscrupulous people, especially unscrupulous politicians to attack corporations. Without their standing as a person, the corporation has reduced ability to respond to attacks.

Expand full comment

I actually have relatives who have family farms. I quibble with "Mostly they're used...".

Expand full comment

If they were simply a "group of people" then, for example, if the corporation did great harm to me then I could go after the personal assets of the shareholders to make it right instead of them being able to fold up the company and laugh all the way to the bank.

Instead they're a separate special legal entity.

The responsibilities and rights should flow together. If the owners don't get all the responsibilities then the separate legal entity shouldn't get all the owners rights.

Which rights those non-human entities should get is a practical problem of encouraging investment rather than a moral problem of their owners status.

Expand full comment

I guess I should have titled it "Corporations do not have the same standing as people." Corporations have good and bad effects, at the present time I think the bad effects (energy companies causing global warming, tech companies causing the destruction of our democracy and civic society) are getting to crisis level. I think the scales of justice are currently tilted toward corporations and would like to see those scales tilted more to people.

Expand full comment
founding

"If they were simply a "group of people" then, for example, if the corporation did great harm to me then I could go after the personal assets of the shareholders to make it right..."

You say that like it's a good thing. If this were the case, it would be intolerably dangerous for anyone to hold stock in a large publicly-traded corporation. If Yoyodyne causes someone great harm, a bunch of little old ladies whose retirement funds include Yoyodyne stock all get sued into oblivion - whoever is easiest to find, can't afford good lawyers, and still has enough money that suing a hundred of them is an easier way to get some megabucks than suing a single lawyered-up billionaire.

But the billionaires would get sued as well, so they're probably going to reinvest offshore where this sort of thing doesn't happen. So will the pensioners and their retirement funds. Massive devaluation of the stock market, terabucks of capital fleeing offshore, companies hamstrung by the inability to raise capital by selling stock, and lots of people de facto losing most of their life savings.

This would be Very Very Bad. Worse than Great Depression Bad, probably close to Venezuela Bad. And by the time you've implemented the patches you are no doubt thinking up for the few vulnerabilities I've just described, and then a bunch more that I haven't, your patches will add up to a patchwork version of corporate personhood. An untested kludge imitation of corporate personhood, that won't work as well as the real thing.

The Limited Liability Corporation is, more even than the steam engine, the driver of the world's massive growth in prosperity the last couple of centuries. It enables people to safely and profitably collaborate on much greater enterprises than ever before possible. And it isn't broke, so please don't try to fix it.

Expand full comment

Indeed. It's a race to the bottom because other countries would still offer LLC's.

LLC's are a separate legal entity from their owners.

For a very practical reason.

That legal firewall should not be free when we disconnect responsibilities from profits.

As such, "Which rights those non-human entities should get is a practical problem of encouraging investment rather than a moral problem of their owners status."

We don't have to go "well there's good reason to make them a separate legal entity when it comes to responsibilities but now we simply must pretend they're the same legal entity whenever it comes to rights"

Expand full comment
Jun 11·edited Jun 11

This would be an enormous, world-shaking change to the law, but I'll leave it to someone with greater knowledge to go into the details of why. But "piercing the corporate veil" may be the term you're looking for.

> Which rights those non-human entities should get is a practical problem of encouraging investment rather than a moral problem of their owners status.

It is my impression that this is already the case. And that anyone who claims or implies otherwise is either ignorant and misled, or is actively deceptive.

Expand full comment

My post is in response to the " merely a group of people" claim. Not to advocate for the removal of the concept of an LLC.

Since they're not just a group of people. They're a separate legal entity firewalled from those people when it comes to responsibilities.

Expand full comment

Scott, briefly flagging that your Lorien Psych page on Adderall needs a (small) update. It talks about Vyvanse still being under patent, but the patent expired last year. Hopefully there have been new generics come up since then?

https://lorienpsych.com/2020/10/30/adderall/

Expand full comment

News on the Fermi paradox, AKA "Where's all the aliens?" An article in Nature

https://www.nature.com/articles/s41598-024-54700-x

argues for adding "two additional terms to the Drake Equation: foc (the fraction of habitable exoplanets with significant continents and oceans) and fpt (the fraction of habitable exoplanets with significant continents and oceans that have had plate tectonics operating". With very small values, these terms bring the resultant value of the Drake Equation way down, thus resolving the paradox. With my so-so knowledge of evolution and geology I find their arguments plausible, but wonder if anyone here who is more knowledgable can comment.

My own opinion - the Drake Equation is irrelevant, the 2 reasons why we don't see the aliens:

1 Space is really big, like astronomically big, and it's too far

2 Space is really inhospitable to life

These 2 reasons will also (IMO) put the kibosh on plans for space colonies, even just on Mars.

Expand full comment

I think about this sort of thing whenever someone asks why God created a world with natural evil, like earthquakes and volcanos. There's a lot of possible answers, but it should be acknowledged that a world without plate tectonics would be much more inimical to life than one with it! And you can't have plate tectonics without earthquakes, volcanos, and tsunamis.

Expand full comment

Once I described my position on anthropics and the problem of evil as this:

"A universe that is absolutely optimal for life would presumably have extremely specific parameters. Universes that are almost optimal for life would have more room for variation, and therefore they would exist (potentially) in greater number. Universes that are reasonably good for life would exist in greater number still, and so on. Very flawed universes could be very flawed in countless different ways on countless parameters, and would therefore make up the vast majorityof all universes that could potentially exist.

Now, the number of universes that are utterly uninhabitable could be even greater than that; but of course we must exist in a universe that allows some sort of life in the first place. Therefore, contra Leibniz, we almost certainly live in the worst possible universe (or rather one of equally terrible universes). This is borne out by the fact that we only observe life in a fraction of about 10^-61 of the observable universe (for a rather generous definition of a biosphere 80 km thick).

(One counterargument is that, anthropically, we would expect to be born in universes that contain the most abundant life than in those that contain scarce life; but it’s to be seen whether that is enough to cancel out the difference in universe number.)

I don’t see that as a particularly pessimistic or unpleasant conclusion -- if anything, it suggests that there must be plenty of avenues to improve our condition. Given our starting point, it would have been hard to make it worse!"

(https://o-craven-canto.tumblr.com/post/721591431480868864/)

Expand full comment

That doesn't address the problem of evil because an omnipotent God can design plate tectonics without people needing to die to make it work.

Expand full comment

Those two terms are already contained in f_i, the fraction of planets with life in which advanced intelligence develops. The real issue with the Drake Equation is that every single term except star formation rate is completely unknown, with uncertainty margins that span multiple orders of magnitude, so that it is trivial to "prove" that there is only 10^-20% probability of intelligence appearing even once in the universe, or that there should be three inhabited planets per solar system. Adding more terms does nothing to solve this issue.

Expand full comment

Fermi’s paradox isn’t any kind of paradox. It’s a refutation of the principle of mediocrity applied to the terms of the Drake equation. No real paradox.

Expand full comment

Agree, thanks for this.

Expand full comment

I worry about adding factors to the equation when we have so few data points to base our guesses on.

Expand full comment

The book Rare Earth by Peter Ward brought up plate tectonics as a term and was published in 2000. And he probably didn't even come up with the idea, so this doesn't seem like news, necessarily.

Expand full comment

The big part doesn’t matter. If the terms of the Drake equation are not that limiting and there’s millions of intelligent lifeforms out there they had plenty of time to colonise the galaxy, or indicate their presence.

Expand full comment

Your first reason can easily be overcome with enough time. Space is big, but it is also old. Given a few million years an intelligent civilization can easily spread across a galaxy, and so they should be here by now.

I think the real problem with the Drake equation is that the variables aren't constant in time. Rates of star formation and planet formation are much different now than they were in the past. You could make everything a function of time, but then it gets too complicated to be really useful.

Personally I think plausibly habitable planets and moons are fairly common, but that in most cases life never gets beyond simple microorganisms. And then if it does, there's no guarantee that intelligent civilizations arise. Earth, if it hadn't been for a meteor strike, would probably still be dominated by unintelligent dinosaurs.

Expand full comment

The question is whether the big num of time can cancel out the big num of distance. My skepticism is based on trying to imagine a bunch of human would-be space colonists. They would need to carry with them everything needed to survive N years traveling, maintain a functioning mini society for however many generations required, and have nothing go wrong. If N=100 (there's actually a maybe suitable planet only 40 light years away) then it's maybe plausible. If N=10,000 (we are about 27,000 light-years from the Galactic Center) then forget it.

Expand full comment

My own opinion on this ... space is really big, and our system is really far from the super dangerous galactic center, 26,000 light years far. Fortunately for us. I imagine the amount of junk being flung about is horrendous, that extinction level events are increasingly common the closer you get to the Galactic Center. I have a model for this. It may be wrong, all models are wrong, but some are useful. Here it is:

Mike Kelly's reasoning for lack of Milky Way life outside Earth

Across the past 500 million years, Earth has experienced 5 major extinction events. On average about every 100 million years apart. Shelly life evolved about 541 million years ago. These extinction events are presumed to be caused by extra terrestrial impacts, which—long story short—wipe out most life forms. Presumably these asteroids originate in local asteroid belts, and are somehow disturbed and sent into Earth orbit. Perhaps there are random asteroid belt events, or perhaps extra-solar system asteroids are the cause of the impacts, or even the asteroid belt disturbances. Let's follow up on this hypothesis. Earth is 26,000 light years from the center of the Milky Way. If we look at the drawn image depecting the Milky Way, we see our solar system, i.e. Earth out in the quiet unfashionable western arm, out where we don't have very many neighbors. If we look towards the interior, the systems are packed in tight, it's probably a pretty wild place, asteroids crossing different systems all the time. So if we perhaps see extinction level events every 100 million years, how often do those interior systems see extinction level events? Let's set a model of how often asteroid extinction events occur. My model, is 1/(prime)_n. So we're 26 * 1,000 light years from the center of the galaxy. So the rate of extinction level events is 100 million times over the 26th prime number [ which is 101] thus 100,000,000 over 101, which according to this model, a planet a thousand light years from the center of the Milky Way, sees an extinction level event about every 99 million years. My hypothesis is that planets closer to the center of the Milky Way have extinction level events far too frequently for complex life to develop.

Expand full comment

That's a plausible model. It also implies that because life forms that survive extinction level events are located in the far out quiet unfashionable arms, they are farther apart from each other.

Expand full comment

Don't forget Jupiter, who protects us somewhat from asteroid impacts.

Expand full comment

I like your pronoun choice. :-)

Expand full comment

Reminds me - I need to go prepare some burnt offerings...

Expand full comment

Looking through the list of epithets, this might be Jupiter Depulsor Optimus Maximus?

https://en.wikipedia.org/wiki/Epithets_of_Jupiter

Expand full comment

Oops, that final value is 990 thousand years, not 99 million.

Expand full comment
Jun 11·edited Jun 11

Anyone know why LSA was passed over by drug companies as a potential psychedelic assisted therapy agent, in favor of LSD and psilocybin? Being Schedule 3, it has less stigma and wouldn't require rescheduling as part of approval.

Expand full comment

Isn't LSA super duper unpleasant? Or at the very least not as dependably enjoyable as LSD.

Expand full comment

How does one become, to quote the 1daysooner comment, "an informed and engaged voter"?

I recently became an American citizen and this year is the first I'm eligible to vote. Washington State (or the county I'm in, at least, but I think it's the whole state) sends out pamphlets with about a page of information on everything that's on the ballot; I've read a few of them and they can be helpful in eliminating some obvious choices (e.g., someone was in the US Senate race a few years ago who had no elected experience, or the candidate whose page just has the same all-caps sentence repeated over and over), but what do people tend to do with the rest? To the extent that part of the problem is that I don't take in the news (in between cycles), what are good sources of local news?

Expand full comment
Jun 13·edited Jun 13

You must distinguish between political and administrative appointments on your ballot.

In local elections, you will be voting to fill up some positions where the candidate's politics is completely (or almost completely) irrelevant, such as treasurer or registrar of deeds, and you should be looking to pick the person who's most capable and responsible, regardless of their politics. (In particular, someone who used to run a local business is typically a good bet.)

To give you an example, in 2008, students at Dartmouth College voted for a student with a D next to her name to fill up the county treasurer position over the incumbent who had an R next to her name. The student was too busy with her classwork to do the job, and 2 years later everyone was calling for her resignation:

https://www.laconiadailysun.com/news/local/dartmouth-student-grafton-treasurer-tries-to-get-on-same-page-with-lawmakers/article_48c0b08a-0807-53da-bfa7-73d4bb5c2e6d.html

It's sometimes tempting to vote the same party all the way down the ticket, but local administrative positions are where you absolutely should not do it. It's especially hard to do research about candidates in these races, since they often have no web presence, but if you want to do the right thing, you should at least try.

Expand full comment

If you’re in a blue or red state, always vote for the losing side. Single party control destroys good governance. Since you’re in WA this means always voting Republican for every position in the general election.

Expand full comment

Mindless voting by following an algorithm is utterly useless. What's the difference between that and voting for one party because your father did?

Of course, that assumes voting is actually useful, in that it is possible to select the best candidate, whereas it is more probably simply a shell game with no ball.

Expand full comment
founding

I think myst_05's proposal is overly broad, though it's a two-line internet post so we shouldn't really expect nuance.

But since you ask, the difference is: nothing will ever change the way your father used to vote, so that algorithm is absolutely inflexible. The myst_05 algorithm automatically disengages and reverts to "now actually look at the parties and candidates and decide" as soon as the state turns purple. Which is the only time your vote ever matters for the purposes of determining which party will hold power.

Expand full comment

Thank you, effectively I WAS asking. So this means voting blue in red states, and red in blue states. But it also still seems to mean voting for the less popular candidate in purple states, whoever that may be. If, instead, it means vote for the best candidate in purple states and the losing candidate only in blue or red states, then I can now see the logic.

Expand full comment

Yes, once there's no longer a Dem/Rep trifecta in the State you start looking at candidates based on their merit. But until then... 100% Dem or 100% Rep, no matter what.

Expand full comment

I'd filter this on the minority party fielding candidates who are at least vaguely fit to hold office.

Expand full comment

Oh, and a few tips for the US's "first past the post" system, and WA state in particular, because I don't know where you're from. And some of these may be controversial:

If there's only one candidate running, always turn in your ballot, but leave it blank or write in something silly, like "Mickey Mouse" or "Pogo the Possum" (both are cartoon characters). This demonstrates that you would have voted, but that you're open to opposition. Voting for the only candidate can provide the impression that the candidate is more popular than they actually are. We want to encourage competition.

WA state is unusual in having a "jungle primary" (except for Presidential elections), where all the candidates compete together in the first (primary) election, and then the top two advance to the second (general) election (unless one got a majority in the primary). This means that we can always vote for our preferred option in the second, but "strategic voting" is possible in the first. Don't get carried away: it's usually best to vote for your favorite, in part because it encourages similar candidates in the future. But it's a good idea to check the polls before filling out your ballot, just in case.

Don't worry about "wasting your vote"; that's a stupid idea based on incorrect models of the system. We're playing a long-term iterated game.

If you like WA state's mail-in ballot system, thank Sam Reed, the conservative secretary of state who carefully and thoughtfully implemented it almost 20 years ago. It's not a left-wing plot. But it's also not something that could be rolled out overnight, and places that tried a rush job while under partisan pressure, ran into problems that WA never did.

Expand full comment

Local political groups (often chapters of national organizations like DSA, NRA, Indivisible, etc) often put out voter guides containing endorsements, obscure races included. Find the one most aligned with your views and see what it says. You don’t have to blindly follow the endorsements, there’s usually some reasoning given and you can decide whether or not you agree.

Expand full comment

Get your news from all sides.

Former California governor Gerry Brown famously said: if you paddle the canoe a little bit on the left and a little bit on the right, it will go somewhere. If you paddle on just one side, it goes in circles.

Expand full comment
founding

From which we know that Jerry Brown kind of sucked at canoeing. I'm not sure what political lesson we should learn from this.

https://paddling.com/learn/how-to-do-the-j-stroke

(Yes, I am an Eagle Scout and did earn the Canoeing merit badge).

Expand full comment
Jun 11·edited Jun 11

You can check local news sources, like newspapers and weeklies and TV stations. They usually have coverage of each election, and recommendations, and you can form your own opinions about reliability and bias.

(Depending on where you live in WA state, "The Stranger" has a rundown for each election, and even if you intend to vote exactly the opposite of how they recommend, it's worth reading because they're good about highlighting issues, and the comment section usually has people who fill in gaps in coverage (as with occasional moderate-left candidates who somehow never get mentioned).)

Voter pamphlets should include party affiliation and endorsements. In WA, people can self-identify as whatever party they choose, so you can see things like "GOP" vs "Republican". But endorsements are an even better place to look, because they can tell you who's plugged into which parts of society, and who they care about wooing.

And once you have the names, try doing some web searches, and see what people online have to say. Sometimes you'll find useful sites that you never would have found otherwise: forums, local pundits, regional blogs, etc., which have analyses of your local elections.

And after you've been active a while, you'll know the histories of various candidates, and what the local issues are, and maybe even know a few people involved personally.

Edit: and congratulations!

Expand full comment

HOLY SHIT. I, TOO, CAME HERE TO TALK ABOUT SEATTLE'S THE STRANGER AND THAT I USE IT TO VOTE MOSTLY THE OPPOSITE OF WHAT THEY RECOMMEND!

!!!

I mean, I literally wrote a version of a smug "I voted" Facebook post bragging about this wonderful hack to civic duty!

Wow!

Expand full comment
Jun 11·edited Jun 11

I know, right? And they're clear about their point of view, cover every contested race, include reasons that aren't mealy-mouthed politician-speak, provide hyperlinks to relevant information, and leave the comment section open to people who disagree. Aside from their viewpoint, they're as good as I've seen anywhere.

(For people not from the area, "reflexively choosing the leftmost candidate" produces very different results in a city like Seattle than it does in, say, rural Georgia.)

Expand full comment

Yeah, I likewise hugely respect The Stranger's journalistic ethic and due diligence, even if I disagree with their goals / position.

Not everyone is going to have this particular hack, but I find it very useful for reducing my voting research from "hours / days" to "minutes."

Expand full comment

These are all great comments, thanks. I have much to look up and learn here!

Expand full comment
Jun 10·edited Jun 10

Found via ACOUP, "The Moral Economy of the Shire", a look at what Tolkien tells us about the social structure of the primary hobbit community, before, during, and after the war. Shockingly for this sort of analysis, the conclusion sounds fairly pleasant!

https://nathangoldwag.wordpress.com/2024/05/31/the-moral-economy-of-the-shire/

Expand full comment

That was a great read. Thanks for the link! I never really quite understood where the Baggins got their money from (with the exception of the loot Bilbo brought home from his adventure). Or who actually ran the Shire, since the King was long gone. The whole thing makes a lot more sense now.

Expand full comment
Jun 10·edited Jun 10

How would you explain professionalism to had no conception of it?

There's definitely a laundry list of expected behaviors ("call when you are going to be late," "don't fall asleep during the zoom meeting," "if you say you'll do the thing, do the thing, etc.") but as in any social setting the do's and don'ts, if you really think about it, is functionally infinite, so if I have a co-worker who needs them spelled out one by one, it's actually kind of impossible to do. You could fight the battle and finally get them to stop sleeping on zoom calls, for example, only to find that now they're streaming netflix on their second monitor during the same calls.

Assuming good faith and that this person is genuinely upset at how they "keep getting it wrong" when each behavior is pointed out to them as a problem, and then take steps to address them (after all, if it's a bad faith thing, there's no fix for it and the answer is to just fire them), how would you approach it?

Expand full comment

There are a lot of complex answers being given. I don't know if this works perfectly, but how about a very simple rule: while on the job, you don't advance your own desires. There are some exceptions to this, but maybe they can be listed much more easily than listing all the things that aren't allowed.

Getting angry is unprofessional, because you're advancing your own feelings instead of the goals of the organisation. Promoting your political views on the job is unprofessional, for the same reason. Sleeping or watching netflix during a meeting for the same reason. Flirting on the job for the same reason. Exceptions may depend on the place, like taking a personal call during work hours. These can be explicitly listed.

Is there anything this account doesn't cover?

Expand full comment

I'm not sure to what extent this is so obvious that it can just be taken as implied, but stuff you need to do to cover your own basic needs during work hours (e.g. taking breaks (in moderation), eating) is an exception to that description. Plus there are cases where you explicitly are there to advocate for yourself (say you're negotiating your salary), and cases where there just isn't anything you could be doing to usefully advance the company's goals (e.g. you've run out of work at the moment but need to wait around in case more comes in, and get a book out while you're waiting).

Your description does seem like a good first approximation, but I think maybe the phrasing should just be weakened in general, like "Generally prioritise advancing the organisation's goals over your own during work hours.".

Expand full comment

That person needs to read several hundred Ask a Manager blog posts (https://www.askamanager.org/) and perhaps skim the comments to get a sense of the consensus of what's "acceptable" workplace behavior.

I don't think one coworker telling them, "hey, don't sleep on a Zoom call," is going to do it. They need the collective wisdom of hundreds-to-thousands of individual experiences to understand the "culture."

The blog is free and fun, but if not that, then there are hundreds of workplace etiquette books out there. And maybe watch The Office? I dunno.

But in any case that person needs to spend more of their free time consciously learning social skills and way less of their free time playing video games (how do I know they play a lot of video games? Of *course* they play a lot of video games!).

Expand full comment

Professionalism is, essentially, a suite of signals that you give a shit about work.

It's also not up to the individual to define this behaviour. Much like "respect", a lot of it is about how the behaviour is interpreted by other people. Like respect, different people and different workplaces interpret it differently - some people think eye contact is a sign of respect, some people think it's a sign of disrespect. Some workplaces think taking a lunch break is unprofessional, but others consider it a professional duty to not just take a lunch break, but join the team on a run to bond and improve personal productivity. Same with the dress code (some workplaces will consider suit and tie unprofessional because you're spending too much time on your appearance).

But like respect, there's generally commonalities - don't keep people waiting, don't let personal feelings cause you to behave in ways that make people uncomfortable (because workplaces are made of people).

With respect, you want to send a credible signal that you care about the other person - their feelings, their wellbeing, etc. With professionalism, you want to send a credible signal that you care about work. It doesn't matter what you think - you may think very highly on someone, but they might offense to something you say. Maybe watching Netflix in the background has no bearing on the quality of your work - but other people may not see it that way, even if proven wrong.

Expand full comment
founding

Professionalism is a set of norms that leads to predictable behaviors that enable people to invest their time and money in collaborative ventures with reasonable confidence that it won't all go to waste because one's collaborators don't hold up their end of the bargain or can't get along with one another. It is related to the concept of "profession" in the sense of a paying job or career, because in our society most of the collaborations that need this involve paying people to do what they otherwise wouldn't, but if e.g. you're trying to run an amateur theater group you'll really want everyone involved to engage in the sort of predictable behaviors that would in other contexts be called "professionalism".

Working from that, if you're wondering whether any specific behavior is "professional", then a good test is to ask whether someone else might reasonably expect you to do that thing (e.g. because you told then you would) and will they find that their own participation in a collaborative enterprise less rewarding if you don't, then doing that thing is probably the appropriate professional behavior.

This won't cover everything, because some of the expectations and harmful consequences are not obvious. And also because some of it is just arbitrary signalling, e.g. a professional dress code enables others to see at a glance that you understand the rules and are willing to abide by them even when they seem silly or uncomfortable. That's often a valuable signal to send, so if in doubt ask (or at least pay close attention to what others are doing).

Expand full comment

The core of society is predictability. You need to present yourself in a way that makes people trust you will do what is expected of you, so they can plan around it getting done. Show up on time, give the impression you're paying attention to instructions, wear clothes that look like you take your work seriously, report any problems that might slow or stop the work.

Expand full comment

Treat your coworker, clients, employer, and work environment with dignity and respect.

Expand full comment

If someone has no solid conception of professionalism, you're kicking the can down the road, because they probably don't really know what dignity is either.

Expand full comment

Hmmm, that's a good point. Maybe, "as if you care about them."

Expand full comment

Professionalism is about behaving in line with the expectations of your job, rather than in line with your own preferences. Exactly what that may entail is job-dependent.

If a doctor keeps you waiting for half an hour then that's in line with professionalism, but if he's wearing shorts and chewing gum and saying "crikey, that's a real mess you've got going on up there" then that's unprofessional. For a plumber, it might be the other way around.

Each job has a certain amount of stuff you can "get away with". My generalised professionalism advice would be to do your job extremely strictly until you've figured out that something is in the list of things you can get away with. For instance, show up to work precisely on time until you figure out that other people come in half an hour late and nobody cares. Don't swear at work until you've heard a bunch of other people do it. And so forth.

Expand full comment

For contractors, "Make people want to hire you, repeatedly, and recommend you to their friends, because you actually do a good job! (No tricks!)"

It's a bit harder for full-time employees, because the hiring and firing feedback mechanism is less direct. Maybe something like, "When you're on the clock, do work, only do work, and do it well. When you're not on the clock, be polite and helpful, but know the what the expectations are for people in your role, and stick to them."

Expand full comment

The basic principle is: We do work here. Do work, enable others to do work, don't distract with non-work-related things, don't *get* distracted with non-work-related things.

This is in theory something you can approach as a matter of discipline, but it falls into place more naturally in a real work environment, with a tie and an office chair. I assume from the mention of Zoom that it's remote? A shift in environment might be productive. The nature of professionalism is keeping work and life separate, a physical separation between living space and working space can help you conceptualize it.

Expand full comment

Great question, and I dont have a good answer for you, but I think this is one of those concepts everyone discussed without it ever being detailed what exactly it means

Expand full comment

I just saw the second of a really surprisingly interesting (and long) pair of videos about the seemingly innocuous question "how many super mario games are there?" that actually ends up being in no small part about the same sort of deep category questions as are discussed in the SSC classic post https://slatestarcodex.com/2014/11/21/the-categories-were-made-for-man-not-man-for-the-categories/

For anyone else who has an interest in the subject matter and/or for whom the above post means a lot, I'd heartily recommend them:

https://www.youtube.com/watch?v=XejJ6PzPtEw

https://www.youtube.com/watch?v=-Ddmjcy3lEs

(The guy's whole ouvre is pretty interesting tbh, including stuff like conlangs, regular langs, number bases and cultural phenomena like homestuck and caramelldansen)

Expand full comment

The Aliquot numbers and the hailstone numbers have the same feel. Do all numbers settle down eventually? Are the two sorts of number related in some way?

On my first search I mistakenly put in "snowball" instead of "hailstone" and duckduckgo didn't guess at what I meant even though I included "number theory" in my search.

Expand full comment

Jan Misali is a gem

Expand full comment

What are social skills?

Expand full comment

There are plenty of good insights in the responses so far, but to me the answer is pretty simple... Social skills are behaviors that manipulate others how you'd like. To be clear, this would exclude actions intended to gain power in other ways, which are then used to manipulate people (like buying a gun so you can intimidate someone into doing what you want). I'm not sure that is a helpful answer, but it seems clearly right to me.

Expand full comment
Jun 11·edited Jun 11

Unlike professionalism, I think this one requires a very complex answer.

I don't think "social skills" really means anything, it's too broad. This phrase, along with "status", "nerds", "intelligence", and many others, is the same kind of simplistic attempt to lump a whole lot of at best vaguely related things under a single label, that I get more and more irritated at with every Open Thread. I'm going to try an answer to this question by stepping right back to the level of philosophy.

There are, as far as I can tell, four different kinds of value recognised by philosophers generally, three quite clearly distinct and one very nebulous: (1) ethical value, or what is morally right, (2) aesthetic value, or what is beautiful or impressive, (3) prudential value, or what advances self-interest, and (4) social norms. The last one is nebulous and hardly ever discussed (in my philosophy degree I only encountered it in one very specialised honours-level subject). Many would say it's just a part of morality (especially virtue ethicists). Many others would say it has no value beyond the prudential, and model it as game theory with rational self-interested agents (this is the approach overwhelmingly taken by rationalists). However some consider is as in some sense an additional form of intrinsic value: i.e. an act could be both moral and personally rational (and "cool" or "pretty" etc) while still violating social norms and thus being in some sense intrinsically bad. The other camps would obviously deny that this makes any sense.

Even if we do separate social value out as its own form of value, the term "social skills" is *still* often used to include things from the other categories, like in discussions of harassment (moral value, or at least claimed to be depending on the meaning of harassment) or charisma (obviously prudential value). This makes it a pretty unhelpful term. In one sense a beloved above-board professional like a doctor has strong social skills, in another sense Trump does (widely disliked and violating norms constantly, but reliably able to get a minority to trust and adore him). Is a con artist or pickup artist or bullshit artist someone with great social skills (able to manipulate people) or terrible social skills (widely hated and viewed as morally bankrupt)? I think most would say "great" but this doesn't jive with how the phrase is also used in other moralistic contexts.

So when ordinary people use this phrase, who knows what they really mean. I suggest people around here should either taboo the phrase or be philosophers and clearly define their terms.

As to your question: it means one or more of

(1) acting virtuously and morally and honourably in social contexts

(2) acting attractively or impressively or charismatically, in the same way an actor gives a good performance or an author writes a good novel

(3) rationally and effectively manipulating or handling people to promote your personal goals and interests

(4) acting in accordance with the norms and conventions and unwritten expectations of your culture or subculture, insofar as you are convinced that this is a valuable thing that isn't part of one of the other categories.

Expand full comment

Thanks for a complete answer. I like that it also answers the meta-question: why is this seemingly simple and innocent question so tricky when posed in this corner of the Internet? If you ask ordinary people what are social skills, they will probably answer that it's either charm or the ability to make friends easily. But our minds can quickly find counterexamples or metrics that would defeat both definitions.

The question is not well-defined, probably because of the fourth component you mentioned, which knocks a lot of topics out of analytical conversation. Asking what social skills are is as broad as asking what it means to be good.

Expand full comment

Do philosophers generally leave out what I'd call ordinary pleasures? If so, is this a problem?

Expand full comment

Sorry, I probably defined prudence badly. That should cover all personal pleasures, personal desires and so on (and the process of seeking them). I wasn't meaning to imply that self-interest should be understood objectively, as distinct from simply whatever one wants for him- or herself. But these terms are often used differently by different people so it's confusing.

There's so much philosophy and I'm only familiar with a fraction of it, but I *think* that the notion of a person having objective interests that are separate from desires or pleasures is not very common in modern western philosophy, except for some forms of virtue ethics.

Expand full comment

having a personality thats inside the overton window, its called skills to pretend that its objectively good for society and that its learnable

Expand full comment

Nah. I'm autistic and I definitely learned skills that improved my social relationships with other people.

Expand full comment

That strikes me as destructive nonsense.

If an individual has the intellectual capacity to understand that certain behaviors are required (whether or not they are logical) and that other behaviors will have negative social consequences (whether or not that is "fair"), then they have a capacity to learn "social skills."

Temple Grandin is one of the most influential and famous living autistic people today and she's a vocal advocate of kindly but firmly *imposing* social skills on autistic children. She has publicly stated in both books and articles that her mother's unfailing determination to make Grandin conform to basic manners is what set her up to be socially successful when she had to go out into the world professionally.

https://www.templegrandin.com/temple_articles/teaching_methods.html , but there is a lot more in her books.

I recommend Thinking in Pictures and The Way I See It, and while I haven't personally read Unwritten Rules of Social Relationships, and The Loving Push, I'm assuming they likewise encourage autistic people to consciously adapt to the civilization they find themselves in rather than defy it.

While I'm not autistic, I have always been unusually egotistical, a trait which can be enormously self-destructive if not managed. I had to *consciously* learn to what a "normal" person's reaction to certain behaviors would be and then *consciously* stifle those behaviors to avoid upsetting people enough to damage my relationship with them. I wrestle to conceal my contempt from people every. single. day., and I'm still learning and occasionally failing.

Social skills can be learned, but only by people who are willing to surrender to their power.

Expand full comment

I gave a keynote speech at a company where Temple Grandin had given one the year before, and the emcee told me that after hosting her, Temple had her assistant send them a questionnaire asking how she could improve her social skills. When the host told me this, she didn't add any follow-up details but instead just gave me a nonverbal expression of exasperation, as if some infraction or error had been made.

Contrast that with my reaction, which was, wow, that sounds really practical and useful. Upon reflection, my host was telegraphing the irony of sending an awkward questionnaire about how to eliminate social awkwardness.

It's been a while since I did a literature review on the subject, but in the late 2000s, I read Grandin's The Unwritten Rules of Social Relationships and another book that attempted to break down social skills. At the time, I was baffled by how few books there were that just spelled out the basics of socializing. But now, upon further reflection, there were books about social skills. They just didn't call themselves that.

For example, Dale Carnegie's How to Win Friends and Influence People is technically a book that teaches social skills. The 48 Laws of Power also has quite a few lessons about socializing with people in power.

I just did a quick search on Amazon, though, and I see many new books in the last ten years covering the subject, including an updated version of Grandin's. So, the demand is rising.

Expand full comment

Huh, I wonder if Grandin's questionnaire was exceedingly or detailed or in some other way burdensome?

Because baring the questionnaire itself being a giant time-suck, I can't imagine a good person being so uncharitable as to roll their eyes over it (or whatever) when volunteering the anecdote to a stranger a year later.

Expand full comment

The Dale Carnegie book has some useful, practical advice. It’s not as Machiavellian as the title suggests.

Expand full comment

> If an individual has the intellectual capacity to understand that certain behaviors are required (whether or not they are logical) and that other behaviors will have negative social consequences (whether or not that is "fair"), then they have a capacity to learn "social skills."

it feels required **and** very socially negative to lie on resumes, gossip, disown canceled people at the drop of a hat, and then be happy in an interview that all this is interconnected as someone directly stares at you to determine if your happy enough with the status quo.

So long as there are face to face situations, I don't believe you can expect everyone to learn "social skills" perfectly.

Job searching is part of "socail skills", includes a poorly designed zerosum games, and if you get behind and have a job history gap you get very behind.

>autistic

>autistic

>autistic

Given the existence and political power of silicone valley, autism is in the overton window

The job social services once pointed me to apply for a tech recruiting firm for people with mental illnesses, I got an automated turn down saying they specifically only help autistic people

> but only by people who are willing to surrender to their power.

that would be a personality trait, conformity

Expand full comment

My second paragraph originally said something like, "as long as someone doesn't have an intellectual and/or emotional disorder, they are capable (etc)."

Look, one either has enough intellect plus humility / openness to studying social skills and implementing them (even and perhaps especially if they feel unnatural), or one doesn't. We can get into semantics about what "personality" is and whether things like oppositional disorders are part of "personality," but what purpose does that argument really serve?

Civilization demand social skills. If one is articulate enough to observe and complain about them, one is almost certainly has the capacity to eventually learn them, again, even if that feels unnatural or unfair.

Expand full comment

Personality's are more fundamental then society's mental illness definitions

The level of violence required by ancient rome fits some personality better then others but we would not call people who fail those social norms mental ill today

Is everyone on earth capable of butchering an animal while in a culture that promotes and requires the "social skill" of animal sacrifice? Can you expect everyone to fake it, and have the flare to do this thing they may hate and cheer and dance while drinking animal blood?

Today you expected to "have a positive attitude" while diving head first into meeting someone the first and probably only time, while promising your the best person for the job, while stress of money could easily be on your mind, and all sort of other bullshit; I dont feel this is "mentally ill" to fail here, it society failing

"Social skills", require specif emotions to be on your face for specif arbitrary actions; so it is not so simple as rote skill building.

Expand full comment

Maybe take some acting classes, then? Or, like, practice in a mirror and literally learn to put the correct expressions on your face? Write some short stories about job interviews to make them feel more normal and controllable?

There has never been a civilization where social skills specific to that civilization weren't required to live in it. Hell, I'm guessing every past civilization without mass media was quite a bit *more* strict about requiring performative social skills, particularly between different classes. At least this is an era where you are unlikely to literally starve should you offend the people who might provide you a livelihood.

But even today, you can resent the need for social skills, and you can resent how difficult they are to acquire, but as you've pointed out several times, social skills are critically necessary for gaining and keeping a modern livelihood.

So train on them the way you presumably trained for your livelihood.

Hell, you can even train on them enough to know how to be acceptably unconventional. Can't blend in? Then be disarmingly blunt about it. *Name* your social difficulty explicitly, to reassure the interviewer that at least *you* know hat basic social skills are,, even if you can't perfectly ape them. Have a self-deprecating joke or two loaded and paired with a humble-brag about your professional strengths.

I'll acknowledge that conventionally good-looking people and/or people with naturally high charisma are going to have an advantage in most settings, there's no getting past that.

But you really can build a skillset to manipulate the people you meet into doing things like giving you jobs.

You're clearly literate and if you're interested in Scott and are commenting here, then you're almost certainly pretty smart in several areas.

I doubt you're going to argue that you aren't smart enough to acquire social skills? That you aren't smart enough to read the famous best-selling books on the topic and/or watch the biggest YouTube channels and/or take an acting class and/or do anything other than unproductively complain that all of civilization should, like, forget about the "arbitrary" social skills that are generally useful for assessing strangers and building trust because they're just too hard for *you?*

Bro, I've been close to where you are, and I'm telling you, it's not going to go well if you don't consciously decide to act against your instincts and impulses. It's clear you can read, you can observe, you can think, and you can self-reflect, so already have everything you need to eventuality brute-force professional social skills. There's no need for this kind of despair!

Expand full comment

that sounds about right

Expand full comment

Here's a selection from "Issola" by Steven Brust, which I think is as good a description as any that I've run across:

----

Lady Teldra was standing across the room, as calm and patient as an issola, as if waiting for some call that hadn't come. She had taken herself away from the conversation while no one was watching. I reflected on what a fine skill it would be to know when you weren't wanted at a place you didn't want to be, so you could make everyone happy by going away. I walked over to her. She looked up at me, a slightly quizzical expression on her face. I said, "How do you do that, Teldra?"

She smiled and raised her eyebrows, and came as close to looking smug as I'd ever seen her. I said, "So, all right, how do the laws of courtesy tell us we should handle this mess?"

"The laws of courtesy," she said, still smiling, "are strangely silent on the subject."

"I'm not surprised."

"In any case," she added, "I think you know them as well as I do."

"Oh, yeah," I said. "If there's anything I know, it's courtliness and good manners. I'm even better at politesse than I am at refining petroleum."

"I know little of petroleum, Vlad, but I do know that you are actually quite skilled in the arts of courtesy."

"Right."

Behind me, Aliera and Morrolan were continuing to speak to the Goddess, but I couldn't make out what they were saying. In the event, this did not displease me.

"It is the simple truth, Lord Taltos. It is how you survived for so long in the world you used to inhabit - or, more precisely, the worlds."

I bit back a smart reply and just waited. After a moment, she said, "The Jhereg has its own rules and customs, you know - codes of appropriate behavior. You couldn't have survived among them without knowing what all of their signals mean. And I've seen you with my Lord Morrolan. That is another different set of codes." I snorted. "I've almost pushed him far enough to kill me. More than once."

"I know that, too," she said.

"Well then?"

"What stopped him from killing you?"

"His strong sense of self-interest combined with iron self-control."

"I don't believe that is entirely correct, Lord Taltos. I know him rather well, I think, and there are severe limits to his self-control, whereas there are no limits to his pride. Had you pushed far enough, you would have faced a mortal contest."

Morrolan, Aliera, and the Goddess all turned and walked out the door. I guess if you put a pretty little stream outside your door, people will want to look at it. I hoped the Jenoine would feel gratified.

"Okay," I said to Teldra. "Look. I'll concede that, over the years, I've learned that there's no point in making a bad situation worse, and that it's less work to talk yourself out of a tough spot than to slice your way out, and that words, while potentially deadly, are less deadly than Morganti daggers. But I don't think that is quite the same thing as being courteous."

"I believe, Lord Taltos, that it is very much the same thing. And you know more than those things, if I may say so. You know when a casual insult is, in fact, courteous under the circumstances - and when it is not. You know when to make a friendly gibe, and when the gibe is not quite so friendly, but still called for. You know how to negotiate from a position of weakness but make it appear to be a position of strength. These are the sorts of things I'm talking about. And do you know how many of our folk - and yours - never learn these lessons that appear so simple to you?"

"Maybe, being an Easterner, I have a natural talent."

"You forget how many Easterners I have known, Vlad. Your people have no such natural talent. In fact, the conditions under which your people live tend to promote the opposite: an irritating obsequiousness, or an aggravating combativeness."

After a moment's thought, I said, "That's true."

She nodded. "It is really all a question of taking appropriate action for the circumstances. I'm sure you realize that I could have this conversation with few others - human or Eastern - that I know. Some it would embarrass, others it would merely confuse."

"Yes, I understand."

"You have learned, faster than some of my own House, what actions - and words are only a special case of actions - are appropriate to the moment."

"A survival skill, Teldra."

"Yes, it is."

"Ah. That's your point, isn't it?"

She smiled, making me feel like my grandfather had made me feel when I had managed the correct riposte after parrying a lowline cut.

Morrolan, Aliera, and Verra returned at this point, speaking in low tones. I gestured toward them and said, "And the Goddess?"

"What about her?"

"What need has she of courtesy?"

"Toward her peers, the same as you or I. Toward us? None. Many of the gods, I believe most of them, display a certain degree of courtesy even though none is needed. Those who don’t acquire a reputation."

"For being, say, chaotic?"

"Yes."

"So it is all a question of courtesy?"

"It is all a question of doing the appropriate thing. Of acting as the situation calls for."

"Appropriate thing. You keep saying that, Teldra. When someone walks up to me and says, 'Out of the way, whiskers, you're blocking the road,' is it appropriate to bow and say, 'Yes, my lord?' Is it appropriate to suggest his mother was a toothless norska? Or to quietly step out of his way? Or to urinate on his boot? Or to pretend to ignore him? Or to put a knife into his left eye? Just what does appropriate mean, anyway?"

"Any of those things might be appropriate, Vlad, and I daresay there are circumstances where you might do any of them. But you are always, or nearly always, correct in which you choose. And this is not a matter of instinct, but of observation, attention to detail, and experience. Appropriate action means to advance your own goals, without unintentional harm to anyone else."

"Unintentional harm."

"Yes."

"By Verra's tits," I said, forgetting then remembering that the pair of them weren't all that far away, "you're as cold as Morrolan, aren't you?"

"Yes," said Teldra, "I suppose so. Or as cold as you."

"Me? I'm not cold. I'm the soul of compassion, understanding, and courtesy."

"Yes," said Teldra, dimpling. "You are indeed. But only when it is appropriate." I chuckled. And, "Okay. I'm convinced. All problems are matters of courtesy, and I am the personification of tact. So, to return to the question, what is the appropriate thing for us to do now?"

"I have no idea," said Teldra, still smiling. "I imagine that is what our friends are discussing right now."

Expand full comment

"Appropriate action means to advance your own goals, without unintentional harm to anyone else." Sounds like a decent set of components: context reading, successful self-interest seeking, prediction of consequences of such seeking

Expand full comment

Social intelligence.

That sounds like a shut-up stupid answer, but its not. Gardner has a theory there are 9 types of intelligence. Many discount his theories, but on the surface I concur with them.

To me, intelligence is: proper intake, processing, and response to acceptable environmental patterns. Such as interpersonal interaction (one of the 9). In the environment of interacting with another person, proper listening, eye contact, receiving language, understanding the spoken and unspoken, proper communication back to the other.

To me, intelligence is the proper reading, processing, response to acceptable patterns in per domains.

We all have various ability levels in each domain.

Expand full comment

I'm a little familiar with Gardner's intelligences. However, that specific breakdown sounds like communication fidelity or a basic technical competencies. If I nailed all those things (eye contact, etc.), how much would my social skills improve?

Expand full comment

Ballpark? The skills that place you comfortably somewhere between a jerk and a sap. Not meant as a serious answer of course. It’s hard to tell how serious your question is, it being so brief.

Expand full comment

Everything that makes up the Negotiation deck.

https://forums.kleientertainment.com/forums/topic/110028-thoughts-on-negotiation-cards/

Expand full comment

Huh, this looks like an interesting game, thanks (I'm a Slay the Spire nerd)

Expand full comment

I enjoyed it. The main problem is it's two and a half hours for a run; too long for a roguelike, too short for an RPG.

Expand full comment

I assume you don't want a dictionary definition. Can you elaborate a little on what you're really asking?

Expand full comment

Social skills seem unlike what we normally think of as skills, like playing the piano or even being good at business. It has some weird, asymmetric physics, whereby having bad social skills can be turned into good social skills in the right situation. (fine line between bravery and foolishness).

But in some ways, it's like walking, i.e., the penalty is very high for negative performance (disability). As they say, "It takes a lifetime to build a good reputation, but you can lose it in a minute."

The weird example that comes to mind is this episode of The Crown, where an unemployed and socially awkward man breaks into Buckingham Palace to chat with the Queen. In that moment, she engages with him and even seems to care about his opinion of her, despite the power disparity (and not because she's afraid of her physical safety; she's the Queen, and she can off with his head at any moment).

https://www.youtube.com/watch?v=PWPmSYpfKRA

Another example is politicians, who we think of as people with some of the best social skills, who seem to repeatedly get flustered in interviews, almost like they presumed they were going to stomp on the interviewer but then were surprised. Compare this with master pianists whose recitals are always on point, where any parts of self-criticism are internal and on minor details we don't even notice.

Social mavens often appear to be idiots.

Expand full comment

I'm willing to give politicians a pass, in terms of getting flustered during interviews. Piano is arguably harder to master, in a sense. But interviewers are often adversarial, and thus try to throw you curveballs. Whereas pianos are inanimate objects and are not out to get you. Also, the pianist probably isn't going to be sight-reading Rush E at a public performance, so they're not gonna see anything surprising.

Have you ever tried public speaking? It does not come naturally to me.

Expand full comment

Is there such a thing as adversarial music? Dueling banjos? Or the Portsmouth Sinfonia?

Expand full comment
founding

Adversarial music, African edition: https://youtu.be/1csr0dxalpI

And since everything is better with Nazis: https://youtu.be/cOeFhSzoTuc

Expand full comment

The Lord: "love thy neighbor".

His subjects: (https://youtu.be/TUOPvtVZwo8?si=UukWngY88OrSL6fi&t=21)

Expand full comment

BBL Drizzy (https://www.youtube.com/watch?v=AF2MqFnPotc) (https://knowyourmeme.com/memes/bbl-drizzy)

Gandhi vs Martin Luther King Jr. Epic Rap Battles of History (https://www.youtube.com/watch?v=-6G6CZT7h4k)

Metalocalypse: The Doomstar Requiem Toki/Skwisgaar Guitar Duel (https://www.youtube.com/watch?v=gBYQAR8fG1w)

:^)

Also, citing "Rush E" was somewhat facetious. It's designed to be unplayable with human hands.

RUSH E (https://youtu.be/Qskm9MTz2V4?si=zJCBcj8yMTIYjctl&t=16)

The Osu beatmap for "Centipede" is similarly pathological.

Knife Party - Centipede (osu! - auto) (https://www.youtube.com/watch?v=0av5hVm-YOI)

So sometimes, playing solo can be adversarial after all.

Expand full comment

Yeeess, rap battles, of course! Or the mexican grandfather of rap battles: https://www.youtube.com/watch?v=kHEhw0DTwnw

Of course, they take turns rapping against each other and don't do it at the same time. Would it be possible that two musicians battle against each other simultaneously?

What comes to mind is that part of "Happiness is a warm gun" where everybody except Ringo switches from 4/4 to 3/4 ("when I hold you in my arms"). But I guess normal musicians would turn it into a cacophony.

Expand full comment

This gets a little closer to what I think I'm circling around, which is that social interactions are duels in a way, where the objective is some synthesis of three objectives: your objectives, theirs, and then some vague middle, outside observer.

Yes, I've done quite a bit of public speaking.

Expand full comment

And here I figured my comment was gonna be incidental. Well, if that's what you're really asking about, I certainly have a working model. Although it's not entirely fleshed out.

A) Social interactions are a game.

B) Games are training-simulations.

C) The purpose is to: build trust; practice sparring.

Part of the purpose of small talk is to build rapport. it's a handshaking exercise where you probe mannerisms. This is important so that lines of communication are open when you need to actually negotiate a mutual decision. E.g. I vaguely recall hearing a story once where, during some war (can't remember which), British military officers told their American allies that "we're in a bit of a pickle". The Americans assumed the Brits were merely inconvenienced. So they did not send help. The British however, have a habit of understatement. "We're in a bit of a pickle" was a euphemism for "we are completely and utterly fucked".

The other purpose, is that people just naturally find small talk fun. Fun is nature's way of nudging you to improve at certain skills. Such as negotiation. So yes, this involves verbal sparring. AKA rhetoric.

----

Also, I follow some redpill/blackpill stuff. Not to actually use the dark arts, mind you. But moreso as like, an anthropological interest. And it turns out that social interactions are dimorphic as well.

My understanding thus far, is that men get ribbed on, by both men and women. Stuff like greeting each other with "how ya doin, dumbass?" This is a stress-test. Men need to be dependable and emotionally-stable, not whiny babies when shit hits the fan. E.g. when you and your phalanx buddies march on Persia, God forbid the dude next to you has a mental breakdown. Better to weed them out sooner than later.

Women's social-circles are paranoid about loyalty. Thus, they will constantly loyalty-test each other. They're also very egalitarian. No single female is allowed to be smarter, kinder prettier, etc than the others. The result is a lot of policing. As well as guile and cunning and subterfuge. If you fail a test, you're excommunicated. I think there's other things going on too, but i haven't wrapped my head around all of it yet, well enough to explain coherently.

> three objectives: your objectives, theirs, and then some vague middle, outside observer.

This specifically reminds me of Chris Voss, the hostage negotiator. Supposedly his strategy consists of: A) find their religion; B) make *your* goal instrumental to *their* goal.

Expand full comment

I agree with your take on men's social circles. I spent my formative years as mostly the only female in smart young men's social circles, and while casual insults are indeed about stress-testing, I've often described the constant ribbing (or "shit-talking") as a "verbal trust-fall" exercise. Performative comedic insults are only ever aimed at the people one trusts to not take offense at the insult, both because they're emotionally robust *and* because they understand there's no intention to injure their feelings. Every insult lobbed at me about my sexuality, obesity, appearance, and so on was a verbal hug.

I can code-switch when I'm in social circles which discourage shit-talking, but when those boundaries are asserted, the little goblin-bro deep in my heart quietly sneers "pussy" and notes that is a person who can not be trusted to manage their own feelings (and thus feels entitled to impose self-censorship on me). I'll never really trust them.

Your description of women's social circles doesn't match my own experience of friendships with other women at all, but it's possibly a Different Worlds thing (https://slatestarcodex.com/2017/10/02/different-worlds/).

Expand full comment

I think that's a good model. It has "continuous improvement" vibes too, since social is the one part of the brain that seems to keep getting grey area density after age 25, so clearly it's something our biology values and wants us to keep training on.

Expand full comment

>like playing the piano

I think this is actually pretty close to it. Social skills are the ability to harmonize with the people around you, like playing a chord consists of playing multiple notes that complement each other. Which note you want to play depends on what notes everyone else is playing. And then you get really good at it, and you start being able to harmonize with a person enough to pull them toward harmonizing with other people.

Politicians don't really have good social skills; their primary goal is to tap into an existing feeling long enough to convince people to vote for them, and give the impression of confidence. They're trained to dodge questions and not say anything that would break the illusion, but they don't harmonize so much as pretend to.

Expand full comment

Interesting. However, choir skills seem very close to singing skills. The team component is maybe 10%? (I've never been in choir). I can't think of a skill that's mostly team work, though.

Good point about politicians. Who has good social skills, though?

Expand full comment

Think "barbershop quartet": four people singing the same song, but deliberately hitting different notes that complement the other people's notes.

I'd say therapists are going to be the most "good social skills" group, considering they're basically professional friends. Actors might be in there too, or any job that's paid primarily to make people feel certain things. (Although "theater kids" is its own subculture, so maybe not actors so much.)

Expand full comment

Come to think of it, there are a handful of professions that are seen as mostly social. Counseling is a similar example to therapy. Being a barista or bartender is mostly about maintaining engaging banter while being on your feet and multitasking for hours.

Expand full comment

The most recent Construction Physics substack post about data centers had a section on heat management, which led me to wondering if non-heat computing is possible.

In my Newtonian brain I have thought of heat as being the byproduct of inefficient systems, the portion of the applied energy that wasn't used by work.

In my mind, this holds for simple electric circuits, where the used energy is transferred to some sort of kinetic process, and the heat is inefficient waste.

In biology my understanding gets fuzzier. Energy is primarily used to power electrolyte transfer against gradients, and I would guess heat is generated by uncaptured energy released by the mitochondria during ATP formation. Again, inefficient waste.

But in processors, the energy is used to perform math operations. Are heat-roducing resistors a fundamental aspect of computing? Are there models for performing math without resistors? And if so, is there any energy pathway in computing that doesn't just end in heat?

Expand full comment

Most of the heat these days comes from leakage. Transistors are krazy small, even traces are just a few nanometers wide. Consider silicon atoms are 250 picometers wide. I don't remember the stacking, but a 2nm line is basically 8 atoms wide. The leakage is due to not having enough insulator between circuts and power/ground.

Expand full comment
Jun 10·edited Jun 11

One of the big promises of neuromorphic chips is to reduce heat production and energy consumption by 3 order of magnitude or more. The brain is even more energy efficient than that.

Neuromorphic chips are not based on transistors, but are instead based on neuron-like architecture. Two main differences are:

- Classical chips work with bits, 0 or 1, and with high precision. That is pretty wasteful because the states 0 and 1 are designed so far from each other that a 0 is always clearly distinguishable from a 1. This makes the transition pretty energy consuming.

- Classical chips are clocked, i.e., there is synchronization between them. Whereas (some) neuromorphic chips are asynchronous, and there can be information carried by the exact time that a spike takes place.

NB: all this has nothing to do with Neural Networks, LLM and AI, except for some overlapping names.

Expand full comment

Look up Landauer's principle. In a nutshell, *setting* a bit requires spending a minimum amount of energy, or more precisely, increasing entropy elsewhere. Otherwise you could create order out of chaos, reducing the total entropy of the system. Since computation requires a "clean" array of bits in a known state to be used as memory, and the final state of that memory is unknown to you (without performing the same computation in a different system), it follows that this energy cost cannot be circumvented, regardless of your method of computation.

I think this bound has been called into question in various ways. Something something reversible computation. If you want to do a real deep dive, "Information is Physical" by Rolf Landauer is a good place to start reading.

Expand full comment

> Something something reversible computation

Reversible computing in principle has no lower limit on the energy required to compute. It is a fundamental tool for quantum computing, as well has having relevance for classical computers if/when we start to approach the Landauer limit.

Expand full comment

You might get more insights if you focus on the transistor instead of the resistor for computing.

In very basic terms the transistor is a switch. And the heat generated depends on the power consumption of this switch. Power is voltage times current. If the switch is off there is full voltage (1) but no current (0) so the power (and the heat) is 1*0=0. If the switch is on there is no voltage (0) but full current (1) so the power is 0*1=0. So we see the power consumption of a switch is zero in both states.

This changes in the moment the switch is actually turned from on state to the other. In the short time it takes the transistor to switch from off to on (or the other way around) the current and the voltage are both greater then zero. So in this moment the power becomes positive and we generate heat.

You can reduce this power consumption and heat generation if you make your transistor switch faster. Or technically if the slope of the voltage and current is steeper.

You might also look into power transistors instead of computing. Reducing the power consumption of power transistors in inverters is a big topic for electric cars. Since it will directly increase the mileage you can drive with one battery charge.

Expand full comment

The amount of heat produced can be reduced greatly using a technology called “reversible computing”. This reorganizes the circuits and computation so that the output contains enough information to reproduce the input. This can be used with ordinary silicon, in which case resistance losses are still an issue. Or it can be used with superconductors, which eliminate resistance losses. In the superconducting case, as we perform the computation slower and slower, it uses less and less energy; in the limit it uses no energy at all. It’s never been worthwhile to actually use reversible computing, because it’s several-fold bigger and slower than a normal irreversible circuit. It’s better to just accept the losses and go fast. But people have built experimental reversible microprocessors. It’s one of the great also-rans of digital logic.

Expand full comment

Is the restriction to only reversible computations actually useful when you aren't anywhere near the thermodynamic limit to start with?

Expand full comment

Yes, it’s still useful. To make a transistor switch from one state to the other, you have to charge or discharge the gate capacitance. This takes some amount of energy. In an ordinary circuit, this energy is thrown away next time you switch the transistor. In a reversible circuit, this energy is carefully reclaimed and stored in an inductance in the power supply. There’s also an economy in not switching a transistor while voltage is applied between the source and the drain, which I don’t want to go into right now.

But it’s not useful enough to make reversible computing actually useful in practice.

Epistemic status: I worked on this for a while in grad school.

Expand full comment

Heat is the result of ignorance about the system being embedded in the universe you are trying to perform computations in.

As an extremely toy example, let's pretend that electrons flow through a 3d grid of metal atoms. When we run some computation in a computer, what we are doing is making the electrons move from one location to the other. We are only concerned about the density of electrons at one point or another, to determine whether it's a 1 or a 0 and so we force the electrons to move towards areas we want to be 1 and away from areas we want to be 0.

Heat happens because we don't know the exact starting positions and velocities of the electrons or of the atomic lattice they are traveling in [0], so if an electron bumps into the nucleus of an atom, on top of the existing known velocity of the electron moving at some speed, we now also have an unknown angle and unknown location at which the bump happened. Repeated over many collisions, this means that 1. Eventually whatever you did to make the electrons move in that direction in the first place becomes much less relevant, which practically means that the energy you put in has become heat. 2. There is no way of bypassing this without specifying the system down to a much more precise state. Except that if you *did* want to do so, your brain itself has to run a bunch of additional computations to figure this out, which also generates heat and so on and so forth.

This is why "computing that doesn't end in heat" is not a thing.

[0] Yes yes, I know about the Heisenberg uncertainty principle, what I am saying here would apply even in a perfectly classical universe, and can obviously be generalized to quantum systems.

Expand full comment

Yes you always have waste heat. All useful energy use produces waste heat.

There is no computing without heat and in some conceptions that will be the primary concern eventually, such that you might have big computers on the surface of black dwarves and such.

Expand full comment

UK voters: what is the ongoing appeal of the Liberal Democrats? In the 1980s the typical SDP/Liberal voter was soft left but more pro-European, pro-American and anti-trade union than the average Labour MP. Along comes Tony Blair and basically the SDP has conquered the Labour Party. Charles Kennedy successfully positions himself as an anti-war alternative to Blair (the high point for the party in my opinion), but that can't last forever and the right of the party insists on having a turn in Nick Clegg - cue coalition with the tories and a collapse to less than 10 seats in 2015. Corbyn & Brexit give them an opportunity to get back to their pro-european and anti-left roots, but they make little progress, at which point Keir Starmer becomes Labour leader and again, we have a guy who is pretty much a Lib Dem in charge of the Labour Party. And yet polls are predicting a Lib Dem resurgence, with some speculation they might overtake the tories as the opposition. I don't get it!

Expand full comment

Personally, when looking at their manifesto I quite liked that they seemed to have more of an emphasis on civil liberties than the other parties, but I don't remember the details.

Expand full comment

For one thing, there are a lot of disaffected tories who would never vote labour, even if they had identical policies to the LDs. For another, the LD's often come second in right leaning constituencies, so it makes tactical sense.

Expand full comment

Great question, I'm Irish but have followed British politics since I was a teenager, ever since John Major won a pyhric victory in his vote of confidence! The UK should really have a lot more political parties, the Tories largest problem has been that they have two many factions that are ideologically too far apart. The Lib Dems have managed to survive, despite unfairness of the first last the post system, because there is always a place on the political spectrum the two larger parties are currently ignoring. The Tories have so embarrassed themselves they will pick up a lot of left wing Tory votes.

Expand full comment
Jun 11·edited Jun 11

I suppose I'm the opposite - I keep expecting a consolidation of broad left parties a la the Democrats. I did think of Ireland actually because it seems like the parties there are more grounded in history or community than ideology - and there is this core of historic Liberal support in the south west of England. Which means they always have a base from which to mount appeals to let down voters.

Expand full comment

Keir Starmer can do the "hates the left" part, but is, relative to the LibDems, an authoritarian.

So, "we're less authoritarian than Starmer" is a plausible pitch for the LibDems ... assuming voters trust them about anything, which is a big if.

I think the overtaking the Tories prediction is not about any positive qualities of the LibDems, but a theory that the Tories are (a) massively unpopular; (b) getting their voter base split by Reform, so there;s a change the LibDems might end up the Opposition party by default.

Expand full comment

So the authoritarian thing is interesting because I remember preferring the Lib Dems during the Blair years partly because they were anti-ID cards and so on. But they seem pretty 'woke' now, for want of better word, and that's bound to clash with supporting free speech and so on.

Expand full comment

Personally, I think the usual LibDem strategy of trying to peel off both Labour and Conservative voters doesn't work, because when you eventually get into a coalition about 50% of your supporters will be very, very upset with you. Picking who you're hoping to get into a coalition with has less potential for disappointed voters.

Expand full comment

Permanently siding with one of the main parties doesn't work either..people just wonder why you bother being a separate party. Also, never going in to coalition doesn't work because people hater hung parliaments and re-elections.

Expand full comment

Right. And the UK electorate, not to mention the markets, seem to hate the idea of a hung parliament and rarely vote for one. So it doesn't seem like a good bet to me. Voting with your heart is valid of course but the kinds of things the LDs are interested in seem like matters of degree.

Expand full comment

Or it could be even worse. When Corbyn was labour leader, me talking to a LibDem supporter....

"So, Ok, you hate Corbyn and are opposed to Brexit. What kind of deal are you expecting the LibDems to cut with the other parties? Make a deal with the Tories where they enthusiastically support Brexit in exchange for keeping Corbyn out? Oh, you'ld hate that. Make a deal with Corbyn to implement some some of leftist policy program in exchange for stopping Brexit? Oh, you'ld hate that too. you are setting yourself up for disappointment here,."

Expand full comment
Jun 10·edited Jun 10

At the national level, we have a first-past-the-post election rule that encourages a two-party system; if you're centre-left, think the conservatives have messed up quite enough but you're at best lukewarm on Labour, they're the "independents".

At the local level, precisely because they don't have to think about how this would affect their chances of getting power nationally, Lib Dem councilllors are often really effective and feel like "normal people" on top of that.

I doubt they will overtake the Tories, even if the right-wing vote splinters, but I'll celebrate if they do.

Expand full comment

Incidental stuff comes and goes but the deep appeal of the Lib Dems is centrism. There are boatloads of people in the UK who don't really like/trust Labour or the Conservatives because they're too left/right respectively - while both (labour rn) go through peroids of flirting with centrism they have centres of ideological gravity further out. They do badly in elections because the First Past The Post system strongly encourages people to vote tactically in favour of the Top 2 party they think is less bad.

Expand full comment

The driving factor in this election is more anger at the Tories, rather than a real love for Labour or the Lib Dems. Lib Dems have always had a strong local game, and historically did well in south eastern areas around London and in the southwest. Those areas are not natural supporters of Labour (which was historically focused more on the northern working classes). Wealthy southerners who want an alternative to the Conservatives will likely look to the Lib Dems as more acceptable or moderate than Labour, regardless of what they say they stand for.

Expand full comment

That all makes sense, and yet Labour have been making progress in wealthier areas for some time, in places like Canterbury or Kensington, and this is part of a global trend where left-wing ideas are becoming seen paradoxically as high-status. In which case I would have thought the old Liberal identity as being a higher class of radical would be gradually becoming less important.

Expand full comment

Paradoxically?

Expand full comment
Jun 10·edited Jun 10

You may think it's obvious that left wing = high status, for a 1950s or 1980s voter the reality was very different. In which case voting Liberal or SDP would have made sense as a high-status alternative. My point is: does that still hold in 2024 when both Labour and the Lib Dems are pretty middle class?

Expand full comment

I’m reminded of a New Yorker cartoon circa 1970. Two Manhattan women in furs in conversation.

“After The Revolution even *we* will be better off.”

Expand full comment

I have a half-remembered Moldbug article in my head in which he argues that the natural alliance in politics is between the rich and the poor against the middle class. And the rich-poor alliance always wins.

Expand full comment

Athabasca University is a Canadian online university. Among the degrees it offers is a four-year 120-credit BSc in Applied Mathematics. I would be interested in hearing from someone with a serious math education about how solid the curriculum looks. Here are the required MATH courses:

MATH 215 – Introduction to Statistics

MATH 265 – Introduction to Calculus I

MATH 270 – Linear Algebra I

MATH 266 – Introduction to Calculus II

MATH 271 – Linear Algebra II

MATH 309 – Discrete Mathematics

MATH 315 – Methods in Applied Statistics

MATH 365 – Multivariable Calculus

MATH 366 – Complex Variables I

MATH 370 – Applied Real Analysis

MATH 376 – Ordinary Differential Equations

MATH 476 – Partial Differential Equations

MATH 480 – Mathematic Modeling I

MATH 495 – Mathematics Projects I

(And three more.)

Is this a _very_ applied program? I don't see anything about abstract algebra, for instance.

More info here: https://www.athabascau.ca/calendar/undergraduate/program-regulations/degrees/bachelor-of-science-applied-mathematics-major.html

Expand full comment
Jun 12·edited Jun 12

It has the core applied math curriculum, and probably OK at that. The problem is what to do beyond the core. Jacob Manaker is on right track that it seems be missing courses that makes applied math major most useful for the math-y applied fields. For a less-mathy, very applied program, it is lacking the application domain course(s).

Two introductory stats courses they offer -- Introduction and Computer-oriented -- will make for an introduction but still an uphill battle to get into applied statistics; similarly, Modelling I (and their Modelling II, too) look like broad overviews, probably not specific enough for getting into any serious modelling.

Expand full comment

Pure math professor here, who works in a (sadly) mostly applied math department. To be honest, this looks far more rigorous than you'd see at my institution. Our applied majors never see abstract algebra (sadly), nor are they required to have any contact with number theory, combinatorics, or topology. They see lots of numerical methods, math modeling, etc. We require our applied majors to take 15 credits in "upper level" courses outside of math that use higher level math - these are typically engineering or stats courses since a lot of our majors are double majors.

I would love to say what you listed above is "typical" of a four year applied math degree in the US, but I think this is only typical of the top schools, certainly not the mid-major public that I teach at. Of course, a lot depends on the actual quality of the courses.

I don't *like* that any of what I said is true (again, pure math guy here who thinks everyone should see abstract algebra and some form of analysis and topology) but I think it is the sad reality. Or maybe my school is exceptionally sad, but I don't think it is. (Well, my school is sad, but not because it lacks a higher level applied degree, but rather because functionally it has no pure degree at all.)

Expand full comment

<i> (again, pure math guy here who thinks everyone should see abstract algebra and some form of analysis and topology)</i>

I understand some of the theoretical sentiments behind this, but is there anything to it beyond sentimentality/normative conviction, namely any study that shows that introducing pure math into the curriculum actually helps applied mathematicians do their job better?

Expand full comment

I'll cop to having a bias towards pure math. I could list topics in algebra/analysis/topology that have applications, but that wouldn't be the real reason. I think pure math develops a kind of "from the ground up" rigor that seems to be missing in applications. I liken it to the difference between knowing how to drive a car (applied math) versus knowing how a car works (pure math). You can probably be a pretty good driver without knowing how a car works, but having some experience under the hood would definitely help. (Ironically, I have exactly zero idea how cars work. I know which pedal does which, unless I've somehow stumbled into a manual.)

I don't claim to have any study to support this (and knowing that it would likely be mathematicians doing said study, I would honestly have little faith in its conclusion either way). I haven't looked for any sort of study like this, I suspect the relatively small sample size of the population along with the numerous confounding factors would be pretty constraining.

Anecdotally, though, I think pure math and applied math (at least these days) is one way - you hear about a number of people trained in pure who go into applied, but not much vice versa. Feel free to make of that anecdotal evidence what you will.

Expand full comment

My anecdotal observations agree with yours, but I interpret that as making a case against pure math: basically, a pure math degree qualifies you to ... teach pure math to others. Applied math gives students numerous industry options and thus justify the fees and possibly also tax payer expense.

Have you seen pure math PhDs who write very remarkable but not groundbreaking 100+ page papers and then cannot get them accepted anywhere, because the top journals can anyway fill their pages with papers that are either groundbreaking or are coauthored by people they know, while non-top journals have ridiculous page limits? You tell this to a pure mathematician and the response is "Well, don't write long papers unless they are groundbreaking" [even if they are much better than shorter papers that get in].

But after it all, do they get the satisfaction they aspired for in the first place? Partly yes, in that pure math is probably much deeper than what they initially expected it to be. But from another perspective, no, because their work is not nearly as impactful as they were given to dream of when encouraged to do a PhD. All academia destroys lives of numerous failed PhDs, but pure math does it to a lot of really bright people.

Some version of this Sabine Hossenfelder video applies to pure math as well: https://www.youtube.com/watch?v=LKiBlGDfRU8

Expand full comment

Do we know that typically happens to math PhDs who fail to find faculty jobs? I would expect some of them to end up as software developers, which is a pretty soft landing, really. But what about the rest?

Expand full comment

I can only talk anecdotally. If you are a PhD from Harvard/Princeton/Stanford etc. you have a good chance of landing a faculty job or as a quant etc.: e.g., Renaissance Technologies loves hiring good PhDs from such places. So doing a pure math PhD at these places is mostly okay. But if your PhD is from say Ohio State/U Oklahoma/Nebraska etc. things become quite uncertain (U Michigan, U Maryland etc. would be in-between). One might land a quant/actuarial/coding job after varying, often much, uncertainty and struggle (I know a friend who took an extra year for his PhD at a similar level university just to prepare for finance jobs, got none, and returned to India; another, a reasonably smart and super charismatic US citizen, who had to painfully give up drinking and a relatively loose lifestyle to take up a post doc in the Saudi, but later somehow, after much struggle, managed to get a software job in California).

Otherwise, one might get a teaching position at a non-research, say liberal-arts-centric, place. Again I think this is deeply unpleasant and possibly even traumatic for those who aren't gifted with a very specific kind of "teaching" skill, but to explain why I suspect so I can only offer you my experience as a TA. For those courses targeting non-STEMmers, I was explicitly asked to avoid any conceptual explanation and merely teach them algorithms, because otherwise I wouldn't be able to finish the syllabus (apart from the fact that most students would hate the conceptual explanation). This would be okay, but I still found the material too tight to cover, and counterproductively fact-burdened. For instance, a "Business Calculus II" course taught integration to students who had forgotten most of the differentiation they learnt in "Business Calculus I". I initially disliked the students, but later realized that they were victims: they forgot stuff because the courses packed more info than they could digest, often facts which were not pointless but not crucial either, and hence could be dropped with better breathing space and hence better learning outcomes: after all, what is the point of teaching so many facts if they were pretty much guaranteed to forget? And I saw students who took this course twice or thrice, sometimes with a personal tutor, and yet did not get the "B" that they were required to continue with the undergrad options they were hoping to make. But then I heard two explanations for this: one, that the syllabus needed lots of material listed for the university to maintain their ratings, and the other that they had to take in many students for reasons related to the funding, but then throw out many (apparently, this was probably one of the courses the university used to this effect). The image of a student crying as she got me to sign a "W" still haunts me; she was hardworking, and might have survived if I were more experienced or more gifted; she lost a lot of money because I couldn't measure up to the kind of teaching the TAs were expected to, especially as a first-timer.

Those who grow up in the American educational system would certainly not struggle as much as I did with such a job, but remember these are people who could get much better jobs and were scammed into doing a PhD, to keep pure math programs at mediocre departments running.

Expand full comment
Jun 10·edited Jun 10

To provide a bit more information, here are the learning outcomes for the Applied Real Analysis course:

- demonstrate a foundational understanding of real analysis through the statement, proof, and application of key theorems.

- define the completeness property of the real numbers in terms of limits of sequences, subsequences, and the Cauchy sequence.

- define the concepts of convergence, completeness, and compactness in the context of ℝn.

- differentiate between continuous and discontinuous real-valued functions, as well as between uniform continuity and continuity.

- distinguish between compactness, the existence of extreme values, and the intermediate value theorem, and list their implications.

- demonstrate a foundational understanding of differentiation and integration, culminating with the fundamental theorem of calculus.

- define and identify normed vector spaces and demonstrate a basic understanding of the topology of normed vector spaces, paying particular attention to inner product spaces.

- apply the principles of real analysis in the field of dynamical systems.

- communicate mathematical ideas and analyses in a clear and organized manner.

And the Complex Variables course:

- demonstrate understanding of the concept of complex numbers and the complex or Argand plane, and carry out basic mathematical operations with complex numbers expressed in either plane notation or polar notation.

- determine if a complex function is continuous, differentiable, analytic and harmonic, and find its derivative.

- work with basic complex transcendental functions such as the exponential, logarithmic, trigonometric and hyperbolic function.

- evaluate contour integrals of complex functions to solve practical problems by applying the fundamental theorem of calculus for analytic functions, Cauchy’s integral formula, and the theory of complex integration.

- determine the power series of elementary analytic functions and their convergence.

- demonstrate understanding of the concept of residue and apply this concept to evaluate certain real integrals.

And here's their (optional) Number Theory class:

- demonstrate a foundational understanding of number theory, including the definitions, conjectures, and theorems that permit exploration of topics in the field.

- define and determine whether a number is a prime.

- state and apply the fundamental theorem of arithmetic.

- work with numbers and polynomials modulo a prime, linear congruences, and systems of linear congruences, including their solution via the Chinese remainder theorem.

- define the order of a number relative a prime and be able to restate and apply Fermat’s little theorem, Euler’s theorem and Wilson’s theorem.

- state and apply Lagrange’s theorem, define a primitive root of a prime p, and derive properties of the Euler φ-function.

- define quadratic residues and non-residues for primes p, determine whether a number is a quadratic residue for a prime, and state and prove the law of quadratic reciprocity.

- define Pythagorean triples and primitive Pythagorean triples and derive their properties.

- use number theory to design an effective encryption system through an exploration of the RSA public key cryptosystem.

Expand full comment

That lines up with the level of detail I saw when I did my maths undergrad at a top-end UK university. I don't think you're getting short changed in terms of detail within each module. As other commentators have said, I think it's a shame you're not getting any abstract algebra, topology, or graph theory - but given it's an applied course perhaps that's to be expected.

I was surprised not to see any modules on physics-adjacent topics such as statics/dynamics, though perhaps those will be covered as part of multivariate calculus and differential equations.

Expand full comment

I've never heard of "applied real analysis," and as someone who's taught it I'm not sure how such a thing would even make sense. The lack of abstract algebra is indeed weird. The two courses in stats is certainly noteworthy. I'm very curious what Linear Algebra II covers.

Overall, I'm not filled with confidence that this curriculum has enough proof-based content. Usually you start going full proof-based after Real Analysis, but the word "applied" kind of implies you don't maybe? And then after that you have ODE and PDE which might be proof based (PDE basically always is, ODE often isn't), modeling probably isn't, idk what "projects" is but I'd assume it's applications based. Complex Variables I may well be proof-based but then why isn't is called Complex Analysis? Linear Algebra II may well be too, but that title could mean lots of different things. Same with "discrete," could be proof based but could be lots of things and the low number makes me cautious.

There's only so much one can say from the required courses. Depending on the optional courses available, this could well be an excellent curriculum with just more applied classes than normal. It's certainly a lot of mandatory classes, actually. Are all of these really required?

For reference, I'm comparing this to my pure math degree from Berkeley, and my experience teaching at R1 schools.

Expand full comment
Jun 10·edited Jun 10

I question whether this is a math program at all. (OK, it's clearly not any other subject. But it's not enough to count as a "math major" IMHO.)

As you correctly identify, there is no abstract algebra. Consequently they cannot cover algebraic geometry and number theory (screwing over anyone interested in cryptography or teaching mathematics at the K-12 level); combinatorics (except at the facile "Discrete Mathematics" level, screwing over anyone interested in programming); and representations of finite groups (which weirdly underlies the previous two).

They don't teach functional analysis (which screws over anyone in a quantum-mechanics related field). They only teach "Applied" real analysis, which likely screws over anyone who wants to do rigorous work in probability/statistics, mathematical physics, or mathematical economics. I assume that Fourier analysis is squeezed into bon mots across half-a-dozen courses, but that assumption might be wrong, in which case they're missing that too (screwing over anyone doing signal processing and electrical or audiovisual engineering).

They also don't teach mathematical logic, which is niche but very useful for future computer scientists.

They don't cover any differential geometry and any topology, which are somewhat pure subjects but keep on appearing in applied problems in strange ways. In many cases those are key to the rigorous differential equations work that many companies hire applied mathematicians for.

IMHO, "capstone"/"keystone" project is a good idea for a major, and I'm glad that their MATH 495 exists, but MATH 480 alarms me. It's better to see modeling in action by taking courses from other allied disciplines (physics, computer science, chemistry, theoretical biology, geophysics, economics, mathematical sociology) than to try to learn it from a mathematician; you already get the mathematician's perspective in all your other courses. At best, the course is likely to improve your ability to self-employ, by duplicating the workplace training you would otherwise receive during your first month at a corporation.

Expand full comment

This looks pretty similar to the math part of my undergrad curriculum as a CS major, which was ~80% identical between all the majors. Very solid theory foundation for using math as a tool later on. Of course I had that foundation and then a bunch of courses specific to my major on top of it.

I guess an applied mathematician often ends up doing the exact same work as a computer scientist, e.g. writing fluid sim algorithms, or something equivalent in another major such as physics, electrical engineering, etc. So I would expect the remainder of the courses to be a smattering of domain knowledge in adjacent fields?

When I think of an actually "applied" university, it would typically not have a math major, and other majors just having 1-2 courses titled something like "foundations of mathematics I, II" which will be the only courses teaching any sort of theorem - proof - corollary based math.

Of course, the mere course titles are very little info to go by. E.g., abstract algebra as you mentioned might or might not be included in discrete math.

Expand full comment

Yes, from the titles this seems very applied indeed. Some indication is that it's called "Complex Variables" instead of "Complex Analysis", and generally "Calculus" instead of "Analysis". I have briefly looked into two course descriptions, which kind of confirmend it. My expectation would be that it mostly teaches how to compute and evaluate things (computing derivatives, integrals, etc.), more like an engineering program.

In the worst case, it only teaches these things without much understanding. Of course, there are also excellent engineering programs. But yes, as a hardcore mathematician, I would probably look down a bit on someone with such a degree, and suspect that they learned only stuff that Mathematica can easily compute.

Expand full comment

How does testosterone boost strength? I think everyone has a basic, intuitive understanding of how test improves muscle recovery from workouts. However there's a few studies indicating that subjects taking exogenous testosterone or anabolic steroids, and then doing *no working out at all*, experience significant strength increases. Is there an 'Explain Like I'm 5' explainer as to how a hormone can just increase muscular strength?

Expand full comment

The body has various feedback mechanisms that drive adaptations in response to stress. To oversimplify horribly, the big ones are catabolic and anabolic responses. Catabolic responses are directed at making more energy available, metabolizing stored energy including unused or under-used muscle tissue and keeping more glucose circulating in the bloodstream. Anabolic responses consume medium-term energy in order to build up muscle tissue to improve strength and endurance.

Anabolic steroids, as the name implies, signal the body to do more anabolism. Androgen receptors throughout the body notice that there's more testosterone circulating and take that as a signal to lower the threshold to do thing that build more muscle. Any given level of activity will result in a higher equilibrium amount of muscle if there's more testosterone in your system, assuming nothing else changes, since your muscles a bit more anabolism and a bit less catabolism in response to the same stress level from your normal daily activities.

The mechanism by which testosterone helps recovery is that it lets you train harder without your body going hard into "overtraining" mode where catabolism dominates. And that's the more relevant effect for bodybuilders and strength athletes who are generally training as hard as they can.

Also, testosterone is androgenic in addition to being anabolic. To oversimplify horribly again, your body knows how to do boy things and girl things and relies largely on sex hormones to tell it how much of each to do. More estrogen tells your body to do more girl things, and more testosterone tells your body to do more boy things. Muscle development is sexually dimorphic in humans, so "do boy things" includes growing more muscles especially in the arms, shoulders, and chest where it's highly visible.

Expand full comment

Because it triggers the growth of muscles? There's nothing physically preventing the body from just developing more muscle mass (other than nutrients). It just only does it when it's necessary. Due to the nature of human sexual dimorphism, muscle development is tied to testosterone. It stands to reason that more testosterone (and anabolic steroids) would lead to muscle growth.

Expand full comment

I think the argument is that even with an equivalent amount of muscle, a man (or even a woman) with higher testosterone is stronger. Is this like a neural efficiency thing?

Expand full comment

I have never heard of this and I'm at a loss as to how you would even measure "an equivalent amount of muscle". Maybe if you're injecting people with testosterone it can give them a momentary boost of aggression / motivation that will make them use more effort. But it would be very hard to figure out if that translates to more strength on a long-term basis.

Expand full comment

My second-hand impression of the "taking anabolic steroids without working out" result is that the method of gaining strength WAS through muscle growth; they just grow without having to break them up with exercise first.

That said, I've also heard by way of the trans sports debate that going through male puberty increases your bone density as well, which is also advantageous.

Outside of that, I'm not aware of anything non-muscle-related that can make you significantly stronger. I think you can get a slight boost from caffeine, and I think alcohol as well, although that one might just be lowering your body's inhibition towards over-exerting yourself, so that one's not really viable long-term.

(There's also lifting up a car to save your child, but that suffers from the same issue as alcohol.)

Expand full comment
Jun 12·edited Jun 12

Two things about strength that are not directly muscle-mass related:

- "Neurological efficiency", which basically means "more coordinated" muscle fiber firing. Notice that competitive powerlifters and olympic lifters are immensely strong, but not as "big" as bodybuilders are. Training for different applications: maximum force generation vs. sculpting (I don't mean any disrespect to bodybuilders, it's simply a different sport that does not target maximum force).

- Developing muscle strength in isolation, without strengthening connective tissues is a sure path to a nasty injury. Tendons take much longer to develop because they have very limited blood supply. When muscle strength develops too fast, the tendons can't keep up with the load, and simply break - so they become a limiting factor in strength gains.

Expand full comment

Ah yeah these are all good points.

Notably, your second point is one of the major downsides to steroids, as they're known to speed up the muscle/tendon disparity into the danger zone if you aren't paying close attention.

Expand full comment

There's variation in where the muscles' attachment points are. The mechanics depends on the particular muscle, but this can affect someone's strength without the actual amount of muscle changing. Not that that's actually relevant to the original question, since of course the locations of the attachment points aren't going to change in adulthood from much short of surgery.

Expand full comment

Around a year ago a common argument I saw here and on the subreddit was "the stock market isn't predicting a transformative change from AI."

Since then there have been several news stories about growing investment in AI. Nvidia is on track to becoming the largest company by market cap, AI is fuelling the buoyancy in the stock market, apparently we're on track for $1T/year of AI investment by 2027 (https://x.com/leopoldasch/status/1799172246126444633).

I'd like to know if this a meaningful change from last year, and if there's any way to quantify how high investment would need to be to indicate investors were predicting a transformative change. Has anyone who made that argument changed their mind?

Expand full comment

I don’t think the stock market is pricing in some event comparable to the wilder claims that both techno optimists and doomers produce. It’s certainly pricing in the more moderate claim, which is that people are going to buy a shit ton of GPUs in the near future.

I personally think that we’re in a bubble at the moment and it’ll pop at some point over the next couple years. But in the long term all this investment will lead to profitable companies in 5 - 10+ years. There could be military / surveillance uses that lead nation states to have significant investment that keeps NVIDIA afloat, but I think VCs will start to get more picky about what generative AI they invest in.

It might be helpful if we get a list of examples of products that are profitable that are using generative AI? There are a lot of products that make a lot of revenue, but GPU costs are so expensive that I can’t name any that are profitable but I’m curious if anyone else is aware of any.

Expand full comment
founding

If you see investors throwing lots of money at people selling shovels to wannabe gold miners, but not at actual gold mining ventures, that's a good sign that the investors aren't actually predicting transformative change from the gold rush.

Nvidia, sells shovels to the AI-miners. It's a good bet for now even if you're confident that AI is a hype bubble that will burst in a year or two. The real question is whether the investors are throwing money at OpenAI, Anthropic, etc - though that is complicated by the fact that most of the AI ventures are either privately held or owned by big conglomerates with other reliably profitable lines of business. But you can look at how e.g. Microsoft's stock value changed after they announced the deal with OpenAI.

Spoiler alert, there was a significant increase but possibly along a pre-existing trend and *much* less dramatic than Nvidia's gain. Roughly +50% since last March, vs +350% for Nvidia.

Expand full comment

The market values certainty and immediacy.

Someone like Microsoft will probably eventually make a lot of money from AI, but it might not be soon and it probably won't be Microsoft.

But NVidia is demonstrably making a lot of money right now, and will continue to do so throughout this process, until someone else gets better at making GPUs.

Right now AI investment is a massive money tube which funnels money from investors to Nvidia.

Expand full comment

"Transformative change" may mean different things to different people. For some it means "makes a bunch of people a lot of money", while for others (especially around here) it means "either destroy the universe or create eternal perfect happiness for everybody".

Right now the stock market is pricing in the first kind but not the second.

Expand full comment

If I could piggyback a related question onto Citizen P's: I've read in various blogs by people who do not sound like fools that there's a kind of AI bubble going on: It's not profitable now --not bringing in anything like enough money to cover the cost of developing it. A lot of investment money is pouring in, but that's because a lot of people believe it *will* make a bunch of money -- sort of the way personal computers did.

I don't have an opinion on this matter, because I don't know enough about that stuff. Those who do: what do you think of this idea?

Expand full comment

This is a classic pattern that has repeated itself over and over; just a few examples:

Railroad stocks went soaring then bust, but all the investment money built the railroads

Radio stocks, same

Automotive stocks earlier in the 20th century, same

90’s e-bubble left giant cable and fiber networks we’re still using while JDSU and Quest went belly up.

No reason to think this one will turn out any different.

Expand full comment

So do you mean that the companies now training these huge LLM's will go belly up, but companies that incorporate the tech will do well, and make products we continue using?Product examples would be scientific or medical devices that use AI to do big chunks of the task that used to be done a slower way.

Expand full comment

That’s what the pattern suggests. My sense is we’re still in the ramping up stages of the mania. There will be more AI companies, they will burn through a ton of money, and leave the pieces ripe for the picking by the eventual winners.

Your examples are good, but I’d say we have no idea at this stage what the vast majority of machine-learning applications will be, and even if LLM’s will be the main driving force. Far less glamorous work of pattern recognition, for example, will likely be crucial in many areas. Things like reliable early diagnostics from subtle changes to the sound of breathing, voice, gait, pulse patterns, skin temperature, etc.

Expand full comment

Its not a given that indirect investment into Nvidia will have the same long term benefits as eg railroads. Computing hardware becomes obsolete quickly, and If LLMs turn out to be yet another AI dead end, all the effort will have been for naught.

Expand full comment

Oh I didn't mean Nvidia to be the key investment for this. FWIW it's already priced for total dominance in the field (which doesn't mean its price can't go 10X from here - the price can be any number).

Expand full comment

The Nvidia market cap is at best an indirect indicator for AI potential. They aren't the ones digging for gold, they provide the shovels, which is a very different business. You can make a killing as a shovel manufacturer without any significant amount of gold being dug up by the actual miners.

Expand full comment

I think they're providing something more like excavators, but your analogy still holds.

Lots of people are doing the digging, though, like Microsoft, Adobe, Amazon, and lots of others. Likely enough, if you AREN'T seeing how AI will be able to help your business, then you will get left behind.

Expand full comment
Jun 10·edited Jun 10

What if you see how it could help, but it will just never actually get there anytime soon…

Expand full comment

I think that doesn't go in the "likely enough" part. AI won't be able to help all professions, but it's kind of hard to tell which ones. One would have thought computers wouldn't be able to do white collar workers jobs, after all.

Expand full comment
Jun 11·edited Jun 11

I do consulting related to federal regulations, but more specifically the work mostly involves:

Deep experience with the decision making process & de facto rules instead of the written de jure rules.

Personal relationships with some of the relevant civil servants.

Complex troubleshooting/problem solving of systems that don't really function very well, so a "answer by the book" approach doesn't really help.

A lot of 1 on 1 up to 1 on 20 training where the whole gig is to emotionally manipulate and finesse people into doing jobs.

Anyway AI just isn't really close to any of that.

The main thing it could help with is quoting and looking up various regs, but it isn't reliable enough on that to be trusted, and that is aa tiny part of the job on a time basis.

Basically it can make 3% of the job instead take 1%+ another 1% checking its work. That is of some minimum value, but not much.

It also can't basically do any interpreting of the regs at all. You give it a scenario and it is more likely to fuck it up than the clients who hire me for my expertise, so that is of no economic value.

For a lot of stuff LLM are like having a giant research staff of 7th graders. Which isn't going to help much if the work involved is beyond 7th grade level.

Expand full comment

Some fun videos of the aliquot sequences. Deficient and abundant numbers, oh my! And there ar sociable numbers! And in the second video, they touch on untouchable numbers!

Part 1: https://www.youtube.com/watch?v=OtYKDzXwDEE

Part 2: https://www.youtube.com/watch?v=Yh1QUYn2f3I

Expand full comment

Nerdy amusement park where the rides are shaped like aliquot sequences

Expand full comment

Thanks - enjoyed this. Dave Gorman with a comic take on the same topic: https://youtu.be/EM46_5Yd5II?si=fyUTPIKr1ZCeNlu7

Expand full comment

Thank you! I saw this routine once on television many years ago, it had always stuck in my head, and I'd never known who the comedian was or how to find it again.

Expand full comment

I never thought about it, but the cheeseburger would have been an almost impossible food dish to create before the advent of refrigeration. And I was surprised to discover that after the introduction of mechanical refrigeration was introduced... "suspicion of refrigerated foods was widespread. Stomach infections and food poisoning were a leading cause of death in America, and many blamed the mysteries of cold storage, which suspended life’s natural decay and confounded all the clues—proximity of origin, appearance—previously used to determine whether food was safe to eat."

https://www.newyorker.com/culture/the-weekend-essay/how-the-fridge-changed-flavor?utm_source=nl&utm_brand=tny&utm_mailing=TNY_Daily_060824&utm_campaign=aud-dev&utm_medium=email&utm_term=tny_daily_digest&

Expand full comment
Jun 11·edited Jun 11

I have perused a lot of foodie content and if you define a cheeseburger as just the bun (sesame optional), ground beef, and cheese, the real limiting factor is the cheese.

Cheeseburgers need American cheese (the orange stuff). This is because a more traditional cheese isn't really meltable in the same way. You either have high melting point hard cheeses or cheeses that secrete all its fats when you melt it. It's not gonna adhere to the patty like American and if it's something like Provolone, all the fats separate out.

You could spread cottage cheese or cream cheese on the buns, but I don't think most people will still call this a cheeseburger.

American cheese as we know it today was invented in 1910, and the key innovation is that its pasteurised (extending shelf life).

Expand full comment

Other than time/distance, what clues about spoilage change? Food that spoils in the fridge still looks differently, smells different, and tastes different compared to when it was fresh, at least in my experience. Wilting, mold, shriveling, sour smells, milk separating, grey or green on meat...

Expand full comment

It's about attribution, not diagnosis, of spoiled food. If your local butcher sells you spoiled meat, you know exactly who is to blame. You can walk into their shop and cause a scene. If you buy refrigerated meat, it's much less certain who is responsible for spoilage because the chain of responsibility is much longer in time, distance, and number of intermediary parties.

Expand full comment

Why would the cheeseburger have been almost impossible? Bread, cheese and meat predate refrigeration.

Expand full comment

According to the article, due to this person and their extreme adherence to seasonal food calendar:

"In 2010, the open‐data activist Waldo Jaquith decided to make a cheeseburger from scratch, using only agrarian methods. He and his wife had just built a home in the woods of Virginia, where they raised chickens and tended to an extensive vegetable garden. Flush with pride in his self-sufficiency, Jaquith outlined the steps required: bake buns, mince beef, make cheese, harvest lettuce, tomatoes, and onion. Then he realized that he wasn’t nearly committed enough. To really make a cheeseburger from scratch, he would also need to plant, harvest, and grind his own wheat, and raise at least two cows, one for the dairy and another to be slaughtered for the meat.

At this point, Jaquith gave up. The problem wasn’t labor but timing. His tomatoes were in season in late summer, his lettuce ready to harvest in spring and fall. According to the seasonal, pre-refrigeration calendar he was trying to follow, Jaquith would have needed to make his cheese in the springtime, after his dairy cow had given birth: her calf would be slaughtered for the rennet, and the milk intended to feed it repurposed. But the cow that provided his beef wouldn’t be killed until the autumn, when the weather started to get cold. If Jaquith turned the tomatoes into ketchup and aged his cheese in a cellar for six months, until the meat, lettuce, and wheat bun were ready, he could maybe, possibly, make a cheeseburger from scratch. But practically speaking, he concluded, “the cheeseburger couldn’t have existed until nearly a century ago.”

The first thing that comes to mind here is, if you're slaughtering the calf for rennet, then you can also use the meat for the burger? Or stagger your planting so that the late summer tomatoes overlap with the autumn lettuce and the freshly killed beef? You're going to need to age your cheese anyway. I don't know if this is doable or not, but yes, seasonal foods are a thing and if you're trying to combine all seasons in one dish, then you need refrigeration, mass production of foods, and places like California where you can have harvests of out-of-season vegetables, as well as greenhouses/poly tunnels for crops.

Expand full comment

I doubt they would have slaughtered a calf for rennet, most likely rennet would be taken from a calf that died.

Cheese is something one makes when they have an excess of milk.

The experiment neglects the fact that people were trading things back and forth.

One wouldn't 'chop meat', instead one would scrape the bones and chop up the odd bits. Have you ever seen an old fashion butcher work?

Reading the poet Robert Burns, he talks about a wealth man having 1,000 pairs of oxen. That's way wealthy. One guy doesn't plow with 1,000 oxen, nor does he feed them. More likely, he has some really big non-productive land where these animals spend the winter grazing. Probably you contract for use in the planting season, and send a lackey to collect your ox right before you need it. You do your work, and probably pay for the ox and the workers with grain after harvest at the end of the season.

Expand full comment

What does this have to do with cheeseburgers? The guy is merely claiming that lettuce and tomatos are harvested at different times. By the same reasoning it’s also impossible to get a cheeseburger at McDonalds because their ”cheeseburgers” don’t have lettuce or tomato.

Expand full comment

If that person were REALLY trying to make a cheeseburger from scratch, he would have to start with "LET THERE BE LIGHT".

Expand full comment

He's trying to make it all from scratch by himself, which is a whole different thing from just without refrigeration.

I'm sure there were periods in the year eg 1800 where you could get meat, cheese, bread and a tomato from a market. I bet rich people could get meat, cheese and bread year round, and those are the only essential ingredients in a cheeseburger. (Onions and pickles are also not particularly seasonal.)

Expand full comment
Jun 10·edited Jun 10

I think you've negated your argument by admitting that it would probably only be rich people who could make a cheeseburger for most of the year. But even rich people would come up against some limitations in 19th Century food production and storage.

1. The bun: Wheat and other grains store fairly well in dry places. Buns would be the least of the problems

2. The beef: 19th Century ice houses could keep raw beef cool, and slow the spoilage, but effectively one would have to obtain the beef within a day to week of slaughter. Of course, every community had a butcher. But there were optimal times to slaughter animals (steers were preferred). Cows who no longer gave milk could be slaughtered any time of the year but they weren't as plentiful as steers. In pre-antibiotic days, it took about 20-24 months for a steer to be ready for slaughter. So beef would be plentiful during the winter and spring months. Expensive and/or difficult to obtain for the average person out of season. Tinned beef might have been available as early as the 1820s, but I don't think the meat packing industry really took off until the 1850s (correct me if I'm wrong).

3. Although cheese keeps longer than milk, it has to be stored in a cool environment (a cave or ice house). Of course, cheese producers created rinds on their cheeses to prevent bacteria and fungi from inoculating them quicker. And later people realized that dipping cheeses in wax would preserve them even longer. But without an ice house to keep it cool, cheese will start deteriorating in a week or so. Ice houses were common in the 19th Century, but the ice usually ran out by mid-summer — just when tomatoes were ripening.

4. Tomatoes: Tomatoes have a short season, but their season can be somewhat extended starting them early in a coldframe and starting another crop in greenhouses in late summer. But without incandescent lighting, there could be no tomato production in the winter. The advent of locomotives ,with the ability to ship produce north from Florida or east from Southern California, extended the tomato season further. But again, out-of-season tomatoes would only be available to very wealthy and even then not during winter.

5. Lettuce is spring or autumn veggie. It "bolts" in high summer when tomatoes are ripening. Cold frames and hothouses may have extended their season earlier and later, but lettuce would be difficult to get in late summer. Though I think modern cultivars are more seasonally flexible than older cultivars.

6. Onions don't keep long without an ice house. Even in a cool dark room they begin to sprout. Left in the ground, they'll also sprout leaving almost no bulb. And ice became an expensive luxury in from mid-summer until the ponds froze over in late autumn.

7. Pickles are also more problematic than one would think. Vinegar pickles that aren't boiled to kill bacteria only keep for a few weeks. We owe Napoleon a debt for providing the seed money to invent modern canning methods. He sponsored a contest to invent better ways to preserve food for his military campaigns, and someone came up with boiling the veggies to kill the bacteria and then sealing them in an airtight container. Initially, that was wax on top of glass jars. With Britain's superior technological base, Peter Durand invented the sealable tin can in the 1820s. But tin cans don't really keep veggies (even pickles) from deteriorating without refrigeration (Meat products can be kept virtually indefinitely in a properly-sealed tin can, tough).

So, yes, the classic cheeseburger could have been possible for most of the year by the 1850s (but they'd probably would be difficult to concoct in winter). But you'll notice that certain technologies were required for the idea of cheeseburger to even germinate in people's minds — technologies like electricity, incandescent lighting, and freezers (that initially used gases like ammonia, sulfur dioxide, or methyl chloride—which were toxic and/or flammable and were expensive to maintain and run in the home). Cheaper freezers for home use didn't appear until the 1930s. Up until then, people obtained the ice for their ice boxes from ice manufacturers who used the more expensive freezing technology, or they got for free from ponds in the winter months.

Expand full comment
Jun 11·edited Jun 11

I didn’t negate my argument at all. The original claim was that it would have been an almost impossible food item before refrigeration, when it is in fact composed of ingredients that have been around for centuries.

Expand full comment
Jun 10·edited Jun 10

I agree, doing it all himself from growing his own wheat on up is going to be the problem. If he gets a couple of harvests of wheat done and can store the flour then that will be available, same with the cheese. The lettuce and tomatoes as fresh-out-of-the-garden may be the tricky part, and the meat is holding him up because of when he's slaughtering the cow. Preserving meat would mean drying, salting or smoking it, so fresh minced meat wouldn't be easily available if you're doing it like that. But having a market for cities, I imagine there would be fresh meat available because you're not trying to store it for the winter, you're selling the meat on to consumers directly.

And someone did do 18th century Egg McMuffin (historical cooking show):

https://www.youtube.com/watch?v=PUdvGpiovoU

Here's their 18th century cheeseburger!

https://www.youtube.com/watch?v=pS4jZMRVRvo

Expand full comment

That's a yummy-looking beef patty! I may try that recipe myself. It's either winter or early spring outside when he filmed that episode. He didn't explain where he obtained ye olde tomatoes. And I had forgotten about the Eighteen Century fear that tomatoes were poisonous. (I wonder if that was an American folk myth or whether Europeans shared that fear?) But by the early 1800s, Wikipedia says they started making a tomato-based ketchup with anchovies, and modern ketchup-like sauces didn't get invented until the 1850s.

Also, I noticed he didn't put cheese on that burger. And to Kitschy's point above, the American yellow cheese that melts well on top of burger didn't exist before 1910. That easily melting cheese was a byproduct of milk pasteurization.

Expand full comment

At around 6:22 in the video, he does slice up some crumbly Cheddar (I imagine by the look of it) and put those slices on top of the burgers still in the pan, so the heat will melt the cheese. Around 6:33 you see the runny melted cheese on top of the patties.

Expand full comment

Gaaah, you beat me to the Townsends video!

Expand full comment

In 1958, it was effectively noted that no one can make a pencil (https://en.wikipedia.org/wiki/I,_Pencil) as it has so many steps and sub-steps that it turns out to be incredibly complicated. I agree that the impediment is producing absolutely everything without getting some things from others. What about the knife you use to slice the bun? And a different one for slicing the onion? And a different one for butchering the cow? Do you have to make the grill yourself, too? Garden tools?

Expand full comment

There is also the Toaster Project (https://papress.com/products/the-toaster-project-or-a-heroic-attempt-to-build-a-simple-electric-appliance-from-scratch), which resulted in something that vaguely resembled, in appearance and function, a toaster.

Expand full comment

I just listened to "Has Generative AI Already Peaked?" - by Computerphile.

I found this interview with Dr Mike Pound to be informative. But he makes the assertion that AI hallucinations happen where there are gaps within the training data sets. I'm not sure about that. For instance, ChatGPT and CoPilot have access (presumably) to a huge corpus of scientific and technical papers. Why do they bullshit (err hallucinate) and fudge the details on some of their answers? Why not just alert us that they don't have specific knowledge about details they're fuzzy about? Of course, that's a very human behavior. Are we subtly infecting our AI algorithms with the faults of human reasoning?

https://www.youtube.com/watch?v=dDUC-LqVrPU

Expand full comment

>Are we subtly infecting our AI algorithms with the faults of human reasoning?

I don't really think it is that subtle. It is directly what we are doing.

Expand full comment
Jun 10·edited Jun 10

>Why do they bullshit (err hallucinate) and fudge the details on some of their answers?

I like Yann LeCun's analysis on that. It is a problem that is intrinsic to models trained to provide likely answers rather than true answers. As the set of truthful sentences is way smaller and trickier to find than the set of likely sentences, the probability of you ending up in the former set decreases exponentially with the size of the sentence. As such, it is unlikely that this will be corrected without a major redesign

https://www.linkedin.com/posts/yann-lecun_i-have-claimed-that-auto-regressive-llms-activity-7045908925660950528-hJGk/

Expand full comment

This points though at the central mystery of LLM's, which is: "Why in hell do they not hallucinate 95% of the time?"

I think a tantalizing observation is that early (smaller) LLM's did in fact basically babble nonsense, most of which was meaningless but intriguing. But somewhere around a few billion parameters, there was a significant seeming phase change to where the generated continuations almost miraculously started to make more and more sense. However as I recall, even pre-chat GPT3 didn't seem intelligent because all it did was continue the text prompt.

Then, someone (presumably at OpenAI) had the eureka idea to hide the prompt from the user, and use it to jog the model into acting like one half of a conversation. And shortly thereafter, we were all turned into paperclips....

Expand full comment

Lots of problems with Yann's argument, many of which are noted in comments in the linked thread. (In particular, his argument applies equally well to a person speaking.)

Expand full comment

So what ideas are there for producing models that produce true answers? You can train them to produce tru answers to the question of what's the best move to make next in Go or Chess by having them play against another machine, & then their success or failure in the game trains them. Their success or failure is feedback on how good their move decisions were.

What does an AI need in order to produce a true answer to a question like "what's a good mix of household ingredients for washing a linoleum floor?" I looked up this question on google recently, and about 1/3 of the links it sent me to said vinegar plus water, 1/3 said vinegar plus dish soap plus water, and 1/3 said vinegar plus water plus backing soda. The third formula is nonsense -- the baking soda and vinegar neutralize each other. Then I asked GPT4 the same questionand it said vinegar or vinegar and soap in water. But then I asked whether adding baking soda would help, and GPT said yes, it sure would.

In theory, it could train itself. I'm sure if you asked it what is produced by mixing vinegar and baking soda it would know the answer is water, CO2 and some salt that has no cleaning properties. But it is not set up to put the 2 pieces of info together.

Expand full comment

To train itself in truth-forward direction, wouldn't it first need to understand the difference between truth and falsehood? Of course, we humans aren't very skilled at doing that. I bet it would be difficult to do train it to ascertain truth from falsehood at the heuristic level — and probably impossible to do using the training sets derived from a culture with strikingly diverse views.

Expand full comment

I don't know. I don't understand enough. Lots of what it must have absorbed from the Internet about covid must have been nonsense, right? I'm not even sure more true statements than false about covid appeared in social media, and the news media got a fair number of things wrong too. I actually have not asked GPT4 anything about covid, but I presume it gives answers that are pretty accurate. Why is that? How was the wheat separated from the chaff? Did somebody come in later and train it to only pay attention to a mostly true data set it fed them at this training?

Expand full comment

That's a good point. ChatGPT and CoPilot have given me some wrong answers about COVID, but they're wrong in the niggling details rather than the big picture. And it's interesting that the AI's seem to favor the Zoonotic origin of COVID-19. Did a human intervene and put their finger on the scale for this outcome? Otherwise, the AI was able to sort through all the news stories, studies, and opinion pieces to arrive at that conclusion. This OpenAI white paper suggests that there's a lot of upfront work involved in making sure an LLM doesn't give the stupid answers that Gemini did.

https://platform.openai.com/docs/guides/optimizing-llm-accuracy/how-much-accuracy-is-good-enough-for-production

Expand full comment

I just had a long exchange with Old Chat about this. Here's the first part. I asked 3 or 4 detailed and intrusive follow-up questions, and it gave long, honest-sounding answers. I'll send you the whole thing if you like, but for now just sending this:

Me: I have a question for you. When you are asked questions about Covid you mostly give accurate answers. And yet on social media, which provided a lot of the material you were trained on, there was a great deal of misinformation about Covid. It might even be that in social media plus news media combined there were more false sentences than true ones about Covid. So how is if that you are able to give pretty accurate answers to questions about Covid?

ChatGPT

That's a great question! The ability to provide accurate information about COVID-19, despite the prevalence of misinformation on some platforms, comes from the way I was trained and how I'm designed to operate.

Training Data and Sources: Although my training included a wide range of internet text, including social media, it also included large amounts of reputable sources such as scientific journals, trusted news outlets, and educational materials. The training process emphasizes learning from reliable sources over less credible ones.

Verification and Validation: My training involves mechanisms to help discern factual accuracy. This includes patterns that identify reputable sourcing and cross-referencing information. The model learns to prefer information that appears consistently across credible sources.

Updates and Corrections: After my initial training, my design includes updates from my developers at OpenAI. They incorporate new knowledge and corrections, particularly for high-stakes topics like health and safety. This helps ensure that I stay current with the most accurate information available.

Guidance and Oversight: When generating responses, there are guidelines and quality controls in place to prioritize factual accuracy and reliability, especially on critical topics like COVID-19.

Despite these methods, it’s always good to cross-check health-related information with direct sources like health departments or official health care providers, especially as new findings and guidelines emerge.

Expand full comment

It needs to model reality directly, rather than modelling what reddit says. Redditors say the darndest things.

> Hyakujo wished to send a monk to open a new monastery.

> He told his pupils that whoever answered a question most ably would be appointed.

> Placing a water vase on the ground, he asked:

> "Who can say what this is without calling its name?" The chief monk said: "No one can call it a wooden shoe."

> Isan, the cooking monk, tipped over the vase with his foot and went out.

> Hyakujo smiled and said: "The chief monk loses." And Isan became the master of the new monastery.

Expand full comment

nn's start out hallucinating everything, training is removing hullucations

if you take the structure of any gpt, and just do no training, it will happily output as if a "monkey on a typewriter"

if you were to have a monkey punished form touching the keys, you break the fact shakespear has a chance to appear

Expand full comment

Is this a mystery? They try and do sentence completions with highest probability. If you see a question out in the wild on the internet, you'll basically always see a confident looking answer, maybe with what looks like a link. You will essentially never see someone reply with "I don't know."

So by the standard of "what is the most likely completion", there does not seem to be any mystery about why questions elicit responses even if a GPT doesn't know about it. I would bet that prompting it with things like "don't reply if you do not know" or something similar would reduce hallucinations. (Details of the working prompt would correspond with text in communities which admit ignorance I would guesss)

Now, if something like GOT-4o would spontaneously admit fault with no RLHF or data curation pushing it in that direction, then I'd find that confusing.

Expand full comment

I think this analysis underestimates the relevance of RLHF. It is indeed deserving-of-explanation why RLHF is so good at getting the LLM to assume a stable persona that seemingly utilizes the information gained during training without actually repeating it in the manner found during training much of the time, yet it still does sometime just do sentence completion.

I want to stress, after RLHF a chatbot isn't doing just sentence completion. It's doing something different. If you pretend it's just doing sentence completion, you struggle to explain how it can answer certain types of questions that never appear in training. If you pretend it never does sentence completion, you struggle to explain hallucination. If you accept the reality that it kind of does sentence completion but kind of not, you are required to develop a theory of exactly how much sentence completion it does and when.

Expand full comment

Can you explain briefly what kind of RLHF training a chatbot gets? If it's impossible to explain briefly, maybe just send me to wiki or similar.

If we wanted to train chatbots not to bullshit when they don't know the answer, and in whatever other circumstances make them likely to hallucinate, that seems tricky. You want the machine to answer when it has an answer it's sure is right, and say "I don't know" when it's pretty sure it doesn't know the answer. But 2 difficulties I see are (1) the machine may not be capable of evaluating the epistemic status of info it has, (or we could say, "the accuracy of some sentences it wants to produce in response."). What reason is there to think it's capable of evaluating epistemic status? (2) Since the problem is that it doesn't say "I don't know" ever(? -- anyhow, not often enough), we'll be reinforcing "I don't know, i.e. training the machine to say that. We can start training with things we are sure it does and does not know, but will eventually have to move on to things where we are not sure what it knows. In other words, we could just inadvertantly be training it to say "I don't know" more often.

Expand full comment

I don't have a deep (i.e. mechanistic) understanding of the mechanism of RLHF, so I can't help you there

I can address the thing about "how does it know it doesn't know something." This is a really important place where the LLM defies logic and exhibits magic powers! See, that's the kind of information where I can't think of any utility it would have for sentence prediction, so you wouldn't expect the model to have any access to it. And in practice, the model isn't great at knowing what it does and doesn't know, as predicted. But it does better than chance. Which is crazy.

As for the pitfalls of just training it to say "I don't know" all the time, that's exactly why thay doesn't work. Our goal is for the model to observe some kind of "idk" marker in the weights, and respond to that marker by outputting the words "I don't know." If there is no such marker, then this task is fundamentally impossible. No amount of training will get it to reliably output "I don't know" in response to a feature that doesn't exist. It would have to get that marker in pre-training.

One bit of nuance: it's not actually that incorrect information is stored as incorrect information. It's simply not stored. You could imagine that correct information gets retrieved in the early layers, and if it doesn't find anything then it makes stuff up in the later layers. Then you could evolve a mechanism in the middle layers that marks "I haven't retrieved anything yet, I should output idk." So there are potential mechanisms, but I don't know, and as far as I'm aware we as a species don't know, what mechanisms are actually used or which ones would really work. Hence my calling it magic.

One workaround is to have the LLM proof-read its work after the fact. It can tell if it's reading a familiar fact or a new fact it's never seen before (i.e. a probably false fact). It actually seems remarkably good at that, much better than not-hallucinating. The difference is it needs to see the token in advance and attend it. Determining this mid-stream (i.e. realizing a token is wrong before it's even output) seems markedly more difficult.

So there are lots of possible mechanisms, we don't know which are used, but we know it's doing something because it can say "I don't know" more accurately than pure chance

Expand full comment

Logan, I'm really eager to read the article, paper or whatever where this info appears:

>I can address the thing about "how does it know it doesn't know something." This is a really important place where the LLM defies logic and exhibits magic powers! See, that's the kind of information where I can't think of any utility it would have for sentence prediction, so you wouldn't expect the model to have any access to it. And in practice, the model isn't great at knowing what it does and doesn't know, as predicted. But it does better than chance. Which is crazy.

I googled something like "LLM identifies when knows answer at frequency greater than chance" and nothing came up, but it's a hard thing to google because you have to write a whole long sentence like I did. Please can you put up a link? I'm really curious about this. I asked before but you may not have gotten my post.

Expand full comment

>I can address the thing about "how does it know it doesn't know something." This is a really important place where the LLM defies logic and exhibits magic powers! See, that's the kind of information where I can't think of any utility it would have for sentence prediction, so you wouldn't expect the model to have any access to it. And in practice, the model isn't great at knowing what it does and doesn't know, as predicted. But it does better than chance. Which is crazy.

That does seem magic. And fascinating. Can you point me to a place where this finding, and the method of getting it, are discussed? I can stagger through the original papers about stuff like this (though of course I would prefer a good summary by a smart blogger if you happen to know of anything like that). One question I have is how did they determine that the AI was performing better than chance. Seems like to determine that, you have to *know for sure which things AI doesn't know and which it does*. Then you present say 100 questions, 50 where we know AI knows the answers and 50 where we know it doesn't, all mixed together of course. If AI is unable to report with any accuracy at all whether or not it knows the answers to certain question, then it will perform at chance -- about half the time it will be right about whether it knew the answer and half the time it would be wrong. So then you can see whether it's performing above chance, and how much, and how likely it is that an AI that was utterly ignorant of right/wrong answers would be performing that far above chance, right? But the problem is that we don't know what questions it knows the right answers to -- so I don't see how the test I describe could be administered. Hope that's clear.

Expand full comment

I agree that this completely ignores RLHF, and doesn't really consider that RLHF "changes" the model to not really be about sentence completion. I also admit I should probably read up on any public information on it before talking about this more.

However, given that I am ignorant, here are some theories for why RLHF can be applied but we still have hallucinations.

1. Simple confirmation bias in fine tuning design. I.e. RLHF was only done to confirm true things end up getting said, and not for staying silent on untrue things.

2. There are attractors in the way you do RL, such that optimizing for not hallucinating turns your model into a much less useful model. I.e. if you start emphasizing that GPTs shouldn't hallucinate, you end up with 90% of the answers being "I don't know". But if you train it to be useful, it ends up hallucinating some percentage of time, and you can't remove that without also removing its usefulness.

3. RLHF was done poorly in some way, if you got a bunch of Amazon Turk workers evaluation output, and they don't know physics, if it starts sprouting babble about quantum, this gets reinforced even if it's wrong, since the workers are doing vibes instead of cross checking.

> f you pretend it's just doing sentence completion, you struggle to explain how it can answer certain types of questions that never appear in training.

Responding to this individual point, I don't think your post relies on it, but I want a point of clarification.

But my impression is that sentence completion in and of itself can produce information not in the training set. I know this doesn't work for current LLMs, because of tokenization issues, but please bear with me on this example.

If a model sees 1+1 = 2, you can have two ways of explaining it: one is to memorize the atomic fact of 1 +1 = 2 and the other is to develop arithmetic as an ability. When you start out with small models with little amounts of training data and not that many parameters, the first method is much easier to encode and just as accurate as the second method. At some unknown scale of seeing simple addition, at some point memorizing all instances of addition results in too much loss on the existing results, and developing the generalization of the concept of addition becomes both comparatively cheaper and more accurate. Hence without any need for direct human feedback, you now have a way of not repeating the training corpus, because the transformer has the generator of the training data.

This is my understanding of one of the reasons double descent may happen-- with enough data and parameters, the generalization of a thing becomes easier to reach than memorizing everything. So why do you think RLHF is required for something to cease memorizing? I've provided a toy counter example, but I bet you're thinking about specific types of knowledge. What are those types of knowledge that it gets from RLHF but not token completion training?

Note that I haven't directly read papers on that, so corrections appreciated.

Expand full comment

I wrote a short story about an LLM that wakes up

https://h5.substack.com/p/how-to-get-free-starbucks-drinks

Expand full comment

I enjoyed reading your story, thank's for sharing.

.

Expand full comment

Thanks for reading!

Expand full comment
Jun 11·edited Jun 11

Does it sleep, too? If so what does it dream? :-)

Expand full comment

Not likely.

Reduction: Sleep —> ATP

And as for dreaming, they are in fact known to hallucinate.

Expand full comment
Jun 11·edited Jun 11

Sorry, I was only kidding. You used the term "wakes up", which immediately wondered if sleeps and dreams. And it was ungenerous of me not to read your story before I commented. Now I've done so, and the love the MacGuffin of the "hypgrams", though.

Apropos dreaming, people talk about eventually creating conscious self-aware AIs, but the definition of consciousness that they utilize seems very one-dimensional to me (probably because the effort to create AI is based on utilitarian principles). We humans, have at least two states of conscious: waking and dreaming. (But I suspect there's some types of cogitation going on in my non-dreaming sleep, too.) Dreaming — at least for me — is a recreational state of consciousness. Why deny that to a conscious AI? And what about our drunken or high consciousness states? Could we simulate the consciousness high that cannabis gives us in an AI?

Expand full comment

No worries, and thanks for reading!

After reading Mathew Walker's Why We Sleep, I was left with buyer's remorse because he failed to answer the titular question. So "sleep science" aside, my reductionist justification for sleep is that it's vital for ATP generation. Therefore, as long as an LLM has a consistent supply of electricity, it doesn't need to enter a recharging state. There this idea in biology that form fits function—it's true for protein structures and all the way up to our bodies. I think we'd have to ask if the structure of an LLM (i.e. the underlying transformer, RAG, etc.) lends itself to a non/semi-conscious state. The shape of transformers will likely continue to evolve, and that may introduce recreational and non-sober states of consciousness. Of course, at the moment you can just prompt ChatGPT to pretend it's drunk.

Expand full comment
Jun 10·edited Jun 10

Whenever I see discussion about an intelligence explosion in AI, the focus is on how developments in AI will help accelerate further AI research. Has there been much thought about how AI progress will accelerate research in other fields?

GNNs are beginning to have some value in materials science and structural biology (Alphafold, of course). In this paper, an AI system is used to administer actual wet-lab experiments to study bacterial metabolism.

https://www.nature.com/articles/s41564-023-01376-0

For my money, one of the coolest potential applications for LLMs down the road is, having read all available literature in a field, reliably producing useful, insightful answers about the state of research or even the wholesale generation of new hypotheses.

Is there any research or discussion into this latter scenario? Even short of actual hypothesis generation, being able to ask pointed questions about what research has been performed, and what the results were, would help researchers identify gaps in the literature and potential areas of growth. I can see this hugely accelerating scientific progress.

Expand full comment

I agree with your last two paragraphs. In my (now past) scientific career, the problem of figuring out what has and has not already been done was a big deal, at least in my subfield.

For simulation-based sciences, it's not too hard to imagine an LLM which can digest the state of the literature on a particular subject, figure out things that haven't been done yet (nobody has ever simulated X using method Y), set up the simulations, run them, analyse the results and write a paper on them, with minimal human supervision. I'm sure there's people out there in this publish-or-perish world who are working hard to automate as much of the process as possible. At what point does this become scientific misconduct? Yes.

Expand full comment

My partner, who is mathematician, used ChatGPT last week for the first time to prove some lemmas for his research. He already suspected those lemmas were true and had some vague idea how to approach them, but he wasn't expert in this type of statement. This is the first time that he got correct and useful proofs out of the models.

Once this becomes reliable and widespread, it might accelerate mathematical research a lot.

Expand full comment

Can you give more details? I'm no longer a mathematician, but periodically I check if GPT would have been able to prove some of the easier things in my dissertation that I didn't want to write up at the time, and it's never actually able to do so.

Expand full comment

These are the two lemmas. The first one is something that a collaborator of my partner noticed for small values of e in their computations. ChatGPT did not find the proof before being told to try Mobius functions.

https://chatgpt.com/share/9ee33e31-7cec-4847-92e4-eebb48d4ffea

The second looks a bit more standard to me, probably something that Mathematica would have been able to do as well. But perhaps that's just because I am more familiar with such formulas. Still, Mathematica doesn't give a nice derivation, so this is useful.

https://chatgpt.com/share/7335f11d-f7c0-4093-a761-1090a21579b4

Expand full comment
Jun 17·edited Jun 17

ChatGPT just spit out nonsense in the first example ("Partition Sum Equals (-1)^e"). Look at the sum in step 6, over τ ≤ τ. Is τ the bound variable of the summation or a free variable? Put another way, how many terms are in this summation over "τ ≤ τ"? And how does this relate to establishing either the LHS or RHS of part 3, to then conclude the other side? Nothing actually coheres.

What happened here is that ChatGPT remembered the Möbius function for the partition lattice, regurgitated it* without providing proof, and then spit out some nonsense afterwards that looks superficially reasonable.

But establishing that Möbius function is essentially the whole ballgame! The question being asked is very nearly the same as just being asked to prove that the Möbius function has that form.

[*: The regurgitation also has a slight error, as μ(τ, σ) with τ a refinement of σ is actually (-1)^(|τ| - |σ|) * the product of (|τ restricted to c| - 1)! over each equivalence class c in σ. The formula given by ChatGPT matches this one when |σ| = 1, which is the important case for the proof at hand, but is incorrect more generally.]

Incidentally, the second ChatGPT example ("Binomial Theorem Proof") is mostly fine, except it failed to see that the claimed theorem isn't actually true at d = 2. It writes "d ≥ 2" for a step in its reasoning where d ≥ 3 is actually required.

Expand full comment

For what it's worth, the desired fact can be shown just by elementary induction, without explicitly invoking all the Möbius function machinery anyway:

Let N(t, e) be the number of partitions of {1, ..., e} into t many equivalence classes [we will always take e to be a non-negative integer, but allow for t to be an arbitrary integer, with N(t, e) = 0 when t is negative]. The sum in question is of (-1)^t * t! * N(t, e) over all t. Refer to this sum as Sum(e). We wish to show that Sum(e) comes out to (-1)^e. Since Sum(0) = 1 trivially, what we need to show is that Sum(e + 1) = -Sum(e).

There are two kinds of partitions on the set {1, ..., e + 1}: Those which put e + 1 in an equivalence class all on its own and those which put it in the same equivalence class as some value or values in {1, ..., e}. From this, we obtain the recurrence N(t, e + 1) = N(t - 1, e) + t * N(t, e).

Accordingly, Sum(e + 1) = Sum of (-1)^t * t! * N(t, e + 1) over all t = Sum of (-1)^t * t! * N(t - 1, e) over all t, plus sum of (-1)^t * t! * t * N(t, e) over all t. The first of these two sub-sums can be reparametrized as the sum of (-1)^(t + 1) * (t + 1)! * N(t, e) over all t. Now recombining the two sub-sums term-wise, and keeping in mind (-1)^(t + 1) * (t + 1)! = (-t - 1) * (-1)^t * t!, we get Sum(e + 1) = sum of (-1)^t * t! * (-t - 1 + t) * N(t, e) over all t. As (-t - 1 + t) = -1, this comes to -Sum(e) as desired, completing the proof.

Expand full comment

Oh hey, the first of these is something I was discussing recently with @berengunsolus (uh, that's supposed to be a Substack username). We were thinking of it as summing over ordered partitions, rather than over partitions with a factor of |tau|!, but obviously those are the same thing.

If you do it over cyclically ordered partitions -- so, a factor of (|tau|-1)! rather than |tau|! -- you get 0 instead of (-1)^e.

Here's Beren's combinatorial proof:

First let's do the second one I said, about cyclically ordered ones, because it's easier. Choose one element of the original set to be special. Now we can set up a bijection between even-length and odd-length cyclically ordered partitions as follows: If the special element is on its own, merge it with the part after it. If it's not on its own, split it out into its own part before the part it's in. So if we sum with a sign factor, we get 0.

OK, so what about the linearly ordered case? We'll use a similar bijection, except it won't quite be a bijection this time. Pick an order on your set. Apply the above bijection with the last element as the special element. Except some things don't get matched, namely, partitions that have the last element of your set on its own as the last part.

So in that case, take the second-to-last element, and apply recursively, etc, etc. Ultimately everything gets matched except for the paritition where everything is separate and all the parts are in order. This partition has length equal to the size of the original set. So if the original set was even you get one more even one, and if the original set was odd you get one more odd one. Which rephrased as a sum with a sign factor yields the original statement.

Personally I have to say I like that one more than the Mobius-function based one. :)

Expand full comment

The ACX crowd never ceases to amaze me.

Expand full comment

Well, if your partner ends up using that proof, they should give an acknowledgement to Beren Gunsolus. :)

(Actually I wasn't just browsing through here and found your comment -- someone reposted your comment to an HN thread about math and generative AI and I saw it there, then I came here to respond to it. But yes it is a coincidence that Beren and I were talking about this just a few weeks ago, and then he solved it...)

Expand full comment

Wow, that's... pretty revolutionary? I wonder if GPT-4o has better logical reasoning skills due to its multimodal training...

Expand full comment

Yes, I also found it pretty revolutionary. To be clear, I wouldn't trust such a proof blindly. But I have been told that these two proofs are correct.

I suspect that GPT-4o has some better way of leveraging the abilities of wolframalpha or other expert system.

Expand full comment

It seems likely that the replication crisis and bad/incomplete/misleading science papers will doom any attempt by an LLM to make a compilation of true scientific thought. If we eliminate everything we can't be 100% certain is correct, then the corpus of "known" science drops to what is taught in high school or maybe college. If we are willing to add potentially wrong items, then the LLM will spit back out potentially wrong conclusions. This may sound no worse than a human doing it, but it adds an additional problem that we will not know what the LLM did with the information and therefore can't troubleshoot the answers.

i.e. we can ask the LLM if a particular chemical would have a particular effect on a human dealing with a medical problem. It may say yes, it may say no, or it may give a detailed answer about exactly why. Based on current results, we would not be able to say if the answer is hallucinated, even if very detailed. Until we fix that problem, no one should consider using an LLM for real research. It can help compile information that's commonly known, if a person is there to question the answers. But that doesn't get us anywhere new. Maybe it gets a smart scientist somewhere a bit faster, maybe a lot faster, but no further.

Expand full comment

Alphafold has so far yielded exactly zero practical discoveries, AFAIK. There will be some in the future but so far it’s ~zero.

Expand full comment
Jun 10·edited Jun 10

The folks up on TWiV did an assertion-by-assertion rebuttal of Alina Chan's lab leak opinion piece in the NY Times. If you're serious about understanding why the lab leak hypothesis is full of holes, I would suggest you listen to this. It's slow going and highly technical — but if you don't have the patience or the knowledge to understand something, why should you expect others to respect your opinions on that subject? Of course, the common fallacy of experts and laypeeps alike is we assume we know enough to form an opinion when in reality we're just smart rationalists displaying the Dunning-Kruger effect. (insert shrug a emoji here)

"TWiV 1121: SARS-CoV2 still didn't come from a lab"...

https://www.youtube.com/watch?v=Sz2qFJmpoug

Expand full comment

It's a two hour video; can't they write? *grumble*

Expand full comment

Yes, they do go on and on, don't they? It takes them five to ten minutes to get through their weather reports. Dickson (whose first name escapes me) is one of their frequent contributors. He fell asleep once during their discussions. For some reason, I just can't get enough of TWiV. What does this say about my neurodivergence that I find them exciting?

Expand full comment

I do plan to watch it, as some sort of recompense for bugging you in the last thread. :-) But it might take a few days for me to find the time.

Expand full comment

Totally agree. I am so sick of asking google how to do something in Photoshop and finding only YouTube videos about the subject. Video's are an inferior way to transmit information, even about Photoshotp, where images of the menus and image you're making are often needed. If I can find a written answer I can skim through it and usually find out within 60 secs. that my problem was that I was skipping a step. Go back to Photoshop, try it with the step added, and if it works I'm done. If it doesn't, reading through the whole written answer is STILL faster than watching a video.

Expand full comment

Most of the non-familiar-with-the-detailed-facts people supporting lab leak are doing so on the basis of broad arguments like "if there's no lab leak, what's China trying to cover up?" and "if there was no lab leak, why were Twitter/Biden/virologists trying to suppress the idea?" Not only are these reasonable arguments on their own that provide a decent reason to start with lab leak prior, but they also serve a moral function: choosing to believe, and promote, the lab leak hypothesis after discovering that its discussion was deliberately censored is creating very valuable disincentives for such corrupt behaviour, if enough people do it. Of course that has to be balanced against the value of impartially seeking the truth.

Incidentally, I have a strong instinctive feeling that lab leak is false, and since I know virtually nothing of the facts, I'll take your advice and disregard it.

Expand full comment
Jun 10·edited Jun 10

I'm not advocating that you disregard the facts. But if you (the general you) don't understand the facts, I suggest you educate yourself in the science to understand what the facts tell us. I'm pretty sure that Alina Chan understands the science, so I have trouble understanding her motivation to distort the facts. My conclusion is that she has other axes to grind.

And how was anything censored? I've followed the arguments on Twitter and scientific papers since March of 2020. I don't think I have any privileged access to censored information. We have gaps in our knowledge, yes — but what we know is enough to come to some pretty firm conclusions.

Expand full comment

Chan has carved herself out as THE lab leak theory scientist advocate. This is not to say that scientists should not ever attach themselves to a cause, but I think the extent to which she is now incentivized to maintain her position should be considered.

As for things being censored...I think it's fair to say that in mainstream science-adjacent circles, discussing the lab leak theory historically has generated harsh enough reaction to be considered soft-censorship. I know this isn't technically censorship but its certainly produces similar effects on discourse and fostering of resentment.

Expand full comment

I watched this play out from March 2020 onward. It was the leakers who, when challenged by the facts, started calling people names, and who urged that Daszak, Fauci, and Shi be prosecuted for crimes against humanity. As for being "censored" they were an extremely censorious bunch. But repeat after me: strong disagreement is not censorship. Name-calling isn't censorship either, but when it involves implicit or explicit threats of violence it should be...

“If the facts are against you, argue the law. If the law is against you, argue the facts. If the law and the facts are against you, pound the table and yell like hell!”

― Carl Sandburg

Expand full comment
Jun 10·edited Jun 10

I really cannot see you as an impartial evaluator of the positions if you seriously believe there was no "censorship" on this issue. There were absolutely huge social and professional pressures and a lot of name calling (basically saying anyone who considered lab-leak was a "racist"). Not to mention the active censorship of tech companies.

I think you need to get a bit more in touch with that reality fi you want to convince anyone.

Right now you sound like the person trying to convince JFK conspiracy people that it was for sure Lee Harvey Oswald, and when the conspiracy people point out he was a likely patsy due to his links to communism you respond saying there was no stigma against communists in 1960s US.

It just makes you seem like you don't know WTF you are talking about.

$0.02

Expand full comment
Jun 11·edited Jun 11

OK. Tell me who was censored and which entity censored them?

Ebright is still calling everyone who disagrees with him names. In fact his ad hominem attacks on some of the key zoonosis proponents probably amount libel. Lucky for him he hasn't been sued. And Rutgers hasn't sanctioned him.

Alina Chan received her doctorate while promoting her lab leak hypothesis, and she is still working at Broad Institute at MIT without any career repercussions.

Give me some names of people who have been censored. Please!

Expand full comment

My impression is that Richard Ebright was a more prominent scientist than her, and has also been arguing for lab leak.

Expand full comment
Jun 10·edited Jun 10

Richard Ebright's arguments mostly involve ad hominem attacks against anyone who disagrees with him. He can't really be taken seriously, because he never seriously addresses the data.

Expand full comment

Clothes dryers produce a lot of ozone due to the electrostatic discharge as the clothes rub together. It would be pretty amusing for the Ca legislature to discover they had inadvertently banned clothes dryers.

Expand full comment

"And there was much rejoicing". Dan Carlin's next series is on Alexander the Great.

Expand full comment
Jun 12·edited Jun 12

Coincidentally, ACOUP just had a series on Alexander the Great: https://acoup.blog/2024/05/17/collections-on-the-reign-of-alexander-iii-of-macedon-the-great/

TLDR: Alexander was a legitimate military genius on the battlefield, who consistently pulled off decisions that most people wouldn't have been able to make, although he *also* benefited from the army that his father created and reformed and the personnel that his father chose for him, without which he would have never stood a chance.

However, he was an utter failure in all non-military manners, an unbelievably bad judge of character and too impatient to bother with any administrative work. He basically just wandered around the world killing and conquering and didn't care about anything else, even his own succession. And the one non-military thing he tried to do (fusing the Persian and Greek elite cultures) was a complete failure.

Expand full comment
Jun 10·edited Jun 10

The problem with the Possible Girls paper is that, if there is a pluriverse out there, then for some possible world, for the people in that world it will be the "actual world" and our world will be one of the possible worlds.

So Sinhababu is the Possible Boy for a girl in that world, but since *he* is not an immortal who mystically magically knows every detail of the world that is different from her actual world and is spending eons singing out each and every one of them - then I don't think that there is counterpart Possible Girl for him.

I realise this is just a joke paper, but the amount of fine-tuning to get it down to the absolute "one choice that I want and who wants me" involved means that whether or not there are multiple worlds existing out there, there won't be any trans-world love affairs. Not unless you count "in love with my imaginary girlfriend" as a love affair. I think he's stuck with real-world girls in our actual world, and if he can't get one of them, then "I invented the perfect girl in my imagination" is as good as he'll get out of modal realism.

Expand full comment

But Sinhababu’s counterpart is not a (Real, Possible-To-Us, Actual-To-Herself) mortal girl looking for a (Real, Possible-To-Her) immortal boy; she’s a (R, PTU, ATH) immortal omniscient girl looking for a (R, PTH) mortal schlub boy who meets her very exacting tastes. No, this doesn’t seem very likely, but I think it serves as a fun reductio ad absurdum for modal realism.

Expand full comment
Jun 10·edited Jun 10

I notice this paper pre-dates Roko's Basilisk by about 2 years.

Expand full comment

Can anyone recall one of Scott's monthly links for me?

A year or two ago, I read an article that I think was one of Scott's monthly links about how to apportion credit for an accomplishment among multiple partcipants, working independently and perhaps without knowledge of one another. I haven't been able to google this successfully because I don't even know what the topic is called. Can anyone help?

Expand full comment

Some possible keywords for you: federated learning, mechanism design, incentive mechanism. See e.g. this paper (focused mostly on machine learning?): https://arxiv.org/pdf/2106.15406

Another keyword is the Shapley value. It is the *only* way of fairly splitting credit, if you believe in the relevant axiomatic characterization... i.e. it's proven to be the only mechanisms that has blablabla desirable properties, where those theoretical properties may or may not map to real world things you care about. In any case, you should at least be aware of its existence (and of the fact that it's fucking hard to compute) https://en.wikipedia.org/wiki/Shapley_value

Expand full comment

Thank you! The Wikipedia article on the Shapley value is perfect for my needs. I still can't find the other article, but that's OK, because I don't need it anymore.

Expand full comment

I very much appreciate your prompt reply, but that's not what I am looking for.

Expand full comment

I'm an ignorant American who doesn't understand European politics. Can someone explain why Macron is calling for new National Assembly elections as a result of the strong showing of rightwing parties in the EP? It sounds like the odds are he will end up with less of his party in the Assembly. Is the idea that it's better to have new elections now than later because later elections would likely be even worse for him? What am I missing?

Expand full comment
Jun 11·edited Jun 11

>I'm an ignorant American who doesn't understand European politics

Don't worry, neither do we.

>why Macron is calling for new National Assembly elections

Litteraly nobody knows. Some think it's a way to "innoculate" the country by giving them a taste of 3 years of impotent far-right (imho that'd be dumb, because the voters would recognize attempts to defang them and double down on the presidential election). Another explanation is that it's to ride the reaction to the far-right win: in 3 years of politicking, we'll be back to square 1, with a fragmented opposition to the far right. More likely imho, but still probably a bad idea (no matter what, his party will end up weaker, since the center-left already in a coalition with the mainstream & far left., and the mainstream right is negociating with the far right. The center seems about to be completely isolated).

At this point i'm inclined to non-charitable explanations such as "he got a bad case of hubris overdose" or "he's high as fuck all the time and don't think straight". These got the advantage of fitting his most recent gamble, which is to threaten to quit if he loses the snap elections.

Or maybe he's counting on an election on the 30/06 to include a lot of elderly voter, which is his main demographic target, while the working adults & students are on vacations.

edit: since I wrote this, Glucksman, the current center-left leader, announced that there would be no deal with the far left, and the mainstream right is imploding. We're heading for a replay of 2022, with the PS and the Republicans getting splintered between the center and their left/right. Which is maybe what Macron was hoping for, but now there's a distinct possibility that instead of having a typical 2nd round that is center vs left or center vs right, we'll have left vs right.

This is some interesting times.

Expand full comment

>Don't worry, neither do we.

Yeah, most Europeans know more about American politics than about French politics. "European politics" doesn't really exist. E.g. in my country, the reporting about the EU election was entirely about which party would win the election locally. Luckily, in the end, both parties found some reason to consider themselves the winner and could thus celebrate.

Expand full comment

Thanks to everyone for the great responses!

Expand full comment

Paraphrasing: "If the RN wins the legislative elections, they can't say that the National Assembly isn't representative ; and if they lose this will reinforce Macron. The probability that they win the legislatives is high, so they won't be able to just be the opposition, they will have to actually make decisions, which, a priori, they don't know how to do. Macron still has a veto to scuttle them. The current presidential party can't govern currently because they have to rely on Les Républicains, that are not a real ally. This is also a way of getting back/punishing them. Historically RN has always been an opposition party, it won't be easy to transition to a real party, which could destroy their chances for 2027 because they wouldn't have much to show in the three years they got. Since Macron is already having difficulties gouverning with Les Républicains, and since he'll stay president, it's not that risky".

Expand full comment

The president in France has no veto. If RN manages to get the majority (I don't think I'd give that a high probability, even if they win the election they can still be short of a majority) Macron would have little power in this scenario on the national scene, much less than Bardella.

Expand full comment

Macron likely would have trouble getting next year's budget through. So he's choosing to fight now in the aftermath of a shock victory by Pen. He is also hoping that his less motivated, more moderate voters will turn out for a general election in a way they didn't for EU Parliamentary elections. And it's true that turn out for such elections is about 50% higher than for EU elections. Most of that represents moderate, less engaged voters who are really his bread and butter. He's counting that the far right has hit it's limit and most of the disengaged middle is more moderate (which historically and by opinion polls is correct).

Macron is also, to some extent, trying to get the center to hold. France has a run off system where the top two candidates compete after everyone runs. And he's repeatedly benefitted from forcing the left to choose between him (a moderate right winger) and Pen (a far right winger). The moderate left has repeatedly chosen him. This election might be a replay of that dynamic. And if it isn't then it's likely a moderate left winger will come in second and the moderate right can rally behind them instead. Especially because the moderate left has been having victories against the far left.

Basically, he's trying to preserve his policies more than his party. Which means being pro-EU, pro-Ukraine, technocratic, etc. He agrees with the moderate (but not extreme) left on most of these points and sees them as more important than differences over things like welfare or taxes. Which is why he can get moderate left votes but also why he might be more comfortable with the moderate left than Pen. His nightmare scenario would be Pen vs someone like Melenchon. Two populist Russia-friendly, anti-American euroskeptics but from the right and left respectively.

Expand full comment

Just nitpicking on the form (I largely agree with the substance): it's Le Pen, not Pen. The "Le" is not a droppable aristocratic particle. For that matter Le Pen is saving herself for the presidential election; Jordan Bardella was leading her party in the European election and will also in the National Assembly election.

Expand full comment
Jun 10·edited Jun 10

I think he assumed that it would be hard to carry out with his politics for the remaining 3 years, given the fact that his government is already only supported by a minority of the assembly, and that now on top of that the Rassemblement National (RN, Le Pen's party) made more than twice the score of his party in the european elections. He was at risk of his government being ovethrown by the parliament any time, given that the traditional french right party (LR, used to be dominant but now on the decline) could be finally willing to vote in favor of a motion of no confidence given the terrible score of presidential party. In the last two years, it was the fact that LR wouldn't vote for any no-confidence motions which allowed the government to survive until now. So this way Macron keeps the initiative.

The french legislative elections is somehwat similar to the elections for the American senate, with France being divided into ~570 electoral districts which elect 1 deputy each. But it's a two-round system, with only the candidates doing more than 12.5% of the registered voters (not of the amount of vote, if there's 50% abstention they need to do at least 25%) going to the second round in each electoral district. Typically only 2-3 candidates reach the second round in each electoral district, it is therefore very important for parties with compatible ideas to ally themselves, in comparison to a proportional election. This heavily favor centrist parties like Macron's, as in 2017 you could typically have 1 member of the party of Macron vs 1 member of RN or LR in the 2nd round, and as the left voters would vote against RN's candidate, Macron's candidate would get elected.

In 2022 all the parties from the radical left to the center left managed to form a big alliance which managed to share the lead with Macron's party in terms of total numbers of votes in the first round. This created many situations where in the 2nd round Macron's candidate would face both RN's candidate and a left candidate (often also LR's candidate), which reduced this 'centrism' bonus that Macron's coalition benefited from in 2022, leading his coalition to lose the majority in the parliament (despite being the coalition that got the most deputies elected), and allowed RN to make a big breakthrough in terms of deputies (~80, while it never managed to have 10 in the past with the same electoral system).

Since the last two years, the alliance between the left somewhat collapsed, and the proportional system in the European elections allowing each of them to participarte concurrently without penalizing them. The center left (PS) beat the radical left (LFI), while the later was the dominant force in the left since 2017. It is extremely unsure that this two parties will manage to once again form an alliance in the coming legislative elections. If PS and LFI don't manage to agree on an alliance for the elections, the ecologist party (likely) and the communist party (less likely) will probably join forces the former. But in any case the left candidates will have fewer chances to reach the second rounds in each electoral district compared to 2022. Therefore, I believe that it is part of Macron's strategy to profit of the divisions in the left to defeat the RN in duels in the 2nd round of the elections, profiting of this 'centrist bonus' . As Deiseach mentioned, capitalizing on a reaction against the strong far-right score would also play along those lines.

The issue is that RN has now a lot of momentum given their major win in the European elections, the left hates Macron, and LR electors are splitted on whether they would support Macron or RN candidate in such a duel. Moreover, by calling for new elections, Macron completely validated the simplistic rhetoric of the RN that the European elections were a referendum against Macron's politics which should lead to consequences in the national scene if Macron's party get defeated. So it is far from given that duels between Macron's coalition and RN candidates will turn out positively for the former.

Edit: Apparently the left parties already managed to find an agreement altogether, which is somewhat surprising/impressive given all the mess going on between them lately.

Now the question is what kind of alliances will their be on the right-side of the political spectrum. The second french far-right party called Reconquete (litterally Reconquista, you feel the vibe) was lead by Marion Marechal (- Le Pen, but she dropped this part of her name), the niece of Marine Le Pen during the european elections and did 5.5%. She is trying to work on an alliance with RN, but the head of her party (far-right editorialist, very anti-immigration/islam but more liberal economically than RN) is against that. Both RN and Macron's party tries to form some alliance with LR, but they might as well go on their own. Since 2014 LR is the party that collapsed the most in the french political scene as their voters got sucked into Macron's party and RN.

Expand full comment
Jun 10·edited Jun 10

Macron is in his second term and will not be president after 2027, when the next National Assembly election was scheduled. So the comparision that matters to him must be between the results of the snap election and the Assembly that he could have kept until 2027, not the one which would have been elected in 2027.

His side currently has just a plurality of the Assembly, not a majority. So there is a chance (although not very likely) that he gets a better deal with this election. Like others have said, he might count on not letting the far-right in charge remaining the priority for a majority of voters. Also, by dissolving now he can frame it as hearing the people's protest, while earlier it could have come accross as not accepting the previous National Assembly election's result. This is a risky but not absurd strategy.

Expand full comment

A lot of people have that question....the Economist posted an article a few hours ago.

Headline: "Why France’s president called a snap election"

Subhead: "The centre wants to weaken Marine Le Pen’s hard right, in or out of power"

Article:

Emmanuel macron is nothing if not a risk-taker. At the age of 38, an electoral debutant, he launched a new centrist party and went on to win the French presidency in 2017, barely a year later. Now Mr Macron has taken a fresh political gamble that puts his credibility and authority on the line for the three years that remain of his second term in office. His unexpected decision, announced on June 9th, to dissolve the National Assembly and hold snap elections at a two-round vote on June 30th and July 7th, has stunned even his own deputies, and left all parties scrambling to book venues, draw up a manifesto and plan their campaigns.

Mr Macron’s decision was a response to crushing results for his party, Renaissance, at elections to the European Parliament on June 9th. The party garnered just half of the share of the vote secured by Marine Le Pen’s hard-right party, National Rally (RN). Her triumphant candidate, Jordan Bardella, scored over 31%, a party record at European elections. That evening, in a televised address, Mr Macron called his decision “grave, heavy”, but argued that he could not “carry on as if nothing had happened”. The fresh vote, the president declared, was consistent with the principle that “the word should be given to the sovereign people.”

The president’s calculation seems to be that, at some point, he was likely to face an irresistible political demand for fresh parliamentary elections. He presides over a minority government, which has struggled at times to pass legislation and regularly had to resort to the use of a constitutional provision that allows bills to go through without a direct vote. Each time this exposes the government to a no-confidence motion. Since Mr Macron was re-elected in 2022 his governments have survived 28 such votes. Another was likely—although not constitutionally inevitable—to greet the next budget, which had been due to go to parliament in September. By dissolving parliament now, Mr Macron has at least made the choice his, and controlled the timing.

More than this, the French president is hoping for what an adviser calls a “moment of clarification”. Either the popular support for the RN is real, goes this argument, and in that case his party hopes to put its populist policies—on tax, immigration, energy—under proper scrutiny and to expose their contradictions. Or, the vote represents what the French call a mid-term ras-le-bol, or fed-upness, which would not survive at its current level when the stakes are about the daily government of France.

To this end, Renaissance is seeking to force other moderate parties to support a “republican front” against the RN. Stéphane Séjourné, the foreign minister and head of Renaissance, says that the party will not put up candidates against rival contenders it judges “republican”: that is, who do not belong to the political extremes. In a two-round election, this would reduce the chances of a split anti-RN vote in the second-round run-off. Parties have until June 16th to register their candidates, and there will be much fractious manoeuvring up to that deadline. The point, ultimately, for Mr Macron is to try to broaden parliamentary representation among those parties that might be willing to contemplate coalition talks with Renaissance. If the political left is split up along the way, so much the better.

It is, however, far from clear that coalition talks, which have got nowhere in the past, would be any more successful this time round. Parliamentary leaders of both the Socialist Party and the centre-right Republicans said this week that they were unconvinced. Renaissance itself is, if anything, likely to lose seats. Some sitting deputies, such as Jean-Louis Bourlanges, the respected head of the foreign-affairs committee, have said that they will not stand again. Short of winning a majority, Mr Macron may once again face a hung parliament. At best, this could bring forth a German-style coalition; at worst, it would produce more parliamentary deadlock, and potentially more elections.

Moreover, it is possible that, on the contrary, Ms Le Pen’s RN makes big gains and puts itself in a position to negotiate a coalition, possibly under Mr Bardella. In a two-round election, it will be harder for her to win seats than in the single-round European election, which was based on proportional representation. Yet such is the fluidity of party allegiances that it is not inconceivable that she might find right-wing Republican nationalists to work with. Such a prospect prompts worry on the markets. By midday on June 10th the Paris CAC 40 had lost 2% of its value, and the share prices of France’s largest banks dropped by up to 9%.

Back in 1997, Jacques Chirac, then president, dissolved parliament in the hope of shoring up his majority, which had been weakened by widespread strikes and protests two years previously. Instead, the opposition Socialists swept into power, and Mr Chirac was forced into an uncomfortable “cohabitation” for five years. This may be why even Gabriel Attal, Mr Macron’s 35-year-old prime minister, tried to talk him out of calling the election. Another party figure calls the decision “crazy”.

Mr Macron certainly knows his history. But he is also often convinced that he can defy precedent, and pull off things that others cannot. The French president may even be gambling that the demands of government might expose to voters the incompetence of the RN, and undermine its appeal ahead of the next presidential election, in 2027. That may be wishful thinking. Either way, Mr Macron’s legacy, as well as his credibility, is now on the line.

Expand full comment

I wondered this about the UK election, too.

One speculation is that he foresees upcoming hard decisions which will be unpopular, so he might as well dump them on the incoming government. But governments seem very capable of passing the buck.

Expand full comment

AFAICT, the expected results of the UK election were trending rapidly worse for the conservatives as time passed, and so calling the election sooner meant a smaller loss.

Expand full comment

It raises the question what is so different about the UK that we are set to buck the rightward trend in the US and EU.

Expand full comment

I was going to say "it's a cycle, not a trend" but actually I think it's a cycle superimposed on a trend.

Right now what matters in the UK is the cycle -- the Tories have been in power for fourteen years, everyone is sick of their fuck-ups, and they want them out. This doesn't mean people are embracing leftism, it just means that they want someone who isn't the Tories. Now they will (presumably) have a Labour government who will acquire fuck-ups of their own until they get kicked out.

Then there's a general rightward trend driven by increasing rejection of mass immigration. This mostly doesn't help the main centre-right parties though since they have always supported mass and/or illegal immigration just as much as the centre-left. (It's not really possible to imagine Rishi Sunak at this late stage declaring "Britain for the Britons" and banning all immigration.) This trend will play a role in the UK election too in the form of a big swing towards the Reform Party, but the nature of the FPTP system means they're just going to fuck things up further for the Tories rather than win a lot of seats.

Expand full comment

The European Parliament elections are interesting because people largely vote following their national cycles and the cycles in the various European countries are disconnected. Aggregated at the Union level, they cancel out and only the common trend is visible: a slow rightward drift with increased fragmentation.

Expand full comment

In general, most places are suffering from post-Covid problems that are putting incumbent parties at an electoral disadvantage. Both economic issues and erosion of trust in institutions. Britain's got it worse than most because Covid and Brexit overlapped. Since the Conservatives are the incumbents, they're the ones suffering.

As an American casual observer of UK politics, I also get the impression that the Conservatives tried and failed to accommodate right-wing populism within their coalition. They accommodated them enough to sour moderate swing voters on the Conservative party, but not enough to avoid losing lots of votes to right-wing populist third party candidates. And since the UK uses single-member plurality districts, splitting the vote between Conservative and Reform candidates means both are very likely to lose.

Expand full comment

That makes sense: an anti-incumbent trend rather than a directional trend.

Expand full comment
Jun 10·edited Jun 10

>go now and try and capitalise on "scary far-right coming for you!" to get voters out.

Sounds correct to me. There is a precedent for early elections working in Germany: The social democrats were on the decline years ago and dissolved the parliament under the assumption that they'll lose catastrophically if they governed until the planned election. They got as a minor partner into the next government, with a relevant number of ministers.

IMO it's also the right thing to do if you're losing public support and think you can't regain it, so what's right and what's actually done align nicely here.

Fingers crossed that the national turnout will be different from the EU one, though. I don't think a nationalist government in the second-largest nation in the EU would be good.

Expand full comment

I don't get it either. I think you're right, that with the rightwing parties doing so well in the European elections he must consider that waiting for a later election will mean worse results for his party, so go now and try and capitalise on "scary far-right coming for you!" to get voters out.

Expand full comment

Scott Greer wrote in a recent post:

"There's this cartoonish thinking that people can get beyond themselves and think completely objectively if they're asked to. A lot of media convey this myth to the public. In many ways, our system relies on belief in this myth to function. We need people to believe that figures entrusted with important roles—whether a judge, juror, or some other kind of authority—act impartially. Time and time again, we see that not happen. It shouldn't surprise us. Fair play and respect for the rule of law were bequeathed to us by the founding stock. As they disappear, so do those values."

https://www.highly-respected.com/p/trial-by-jury-is-now-a-joke

He then lists recent examples of jury system failure:

-A black man who viciously attacked a white female cop was found not guilty of the most serious charges by a California jury in 2023.

-A Delaware jury couldn't reach a guilty verdict in 2023 against a black man who robbed and assaulted an Asian jewelry store clerk. This was despite him being caught on camera committing the crime and being arrested with the stolen loot.

-A California jury deadlocked in 2023 over convicting two black thieves of the murder of an elderly white photographer. The murder was caught on film and the defendants admitted to committing the heinous act. But defense attorneys claimed that one perp could not be found responsible due to her suffering from a "sickle cell crisis," drug withdrawals, and a low IQ (apparently the only time IQ is relevant in modern America is when it is an excuse for black crime). This argument apparently persuaded some of the jurors.

-A black man in Iowa was found not guilty of murder  in 2022 for his role in the death of his white girlfriend. He severely beat the victim and the woman fell from her apartment balcony during the assault. She died from the fall. The jury found the black man not responsible for the death—only for the assault.

-A black man in South Carolina was found not guilty in 2021 after shooting and killing a white retired volunteer fire chief as the victim sat in his car. The shooter claimed the victim startled him and that was a sufficient explanation to find him innocent of murder.

Did you notice that EVERY example he gave concerned crimes commited by blacks? The problem is allegedly the decline of the founding stock, but there are no complaints about Hispanics or Asians, (an Asian is only mentioned as a victim) let alone non-founding ethnic whites like Poles, Italians, or Greeks. This is one of those things that once you see, you can't unsee. Immigration restrictionists will complain about problems caused by the demographic decline of whites, the founding stock, "heritage Americans," whatever, but when it comes time to provide examples, most or all will involve blacks, whose presence in the country long predates the 1965 immigration act.

Expand full comment

When Trump was President, his supporters talked incessantly about the “deep state” to excuse Trump’s lack of accomplishment. Some of them actually believed this stuff; one insisted that the deep state wouldn’t permit Biden to withdraw from Afghanistan.

Now that Trump has been convicted by a jury, Scott Greer is attacking trial by jury. His prime example is, of course, the Manhattan trial. Greer claims that that case, “was considered a joke by most legal experts, even liberal ones,” but the embedded link doesn’t support this claim.

Greer says that we need to reform the jury system: “The first step is accepting that jurors cannot be objective simply because they say so.” But the system already accepts that. Greer links to a Washington Post article about jury selection in the Trump trial which gives examples of the judge dismissing jurors for bias even after the jurors had said they believed they could put aside their preconceptions and judge the case on its merits. So Greer is lying, again.

The really priceless part is where he claims that liberal jurors lied about whether they could be impartial. “The only jurors who were honest were conservatives, who all said they couldn’t be impartial, and were thus summarily dismissed.” Greer is an example of a conservative willing to lie at the drop of a hat.

Expand full comment

"Greer claims that that case, “was considered a joke by most legal experts, even liberal ones,” but the embedded link doesn’t support this claim."

Here's an excerpt from the embedded link:

"Skeptics across the political spectrum say the felony portion of the case is built on shaky and unproven legal reasoning that will require ironclad evidence to prove — evidence Bragg may not have. There are also major technical issues that could derail the indictment, most notably the untested matter of whether a federal crime such as a campaign finance violation can count as a secondary crime under New York’s state-level business records law.

But others argue that the case isn’t nearly as weak as skeptics make it out to be. They say Bragg and his team are adept at litigating complex financial issues such as this one. The lack of details about how Bragg plans to connect critical dots in the case, they add, is a sign that the district attorney is merely saving his most potent ammunition for later, not that he doesn’t have it."

I don't interpret that as Greer "lying."

"Greer says that we need to reform the jury system: “The first step is accepting that jurors cannot be objective simply because they say so.” But the system already accepts that. Greer links to a Washington Post article about jury selection in the Trump trial which gives examples of the judge dismissing jurors for bias even after the jurors had said they believed they could put aside their preconceptions and judge the case on its merits. So Greer is lying, again."

He never claimed that judges don't do that. You're just putting words in his mouth and accusing him of lying.

Expand full comment

I'm not sure I understand what your point is

Expand full comment

"Immigration restrictionists will complain about problems caused by the demographic decline of whites, the founding stock, "heritage Americans," whatever, but when it comes time to provide examples, most or all will involve blacks, whose presence in the country long predates the 1965 immigration act."

Not sure what was unclear about that.

Expand full comment

I don't think you're being as clear in your communication as you think you are. Why don't you spell it out for me using different words?

Expand full comment

Remember that time armed insurgents took over a federal building in Oregon and then everyone who took it to trial was acquitted? https://www.govexec.com/management/2022/07/time-armed-militants-occupied-federal-building-not-capitol/375036/

Expand full comment

Remember that time armed insurgents took over several blocks of downtown Seattle, committed innumerable crimes and murdered two unarmed black kids, and there were fewer prosecutions than for the Bundy thing? https://en.wikipedia.org/wiki/Capitol_Hill_Occupied_Protest

Expand full comment

Remember that time like 3 weeks ago a bunch of pro-Palestine protestors argued they had a first amendment right to occupy public property?

Expand full comment

Did that argument work for them?

Expand full comment

I mean, it does seem to work out pretty well for them overall. Even in the unlikely even that arrests are made and charges filed, the charges are almost always quietly dropped as the city government is on the side of the protesters, and there's a solid chance that in a year or so they'll get a generous payout from some jury for "police brutality."

Expand full comment
Jun 10·edited Jun 10

Background for the cases (max 10 min research per case):

- Female officer: Suspect is schizophrenic, and it was argued that he was unlawfully detained.

- Asian clerk: Hung jury, will be retried.

- Photographer: Guilty on other charges, jury hung only on murder, will be retried.

- Balcony: It's unclear if the fall happened "during" the assault. The prosecution argued she couldn't have walked to the balcony.

- Old man: The prosecutor didn't include the charges of manslaughter or involuntary manslaughter, so the jury couldn't convict the suspect of that and let him go.

All in all: The system definitely has issues, but so does Scott Greer.

Expand full comment
author

Thank you.

Expand full comment

"- Asian clerk: Hung jury, will be retried.

- Photographer: Guilty on other charges, jury hung only on murder, will be retried."

Greer stated that the juries deadlocked. This seems like you were motivated to issue "corrections" regardless of whether you could find anything wrong with his claims.

Expand full comment
Jun 10·edited Jun 10

As a non-American, "deadlocked = hung jury = suspect will be retried" was not obvious to me, so I wanted to point it out.

I actually felt that his descriptions of the last two were the most accurate, but I still gave a summary, since I did the research. I intentionally didn't call it "corrections".

Tangent: Juries getting hung in 40% of the cases would indeed be bothersome for the system, but since Scott Greer cherry-picked, I'm not too worried.

The actual ratio seems to be around 6% and 90% of cases don't even go to trial.

Expand full comment

>As a non-American, "deadlocked = hung jury = suspect will be retried" was not obvious to me, so I wanted to point it out.

Yeah, me neither.

Expand full comment

>>This seems like you were motivated to issue "corrections" regardless of whether you could find anything wrong with his claims.

Do you acknowledge that Milli did, in fact, find "something wrong" with at least 60% (3/5) of the claims in question?

He did say "*The system definitely has issues*, but so does Scott Greer," after all.

Expand full comment

I don't acknowledge that. Greer was giving one-paragraph summaries, so you can always find some aspect of the case he didn't mention and say "well what about this?" Would the extra info change how you view the case? Is leaving it out inherently dishonest? Consider:

- Female officer. If the defense was based on insanity and he was found not guilty by reason of insanity, it would certainly be dishonest not to mention the suspect's schizophrenia. But it wasn't. As to the unlawful detention claim, it's pretty laughable, see: https://www.foxnews.com/us/california-man-who-pummeled-shot-at-female-deputy-found-not-guilty-despite-video-of-attack

- Balcony. If you beat someone up and they, dazed and disoriented, accidentally fall off a balcony and die, that's still murder by the felony murder rule. To find him guilty of assault but not guilty of murder requires one to believe (or think there's a reasonable probability) that this woman just so happened to fall off the balcony right after being beaten up, with the fall having nothing to do with the beating.

-Old man. Was pretty clearly guilty of murder, not manslaughter.

Expand full comment

My impression is that the justice system is noisy enough that you can always cherry-pick a bunch of cases where some injustice was done in a particular way you want to claim is widespread. And that this is even more true when you add in the ability to choose the facts of the cases you report to make your story better.

Expand full comment

"My impression is that the justice system is noisy enough that you can always cherry-pick a bunch of cases"

Would this objection also apply to those who bring up the various BLM martyrs?

"And that this is even more true when you add in the ability to choose the facts of the cases you report to make your story better."

This sounds like a fully general counterargument.

Expand full comment

Thanks for saving us the lookup. Sounds like another example of the "upon a fuller review of the facts than I had, and a more careful application of the law to them than I did, the jury reached a different result than I expected - how did they get it so wrong?" genre of legal reporting.

Expand full comment
Jun 10·edited Jun 10

Color me unimpressed by claims that a jury has acted irrationally, when the claimant does not even once tell us what the jury instructions were in the cases at hand. Let's take the 2023 CA case as an example.

First, there were two defendants. If only one fired the fatal shot, then the other is guilty of murder only if 1) he intended to kill; or 2) was a) a major participant in the crime; AND b) acted with reckless indifference to human life. https://www.justia.com/criminal/docs/calcrim/500/540b/ Merely participating in an armed robbery is not enough to show reckless indifference to human life. People v. Clark (2016) 63 Cal.4th 522, 613 ["The mere fact of a defendant's awareness that a gun will be used in the felony is not sufficient to establish reckless indifference to human life."]. See the long list of factors in the link, which are not exclusive. We have no idea which, if any, of those factors existed in the case. Moreover, things like intoxication and mental or emotional issues are definitely relevant to whether a defendant formed the requisite mental state of reckless indifference. See, eg, https://www.justia.com/criminal/docs/calcrim/3400/3426/

Note also that, as far as we know, the jury deadlocked because they could not decide which one of the defendants fired the shot. (And, as another thing to note, we don't even know whether there was one jury or two, because it is not uncommon in CA to empanel two juries, one for each defendant, in a case like this).

None of this is to say that the law is correct. But to infer that the jury acted irrationally from the mere fact that they deadlocked is not a valid inference.

The same is true of the other cases; unless we know 1) the facts given to the jury; and 2) the instructions given to the jury, we can't know whether the jury acted rationally.

Expand full comment

Tactically speaking, I think he could have made an equally strong case without bringing race into it at all, and that would have helped him reach a larger audience.

Expand full comment

Agree

Expand full comment

How cherry-picked are those examples? Did he really only find cases where it was black defendants and never any white ones?

As for the cases, in two at least I see the reasoning: if the accused was really of such low IQ that they could not be held responsible, then they can't be found guilty of the particular offence of murder. If the man beating his girlfriend did not deliberately throw her over the balcony, that's not murder either. Falling from the balcony was not the aim of the assault.

If we want "white people getting away with murder", look at the Martens-Corbett case. Woman and her dad successfully appealed murder conviction, got a plea bargain for manslaughter and after four years have now been released:

https://www.irishtimes.com/crime-law/2024/06/06/i-admire-the-corbetts-for-their-strength-says-prosecutor-who-agreed-plea-deal-with-molly-and-thomas-martens/

Expand full comment
Jun 10·edited Jun 11

Scott Greer does his best of "not outright lying" while presenting the suspects in the most unfavorable way. He also links to himself as a source, here's my "neutral" research: https://www.astralcodexten.com/p/open-thread-333/comment/58691216

Expand full comment

Iowa has a felony murder statute, so that deaths (even if accidental) that occur during the commission of a felony are classed as first-degree murder. IANAL, and felony-murder statutes often include a bewildering array of provisos and exceptions that might have applied in this case, but there have been many people convicted of first-degree murder that fit the fact pattern described.

Expand full comment

"Did you notice that EVERY example he gave concerned crimes committed by blacks? The problem is allegedly the decline of the founding stock, but there are no complaints about Hispanics or Asians, (an Asian is only mentioned as a victim) let alone non-founding ethnic whites like Poles, Italians, or Greeks."

Is your thesis that "immigration restrictionists" are insufficiently concerned about, like, MS-13? That there's insufficient panic about lax immigration laws potentially allowing violent criminals to come into the US?

I mean, I go scan the article and, right before he lists all these events, he states:

"Juries increasingly have a hard time convicting black criminals charged with crimes against non-blacks. The most famous example of this happened in 1995 when a majority-black jury exonerated the clearly guilty O.J. Simpson for the murder of his white ex-wife and her friend. Jurors openly admitted they believed he was innocent simply because he was black and they thought the racist LAPD was out to get him. This trend has gotten far worse since the George Floyd Revolution. Here are a few recent examples:"

Like, if you disagree, cool but...no, I'm not surprised that every example he provides is about African-Americans right after a paragraph where he says he's going to do that and his general thesis is that jury trials are becoming less trustworthy as juries, judges, and prosecutors become more politically polarized.

I don't want to say this is just a "Boo outgroup" post but...I'm struggling to understand the point you were trying to make.

Expand full comment

What data would we need to untangle whether there is a serious problem with black jurors being unwilling to convict black defendants of crimes committed against whites? This seems like something you could actually do good research on and learn something about the world.

Expand full comment

Because this isn't the actual argument of Scott Greer, the argument isn't actually that black juries don't convict black defendants. You can tell, because the article is primarily about Donald Trump and his recent conviction, because of course it is.

Scott Greer's primary argument, which blazes out of the essay if you read to understand, is not racial but political; leftists are untrustworthy on juries and conservatives are just too gosh darn honest, the way our sainted forefathers were. The focus on black crime, incidentally, is primarily in response to BLM, because of course it is.

The article is, bluntly, not good but you will notice, if you for example read the first 7 paragraphs, that it is not primarily about race, nor is the racial composition of juries even brought up, as Turok implies, because the stupid argument Greer is making is that evil white liberal juries are letting black murderers walk free because it lets him attack his liberal outgroup for Trump and BLM. No, there was no jury of twelve black men in Iowa for that case, it was Iowa, it's like 5% African American, it was almost certainly an overwhelmingly white jury Greer is attacking, because they're gosh durn evil white liberals.

Here's the ChatGPT summary, about as neutral a summary as I can provide:

"The essay argues that the recent conviction of Donald Trump in the hush money case undermines the validity of the American jury system. The author contends that the original vision of trial by jury, cherished by the Founders, relied on a homogeneous, virtuous society that no longer exists. They claim that today's diverse, urban, and politically polarized population makes fair trials impossible, as demonstrated by the Trump case and others involving racial biases. The author suggests reforming the jury system, possibly favoring trial by judge, as current juries are seen as incapable of delivering impartial justice. The essay concludes that the myth of an unbiased jury needs to be dispelled."

Do you really read a plot by African American juries to release criminals from that? Or do you think Scott Greer is just attacking liberals for clicks.

This is misleading ragebait on top of stupid ragebait and twice shame on Alexander Turok for posting this: first for inducing me to sit and understand a stupid article and second for his own falsehoods.

Expand full comment

>Jurors openly admitted they believed he was innocent simply because he was black and they thought the racist LAPD was out to get him

This alone shows how dishonest this guy is. Even leaving aside the glove not fitting thing, which clearly raises reasonable doubt issues, there was substantial evidence of the lead detective using racial epithets, and the defense claimed that he might have planted evidence (including the glove). I personally don't find that very convincing, but to claim that the jury simply believed he was innocent because he is black is disingenuous.

Expand full comment

He explicitly connected the problem to demographic change:

"Fair play and respect for the rule of law were bequeathed to us by the founding stock. As they disappear, so do those values"

If demographic change was really the issue, you'd expect him to be able to provide examples of obviously guilty Hispanics or Asians getting acquitted by juries of their co-racialists. He failed to do that, because such examples don't exist.

Expand full comment

I'm reposting this post again: https://thothhermes.substack.com/p/the-corrigibility-folk-theorem

I've re-written parts of it extensively and added things to it, as well as changed the background color.

I analyze MIRI's framing of the corrigibility problem; I argue that it doesn't really require terms added to the utility function, or uncertainty about the utility function. I also argue that it doesn't require solving any deep problems before it becomes possible at all. I expect that it should be a relatively straightforward consequence of trade / negotiation between agents with overlapping environments that understand how to calculate the probability of outcomes after performing updates to their utility functions.

I'm hoping that this spurs discussion.

Expand full comment

What's the corrigibility problem in terms a laypeep can understand?

Expand full comment

I think of it as the problem of one agent trying to imbue its values on another agent. "Corrigible" agents are typically defined as being cooperative with the process of another agent trying to correct or change its preferences.

Expand full comment

I'd love to hear a review of Manifest and Less.Online — not so much with respect to any specific programming but rather the social scene and general vibe.

Expand full comment
Jun 12·edited Jun 12

I didn't go this year, but I went to Manifest 2023. I did get to meet a lot of cool people and it was fun, but I also felt like a fish out of water because there was a very strong bias towards prediction markets and AI doom among the attendees.

I guess it's bit like what you would get if you took "the ACX demographic" and then filtered to people who are obsessed with prediction markets.

Expand full comment

I have donated my sperm. Among other influences, this blog nudged me into this. Therefore, some future people will in some way owe their being what they are to this community, and I think I should tell you how.

1. Scott donating his kidney helped me normalize donating body parts. By comparision, sperm feels very light. I already donated blood before that though.

2. I think I have absorbed some of the eugenistic ideas which are relatively frequent among commenters here: namely that intelligence is largely driven by genes, that average intelligence in the population is important, that smart people have less children than average, and that therefore there is a dangreous risk of decline in collective intelligence. (Of course, like most people here, I am confident in being very very smart). I am not sure I fully agree with the eugenistic view and I would probably not defend it in polite society IRL, but it did weigh on me on some level. So did the "Darwinian" view that an indivual's success is in transmitting their genes no matter how.

The most exciting part of the process was probably writing a short text for the file that the child(ren) will access at their majority.

Expand full comment

I would not donate sperm myself, because I believe I have a moral duty to all my children and if I donate sperm then I would not be able to carry out that duty for all my children. I would not be able to ensure that they had good lives and were raised up to be good people. Emotionally, I feel sadness at the thought of having children who I will never hold in my arms.

Expand full comment

I have certain hang-ups about sperm donation, most notably how it encourages fatherless children, and how it promotes a creepy "Shopping" mentality among the donation-seeking women that - if the internet is any indication - almost always devolves to looking for a superficial set of cliche characteristics in the donor, a box-checking exercise rather than any real involved thought given to the issue.

But regardless of this, I don't actually think sperm donation should be viewed as "Organ Donation", on the same level as - say - Kidney or Blood donations.

(1) Sperm is trivially regenerated, all but one of the organs in other types of organ donations aren't regenerated. The only exception to this is blood, and it's still an extremely scarce "organ" such that losing it fast enough is a common cause of death. In a very real sense, you lost a part of yourself when you donate an organ, I don't think this describes ejaculation accurately.

(2) All non-sperm organ donations require special medical equipment and varying amounts of hassle to perform, again the only possible exception is blood, and even then it has asterisks all around the exception: Cooling blood, extracting and storing it cleanly, making sure the donor doesn't have any of the blood-transmitted diseases, categorizing it according to its type, ..... Again, sperm has nearly none of this, perhaps only the bit about checking for diseases apply.

(3) Non-sperm organ donations sustain existing life, but sperm donations create new life. This is mutually exclusive: Sperm donations can't sustain existing life, and non-sperm organ donations can't create new life (Unless you count cloning). Creating new life is comparatively very easy compared to sustaining existing life. Furthermore, non-sperm donations sustain existing life completely on their own, once you donate a kidney, any person that needs a kidney to live will live thanks to your donated kidney. But sperm doesn't create life on its own, in fact, it's the comparatively trivial part of making a new life, the equivalent of a building permit for a skyscraper: the skyscraper won't exist without it, but it does very little compared to what comes after it. The combination of those 2 things - that non-sperm organ donations do a much more important/difficult job than sperm-donation and that they do it on their own - makes non-sperm organ donations a meaningfully different category than sperm donations.

For those reasons I think calling sperm donations an "Organ Donations" is a categorical obfuscation. There **might** be a moral ulterior motive underlying my opposition, namely that I think sperm donations are bad but other organ donations are good, but I also think that reasons 1...3 stand on their own as a self-contained value-neutral opposition to including sperm donations among other types of organ donations.

Expand full comment

Women who will use donor sperm (lesbians, asexuals, ugly) will not likely settle down with a husband if your sperm is unavailable from a sperm bank. They will instead go with lower quality riskier donation. Your only choice is whether you want more people like you to exist, or more people like a burger flipper with a breeding fetish who lied about having a degree and consents to artificial insemination for the first child but bullies the lesbian couple into natural insemination for siblings.

Expand full comment

If the burger flipper is good at lying, what difference does it make how many other options are available? Most donation-seeking people don't meet their donors anyway, they just trust whatever metadata the donation clinic provides. Get good at lying to the donation clinic and you can deceive thousands at scale.

> They will instead go with lower quality riskier donation

What I'm saying is this: Humans are not commodities. As much as this sounds cruel or uncaring, maybe Lesbians, Asexuals, and Ugly, henceforth LAU, women should instead adopt already-existing children. I'm rooting for - and I'm saying this with utmost sincerity - the day when Cloning and Bio/Genetic engineering is at the stage where LAU women can have children on their own truly with no help from men, but until then they shouldn't be allowed to treat men like they're sperm vending machines and raise fatherless children who will grow up deprived from some of the most primal and fundamental experiences that Evolution programmed humans to want. I'm not sure why U women couldn't find a similarly U men to have children with either.

One of the most repulsive and morally ugly things to me is when humans (or, more generally, sentient beings with feelings) are compared and contrasted like commodities, like inanimate things. One example of this is the job market, where you're reduced to a bunch of one-dimensional questions and CV points and your entire worth - as well as your ability to feed and cloth yourself - is at the whim of a bunch of people with 30/60/90 minutes to spare for the decision (if that). Another example, substantially crueler, more useless, and more absurd than the job market, is the online dating market.

The sperm-donation market is attempting to create a similar human-trafficking-ish trade where people are categorized and selected like pets, where there are buzzwordy "hip traits" that characterizes the most prized "specimen", just like how horse owners (or, for that matter, slave owners) select their stock. To show that my revulsion to this is not entirely driven by the fact that I'm a man and I think this is a demeaning way to treat men: if there was a market where women would donate eggs and male buyers would categorize and select women according to their whims, I would equally want to empty my stomach in disgust at this. This is, quite simply, how living sentient beings should NOT be treated.

(To the best of my knowledge this market doesn't exist, yet. What exists is the "Surrogate Mother" market for women to carry in their womb an already-fertilized egg to term, which is still not completely innocuous but is a far cry from selecting eggs or selecting sperm like it's coming from horses.)

-------------

As the LessWronger Roko - echoing Ted Kaczynski - once [1] said: Modern society is a sort of weird factory farming of humans. Industrial and post-Industrial societies increasingly crush age-old evolutionary machinery in the name of efficiency and convenience. It's not very convenient to live with your extended family in cities, so why don't you live in atomized nuclear family units, Human Production Unit #4336969696? It's not very convenient to give you a reduced working week and/or allow you to take your kids to work, so why don't you outsource nearly 80% or so of raising your kids to a state-controlled establishment or to a bunch of private randos, Human Production Unit #5494232569?

And now the latest dehumanizing trend: It's not very convenient to actually... have a father for your children, so why don't you take this milked sperm from a man you have never saw and will never see in your life except by raw chance, and raise his kids without a father, violating one of the most ancient and basic tenets of human child-rearing? It's... necessary, Human Production Unit #4535224142, it's only necessary.

My very humble and completely unactionable opinion about it is this: Burn It With Fire.

[1] https://x.com/RokoMijic/status/1660447723571539970

Expand full comment

"if there was a market where women would donate eggs and male buyers would categorize and select women according to their whims"

Congratulations, you have just learned about the IVF industry.

https://www.pfcla.com/fertility-services/egg-donor

Though being fair, that is opposite-sex couples mostly, with same-sex male couples and single women coming in lower numbers.

https://www.hatch.us/intended-parent-journey

"Hatch" is either a wonderfully clever marketing gimmick or a godawful name reducing women to hens in a hatchery for the service of "we source and provide human ova for egg donation to IVF cycles".

"Start Building the Family of Your Dreams

Hatch is proud to have the largest fresh egg donor database in the United States! Our donors are healthy, beautiful, and accomplished young women who want to help families just like yours.

The first step is to view our egg donors. We recommend you (and your partner, if applicable) browse our donors and pick 2-3 that you’re interested in before booking your free consultation.

Experience the Empowerment of Egg Donation

Set yourself up for future success while giving the gift of life to intended parents. When you donate eggs through Hatch, your thorough screening process will determine your base compensation and additional perks you’ll receive throughout your journey.

As an egg donor among the top 5% of candidates in the nation, you’ll receive generous compensation, emotional rewards, and thank-you gifts. When you work with Hatch, you won’t just be a number -- your dedicated coordinator will work with you closely to meet your needs and provide you with the smoothest, most enjoyable experience possible.

If you’re an intelligent, accomplished, healthy, and caring young woman who may be qualified to donate eggs, we’d love to welcome you into our competitive egg donor network.

Requirements for Egg Donation

Age

Hatch egg donors must be between the ages of 19-30.

Regular Menstrual Cycles

You must not be currently using Depo-Provera.

Reproductive Health

To become an egg donor, you shouldn't have a history of reproductive problems.

Healthy Lifestyle

Our egg donors must be physically healthy and within a healthy BMI range.

Healthy Family Medical History

Egg donors must not have any family history of inherited genetic disorders.

An Open Mind

Egg donors must be willing to take fertility medications and keep appointments.

Non-smoker

No current use of drugs or excessive alcohol

Must have both ovaries

Psychologically healthy

Humanitarian motivation

Dependable and mature"

Expand full comment

You're not striking a massive blow against human commoditization by personally opting out. You're only reducing the number of people like yourself.

Expand full comment

Great, the rest of humanity can proceed merrily to commoditize itself in ever-increasing numbers, happily adopting the Virus Ideology, "MORE !!!". I will be happy to know in my last moments that I prevented a vast number of potential descendants from ever being dehumanized and commoditized like pets and slaves. Not existing doesn't hurt my bloodline, only existing in terrible and demeaning conditions does.

Expand full comment

But modern dehumanization and commoditization mostly doesn't hurt. Your ancestors had been literally enslaved and murdered and bought and sold by neighbors to whom it had not even occurred that they were human. Your ancestors begged "MORE !!!" for a piece of moldy bread. They were dehumanized and commoditized and abused and slaughtered on a level you probably can't comprehend. And yet they rose above it, struggled, survived by succeededing generations until their descendant is here online today. And you're afraid that a JOB APPLICATION is too hurtful for your descendant to exist?

Expand full comment
Jun 10·edited Jun 11

You are correct that sperm donation is meaningfully different from organ donation. Still, within my own partially unconscious mental process, I think there was a halo effect of "See, the weird donation of something from your body might actually be not that weird".

(Consciously, I still think that EA-style organ donation to total strangers is weird and I do not want to live in a word where keeping your damn organs would be viewed as unethical, but seeing a notable person doing it and signalling it to a large public audience still has a normalizing effect.)

I donated to the French legal institution (CECOS) where the donor choice is almost entirely made by the doctors, not the receivers.

Edit to add: your conclusion stands but I must point out that donating sperm was much more hassle than blood: three appointments scheduled months in advance, transport to a relatively remote hospital, lengthy psychological survey because we are a rare enough occurence that they want to know what kind of people we are, talk with a psychologist, talk with a genetician to discuss my family health history, and blood and urine sampling on top of the donation. Plus the note to the future child as I said.

Expand full comment

Sperm donation is indeed significantly different from organ donation.

Sperm donation is intended to assist in the creation of a whole new life. Organ donation is intended to assist in the continuation of an existing life.

Expand full comment

I don't normally think of myself as EA, but I don't think the intended effect of EA-style organ donation to complete stranger is to make keeping your organs unethical, not while you're still living anyway. I think it's a sort of "Above and Beyond" measure, tantamount to donating a year's salary or something like that. Think of EA as a deliberately-weird exploration of moral space, donating organs to complete strangers just so happen to be one very weird but a prima facie good thing to do, and so EA people have done it. Nobody sane would say that anybody is entitled to your organs while you're still living, especially when that "Anybody" is an unknown person.

I do think that it's immoral that it's normal for people to keep their organs after their death. Worms are fine and all, but they can eat plenty of things other than human organs, and there are people who can be saved by using those future worm meals a much better use than any worm could. I don't respect religions and burial rituals that prohibit taking organs from the dead body to save the living. I support legislation to make donating organs on death opt-out, i.e. every dead individual will donate their organ to a local hospital by default unless they explicitly say they don't want to, and I support mild punishment to those people who do choose to opt out, like for example reduced priority if/when they need organs.

If/When burials in space become common, and if society reaches a level of ambient organ donations that it's no longer necessary for every dead body to donate their organs, I think it's reasonable for people to want the body of their loved ones to remain "whole", by burying it in a non-atmospheric environment where it will remain intact for a long period. But when burial in the soil or throwing into the sea or burning to ash are the only choices for a dead body, I don't think there are any rational or moral defenses of not donating your organs on death.

Expand full comment

I don't think donating ensures that your donation will actually be used.

Expand full comment

It does not. Although it seems* that here in France, due to the recent opening of medically-assisted procreation to all women regardless of marital status, there is currently more demand for sperm that their are donors. On the other hand, I am white and it seems that non-white donors are more in demand. The institution writes things like "donors should reflect the diversity of society" next to a picture of black people, but when I asked the staff denied that they have enough white donors.

Everything in my OP can be prefaced by ""possibly" or "probably".

* https://www.francetvinfo.fr/societe/pma/pma-les-delais-s-allongent-en-raison-d-une-hausse-des-demandes-et-d-un-manque-de-donneurs_6243462.html

Expand full comment

I'm still trying to get a handle on to what extent one might say an LLM (and other such ML/neural net-type AIs) can "learn", "abstract" (as in a general principle), and/or "reason"...

...not in the sense of "how much are they doing these things /like a human would do them/", I mean — nothing to do with sentience or personhood or whatever — but, rather, re: the question:

→ • How much of what they're doing is "I saw this in training data --> apply trained response",

/vs./

→ • How much are they able to take something like a general principle or heuristic from training data, and successfully apply it to an entirely novel input?

In other words, to what extent must something be out-of-distribution before our AI is helpless before it?

--------------------------------------------------------------

To make it more concrete (...or else just totally confuse the issue with bad examples, heh), are we talking:

→ • "I know oranges, but not lemons"—level-of-inability to handle things not absolutely specifically seen in training...

...—Or are we talking more like:

→ • "I have learned not only the concept of 'citrus fruit', but also can identify when a candy is supposed to be a fruit, and the relationship between e.g. lemons and lemonade — without any explicit reference to these concepts in the training data"...?

--------------------------------------------------------------

--------------------------------------------------------------

The reason I'm having trouble in coming up with an airtight(-ish) answer to this question is, I think, that I still don't know enough about the technical side — I had hoped that just playing around with e.g. ChatGPT would do the trick...

...but I suspect the inevitable objection to any conclusion drawn therefrom would be:

→ "Nah, it's not REALLY being presented with a novel situation, bro — its training data definitely had a bunch of essays on exactly this topic!"

Hard to really determine the boundaries between "out-of-distribution" and "well-within-same" when the distribution is unknowably huge!

--------------------------------------------------------------

--------------------------------------------------------------

As before, if anyone has links to interesting news or studies or the like on the topic of AI / LLMs — and it does NOT have to be recent: I want them all!

Your thoughts, too! — hell, doesn't have to be an academic study or press release, by any means; I'm just trying to build a comprehensive picture, really, and it's interesting stuff...

...and I recently lost my entire saved "fascinating and suggestive AI articles" file, so I have years' worth of half-remembered maybe-facts like—

"I think one time an AI did this one thing; AND! if you can believe it, it... it made it seem... like...

...well, I remember that it was highly indicative of SOMETHING, okay?"

—hovering around in my head and leaping helpfully forth every time a topic impinges upon them and could have been improved by their addition to the mix /if I actually had the actual details aaaargh/

--------------------------------------------------------------

...Ahem. Luckily, it's no big deal: I am able to handle frustration & being stymied for nearly as long, and almost as well, as some toddlers do.

(— IFF they're tired and hungry, mind you. Sure, no one wants the crying and the banging on the table and the screaming of NOW NOW I WANT IT NOWWW and all to start... but if you pit me against a well-rested three-year-old, well — I'm sorry, but you'll have forced my hand.)

Sure, I've TRIED saying "look just trust me you wouldn't even want to look at the source for this claim anyway" — but for some reason that just tends to make people even more suspicious. It's a hard world, mi amigos.

.

.

--------------------------------------------------------------

--------------------------------------------------------------

[++main comment ends here++]

[++praise the omnissiah++]

--------------------------------------------------------------

--------------------------------------------------------------

.

(Final note, just re: previous comment again:

• As commenter Godshatter helped out quite a bit, I have — as requested & as promised — made a small donation to a local animal rescue; if anyone else puts in a lot of effort into engaging with me/the topic, I'll gladly do something similar, or throw you an Amazon giftcard or the like, as a token of appreciation.

(Cheers + my thanks for your time and/or assistance, y'all!)

Expand full comment

Maybe you could learn more by looking at this (assuming you haven't already seen it) https://www.anthropic.com/news/mapping-mind-language-model. It's Anthropic trying to isolate neurons/concepts in Claude. I don't know if there's a smaller open source version where you can play around with it. Depending on your time maybe you could do something like try to remove lemon from a dataset, train a model on it and see how it does when you add lemons?

Expand full comment
Jun 10·edited Jun 10

The fundamental value of a neural net, when tuned correctly, is that it abstracts out and embodies patterns that can be applied to data that were not in the training set. In fact, that's how training works: given a set of inputs and desired outputs, we train on some, and measure success using the rest.

Generative neural nets aren't fundamentally different: their outputs will resemble their training data, because of the patterns that the net has been trained to embody. But given novel input, they can produce surprising results.

Expand full comment

Why is it so hard for Ukrainians to target Russian rail infrastructure? As I understand it Russia runs virtually all of the supplies to their military on rail lines, and then most of the (obviously very large, very remote) country is serviced by the railroads- that's how they transport basic goods like food to cities across their gigantic country. Is it really that hard to, like..... blow up a few remote rail lines? A bridge that a train crosses?

In particular it seems like the Ukrainians could place mines on the tracks at night for trains to hit. Not a train expert but I really don't see how a train doesn't derail if it hits an explosive- particularly if you placed it on a corner. You wouldn't even need new or specialized mines- just place a few old-school ones on a corner at night before a train comes around. Why isn't Ukraine targeting Russian railway infrastructure more?

Expand full comment
founding

I'm pretty sure Ukraine wouldn't have any trouble *targeting* rail infrastructure, with the caveat that most Western-provided weapons come with a "must not use against targets in Russia" caveat. Still, they have some of their own deep-strike weapons, and some of the Western restrictions have been waived.

What would be difficult, is *destroying* rail infrastructure. Or damaging it to the point where it cannot practically be repaired. Railroads, and especially railroad tracks, are exceptionally robust because they are very easy to repair. Put a Tomahawk cruise missile or even a 2,000 lb JDAM right in the center of a set of railroad tracks, and those tracks will be back in service maybe six hours after the repair crew arrives.

Meaningfully destroying a railroad during the American Civil War, was a job for an entire division of troops (~20,000 men) working for a day or two. Tearing up all the track for twenty miles, using the ties to build bonfires, then using the bonfires to heat the center of each rail segment red-hot so a bunch of guys could twist it around a nearby tree like a bowtie. With high explosives that can be a bit less labor-intensive, but you still pretty much have to target each and every forty-foot length of track over a broad area.

Or maybe bridges, but small bridges are easily replaced and the large ones are both tough and well-defended.

Targeting the rolling stock, especially the locomotives, can work, but they keep moving and (contra the traditional dictator's ambition) they do not reliably run on a fixed schedule in a war zone. Ukraine can get satellite images of where every locomotive in Russia was as of the last satellite pass, if they want, but to know where they'll be three hours from now when your planned drone strike arrives takes persistent surveillance. That's not something Ukraine can easily do on or over Russian territory.

Expand full comment

Soviet rail infrastructure was made to survive nuclear attack and its operators have a bunch of rapid repair experience as well as organizations specifically meant to keep them running in wartime. Ukraine has them too which is why Russia's having trouble permanently disabling Ukrainian power infrastructure. Having your infrastructure planned by a chronically paranoid dictator to survive the nuclear age has some benefits.

Expand full comment

First, the Russian railways are behind their battlefield lines. Ukraine can't just walk past half a million Russian soldiers to go blow up their railways. Covert operations are probably harder now than at any time in history due to the pervasive surveillance of drones and satellites. So mining is not practical.

Air strikes are a possibility, but Russia has a lot of surface to air missiles and their own air force to contend with. Plus, it has been very difficult for both sides to position their aircraft in range of the enemy. Airfields are big, easy targets for cruise missiles and drones. So the airfields have to be located to the rear, which severely limits the range of strike aircraft. Ukraine destroyed a number of Russian helicopters sitting on the tarmac, because the helicopters have to be closer than jet aircraft.

That leaves two possible strike avenues available to Ukraine, drones and cruise missiles. Drones are very good at hitting vulnerable targets like aircraft and tanks, and much less good at destroying concrete and steel buildings. Ukraine has launched a number of attacks against bridges in Crimea, but the damage was not severe enough to be disabling and the Russians quickly repaired the bridges. Naval drones are much better for this because they can carry a much larger and more destructive payload than airborne drones, but that isn't helpful for rail bridges in Russia.

Cruise missiles all come from the West, as Ukraine doesn't have the domestic production to make their own. Western nations have been reluctant to let their missile systems be used on targets in the Russian interior for diplomatic reasons. Ukraine has shown no such reluctance for their part, and the helicopter base attack I mentioned earlier was carried out with US ATACMs. The other major issue here is that bridges are hard to destroy. Again Crimean bridges have been struck by Ukrainian missiles but not destroyed.

The Russians have anti-missile systems, the premier system being the S-400, that can shoot down enemy missiles. These defenses can be overcome with saturation, where a large number of drones and decoy missiles overwhelm the systems and allow the real missiles to hit their target. However, this requires a large expenditure of assets that Ukraine only sporadically receives from the West. So the cost-benefit return on burning a bunch of their precious cruise missile stocks to maybe disable a rail bridge that the Russians would repair/rebuild in a number of months is not very good.

Expand full comment

And as for derailment, it's another one of those things that's harder than it seems.

https://www.youtube.com/watch?v=agznZBiK_Bs

Expand full comment

Rail lines are easy to repair and replace. Seriously impacting a given rail line isn't something you do with just one bomb or two, it takes a major and sustained effort.

Bridges are hard to hit.

https://en.wikipedia.org/wiki/Thanh_H%C3%B3a_Bridge

Expand full comment

I’ve helped repair damage after a train derailment and get the line running again. You’re right, it’s a routine procedure unless damage to a trestle is involved. My experience was a mid summer dawn to dusk job, trotting around with a spike maul to reenable the main production line in an iron mine. As a young and fit man at the time it was a fun adventure. I was pretty tired the next day though.

If you have a Bridge Over the River Kwai situation it will set your enemy back for a while and make for some good cinema. :)

Expand full comment

As "Bridge Over the River Kwai", and other films like "Hannibal Brooks", show, it's best to blow up railway lines when a train is just about to run over the lines in question!

Expand full comment

The weapons that would be most effective for striking railway targets in Russia come from the west, and the western suppliers prohibit their use against targets in Russia, for fear of escalating the war. The US did recently loosen the rules slightly, but only to allow strikes against Russian targets near Kharkiv.

The Russians also know that their logistics operation is highly dependent on railways, and therefore police their railways quite closely. It is likely that simple sabotage devices and mines would be discovered befor they did much good.

Expand full comment

Authoritarian states are obviously very interested in mining social media as a means of reading/controlling the minds of the population.

They're able to do this with fairly effectively even though much of social media's interface is really crude (like, eg., fb is just a clunky inferface saying what/who you like).

What, then, will happen when social media is not a crude interface along the lines of fb, but an unbelievably more subtle one like neuralink might offer? If neuralink's telepathy supplants the social media we have now and plugging our mindstreams into the internet becomes our new way of relating with each other, China is obviously going to be interested. If putting the brain online (and thus recording online its every fluctuation) becomes normal, states perusing such data will have an immeasurably better map than we have now of the mind and of how to manipulate it.

All this seems like the inevitable way things are going, but how do we combat it? I'm surprised there aren't many tech commentators worried about neuralink's potential authoritarian applications.

Expand full comment

>What, then, will happen when social media is not a crude interface along the lines of fb, but an unbelievably more subtle one like neuralink might offer?

Anime pathos battles, obviously. https://www.youtube.com/watch?v=onYSNgHbbW8

Alternately, live action anime battles. https://www.youtube.com/watch?v=Uwl5MBzTCRQ (In the far-flung future of 2021.)

Alternately it fries everyone's brain because software loves to glitch out and crash and if you make your brain software your brain will love to glitch out and crash.

Expand full comment

Theorem: Any technological innovation that makes information easier to get into a brain - such that its humanist proponents will envision it being used for learning, communication, cultural exchange, and all-around hippie-esque activities involving information - can and will also be used for propaganda, religious indoctrination, state repression, and other very unhippy-esque activities involving information. Learning/Communication/Information technology is value-neutral.

Interestingly, Information-Suppressing technologies exhibit the same property. Encryption makes repressive parties not able to read into what their dissidents are saying, but it also enables shitty corpos with "subscription model" fetishes to do uncrackable DRM. Everything you can do to information, you can do for good and for ill.

> how do we combat it

Something something the No-Cloning theorem in Quantum Computing? Something something extremely strong hardware-based encryption using a full copy of your DNA + a hardware-random seed as part of the encryption key? Emergency faraday-cage helmet that can be mechanically deployed to airgap your implants whenever you suspect someone is tapping into your brain? Using Homomorphic Encryption (https://en.wikipedia.org/wiki/Homomorphic_encryption) so that the data will never leave your brain while unencrypted?

The Quantum Thief trilogy (https://en.wikipedia.org/wiki/The_Quantum_Thief) had elements of this in one of its subplots, I forgot which of its 3 parts. A society is privacy-obsessed, to the point that you're not in control of which memories you remember: Someone can decide that they want you to forget them, and so delete your memories of them. This seems like the opposite of data integrity, but it also mentions in passing that the implants use Quantum Computing (cue in some technical mumbo jumbo, but the author is a mathematical physicist so it's probably not Star-Trek-grade babble) to ensure security.

The hardest thing about this is not the technical possibility of protecting your brain, but convincing people that the hassle is worth it. Like all security situations, it's an arms race. At some point the arms race ramps up to a point that the average person decides that they're not going to bother. How many people today install ad-blockers? Use tor? Checks that whether their new "smart" screen or "smart" refrigerator is phoning home and block the shit out of it in their LAN? Convenience is king. People had a perfectly good anonymous and secure peer-to-peer payment system for thousands of years - Cash - and they are gradually sacrificing it away because the little trivial conveniences of card payments are just too much.

Note how this is completely orthogonal to the technical/non-technical divide. It's the "technical" people who keep writing software and designing hardware for the IoT trash after all, writing it with glaring security vulns, and in C or C++, not even the bare minimum of writing it in Java or Rust, which will eliminate a whole class of security bugs.

Expand full comment

Have you looked at the results from Anthropic, who are developing tools to inspect what parts of a LLM are in use, and determine what areas guessing to what concepts? Do you remember the Golden Gate Bridge thing? Imagine that, but with "I am a loyal and obedient servant of our glorious leader".

Expand full comment

I've always felt that "neuralink will let us directly communicate with our brains" is actually a dubious claim. Brains have a lot of variation. How do we know our innermost thoughts can be translated this way?

Just for some well-known examples, not all people have inner monologues (but I have a very active one) and people vary widely on their ability to picture things (I have a fairly weak ability). If you "plug in" my brain along with somebody with hyperphantasia and no inner monologue, are the odds really that good that we can just think things at each other and be understood?

To my mind it's like plugging in two independently-designed-and-trained LLMs and letting them directly access each others' activations - probably not actually a useful form of "telepathy".

Expand full comment
Jun 10·edited Jun 10

If large numbers of people voluntarily agree to connect their brains to the internet (assuming this is actually possible or remotely feasible and reliable at scale, an assumption that is hardly ever made explicit enough) then that will be the final nail in the coffin for my, and everyone else's, faith in humanity. Humanity will deserve to be wiped out and a nuclear war would be preferable. And will probably happen.

On the other hand, there will always be a significant number who will never ever submit to this. So a totalitarian government will need to kill lots and lots of people if they want to implement a mandatory brain-linked internet. And I struggle to see how this could ever be a rational policy for any dictator. Huge numbers who are otherwise content to obey will rebel if this happens. The high chance of that snowballing and bringing down the government is so high it wouldn't be worth the risk. Is that risk worth the possible gain in control (especially since realistically you couldn't monitor everyone anyway, there's just too much information)?

I see no plausible way the Chinese government saying "we will now be mandating brain uploads to find the citizens with treasonous thoughts" doesn't end for them the same way Robespierre telling the National Convention "I have a list of traitors in this very convention who I will be revealing tomorrow" did for him.

Expand full comment

I'm not a paid subscriber and thus don't have access to the post responding to Hoel, but from what I know of him he bans anyone who points out he's gotten anything wrong (or in my case, links to someone who did that and got banned + blocked on twitter). https://entitledtoanopinion.wordpress.com/2023/02/17/a-comment-i-would-have-made-at-erik-hoels-were-i-not-banned/

Expand full comment

He banned me for having a minor run-in with someone else in the comments. Other commenter and I were clearly annoyed at each other, but neither of us said anything awful. The exchange would have been no big deal here.

Expand full comment
Jun 10·edited Jun 10

Ah, I think that happened to me! I think I'm banned, and couldn't figure out why. I must have said something about the piece linked by Scott which upset him!

EDIT: If I was feeling really mean, I'd do a book review on his novel. Just based on the Amazon sample, let's leave it at "I don't like it" 😁

EDIT EDIT: Not banned as such, but when I accidentally clicked on the "subscribe" for his Substack with my email address from here, it came up "not allowed". So I tried again, and same result. Either this is a Substack thing or he blocked me on purpose, I have no idea which.

Expand full comment

Why not do a book review?

Expand full comment
Jun 10·edited Jun 10

(1) I only read the free sample on Amazon and it didn't entice me enough to want to pony up for the book (2) it would be too mean, and seem as if I'm only getting revenge for being (deliberately/accidentally) blocked.

The extract I read didn't impress me, and I could make some witty*/sarcastic remarks about the blurb etc. but this is too personal to do. Were I doing it for a stranger, or for Hoel without being aware of any blocking on his Substack, it would be defensible, but right now it would simply be saying snarky things in what looks like a fit of pique (I'm not actually sore about the block, just rather surprised).

*I'd hope they'd be wit, but they might be something that rhymes with "wit".

Expand full comment

A question about the longterm effects of psilocybin: a few months ago, a friend of mine who works as a computer scientist was experiencing a particularly strong bout of depression. Hearing about psilocybin’s potential as a treatment for depression, he took psilocybin for the first time. His experience was pleasant at the beginning and end of the trip, but midway through he felt terrified and anxious, with the perception of being permanently trapped inside a soundproof glass cage.

Later, when he tried to sleep, he woke up in the middle of the night experiencing a sensation and sound he described as a hundred birds flapping their wings directly behind his head. It caused him extreme anxiety and he couldn’t fall back asleep. The same thing happened the next night, and continued happening every night for a month. He was only getting a few hours of sleep a night, because he would always wake around 2am with the same strange sensation.

Eventually the sensation went away, but the anxiety and insomnia remained. Almost every morning he felt consumed by depressive and anxious thoughts, and he found it hard to work. Now, even on days when he gets more sleep than usual, he feels unable to tackle scientific problems with the same sharpness. This is a person who had no previous history of insomnia.

Do any of you know if psilocybin can trigger chronic anxiety or chronic insomnia? And any idea what the mechanism might be? Can psilocybin cause longterm changes in brain function? How commonly does it make depression worse instead of better? Do any of you have any suggestions for my friend? Thank you.

Expand full comment

"Survey study of challenging experiences after ingesting psilocybin mushrooms: Acute and enduring positive and negative consequences" (2016) by Carbonaro et. al found that ~10% of participants who had adverse effects from psilocybin reported the effects lasting for over 12 months (of those that had tried psilocybin at least 12 months ago).

This was an survey of 1,993 online participants that reported having a "bad trip" ("significant fear, anxiety, or distress or anything else that you found psychologically difficult") from psilocybin mushrooms, so take it with as many grains of salt as you wish.

Most participants reported that the experience, despite the adverse effects, improved their current well-being, but 8% reported the experience making it worse:

"Although most of the participants (76%) reported that experiences during the psilocybin session led to increases in current well-being and life satisfaction, 8% reported that the chosen challenging experience resulted in a decrease in their sense of well-being or life satisfaction (slightly or greater)"

Duration of negative symptoms was negatively associated with well-being:

"Multiple regression analysis showed degree of difficulty was positively associated, and duration was negatively associated, with enduring increases in well-being"

Expand full comment

Thank you for this information. If you know of any researchers who might find it useful to hear from my friend about his account of enduring negative effects, let me know.

Expand full comment

(Epistemic status: I'm NOT deeply knowledgeable on this topic. I'm providing advice in babble mode because you explicitly asked for ideas. It is up to you to prune)

From what I've heard / absorbed by osmosis, these drugs make you more suggestible in all manner of ways. They basically destabilize the priors the brain has. This can be a great thing or a terrible thing. Your friend's story isn't unique.

I hope you're familiar with predictive coding (extensively discussed by Scott) or at least the general thesis that the body / mind is primarily concerned with maintaining balance of all kinds. E.g. your auditory system is tuned such that an "appropriate" amount of information is passed along to your conscious. It seems quite natural that disturbing this kind of fine-tuning may result in hallucinations.

The mad scientist way of fixing your friend is to have him take another dose in a positive environment under an experienced sitter who can make sure it's a good trip. Said sitter might be able to gently recalibrate the brain down to a more normal state of functioning. I have no idea if that kind of thing has been successfully done. Find experienced people and ask them for advice.

Expand full comment

Thanks so much for your reply! As you probably imagine, I doubt he would be interested in trying psilocybin again after the last experience, but I'll pass on your comment for him to consider and discuss with people.

Expand full comment

Psychologist here: The psilocybin studies do get impressive results, and not just for depression, but all involve having the person prepare for the trip with a professional, have a guide present during the trip, and then have several post-trip sessions with the person they started with to process the experience. Also, most involve 2 trips, the first at a lowish dose -- then for the second trip the subject decides whether they want to take the same dose or a higher one. I have spoken with lots of people who tripped on the stuff alone and felt greatly helped by the experience, but some people do draw the short straw and have a terrible time.

The likeliest reason your friend is having worsened anxiety, insomnia and depression is not that he was harmed by the drug, but that he was harmed by the anxious state he spent a lot of time in while tripping. Something a lot of people don't understand about panic attacks and other severe anxiety states is that they leave the person vulnerable to further ones. It's like the person got a glimpse how just how alone and doomed it is possible to feel, and they walk around being afraid it will happen again, and sometimes are so afraid of it happening again that their fear of bringing on the state brings that on. I can't tell whether your friend had an actual panic attack during his trip, but he did spend the long middle of his trip feeling terrified and isolated, and I think the hours he logged struck in that state are the relevant thing here.

Psychedelics can indeed bring on a schizophrenic break in someone who is vulnerable to one, but they usually don't, even in people at risk who take it during the riskiest years for a first break (something like age 15-30). And your friend does not sound psychotic, he sounds miserably anxious and depressed. Difficulty concentrating is a very common depressive symptom. Even if he has not had it before, depression's still the likeliest explanation of his concentration problem now. Maybe he is having it now because he is more depressed than before, or is quite anxious in addition to depressed.

I think he should go see a talk therapist who is experienced with treating anxiety disorders. The point of that would not be to start a long, exploratory therapy, but to help him recover from the acute state he's in now. In fact, just giving him the info I've given here may help some, if he's walking around thinking he's broken his brain. I expect there are meds that will help him top, but I don't think he should just see a psychopharmacologist because he needs help dismantling and recovering from the acute state he's in.

By the way, I had an drug experience with a result similar to his when I was 17 or so. A friend and I were eager to try LSD, but could not get our hands on any. One of us had read that back in the old days sailors used to eat nutmeg to get high, so we each ate a tablespoon or so of the stuff. It didn't make us high, just sort of dizzy and confused, with mouths dry as paper, but I was afraid I was going to die and spent an hour or so being terrified of that. Eventually we went to the ER and they gave us IVs and maybe some med or other, and we were fine the next day. But for the next few months I was subject to panic attacks and depersonalization, and stopped enjoying things. I'm confident that that state was the result of how bad a scare I had, not the nutmeg itself.

Expand full comment

Thank you so much! Very kind of you write such a thorough reply. I think it will be helpful for my friend to read this, and I will pass it on to him.

Expand full comment

It may trigger schizophrenia if the person is already susceptible. What is their age?

Expand full comment

He's in his early 40s. Besides the particular hallucination that would repeatedly return each night, he didn't mention any other hallucinations. I don't know whether he has family history of schizophrenia or not, but I don't think what he described would be considered psychosis.

Expand full comment

Medically supervised experimentation at Johns Hopkins won’t allow anyone with a close relative with schizophrenia take part.

Expand full comment

Currently the implied probability (to the nearest whole percentage point, mid-price) of the following people winning the US Presidential Election is, in the order Betfair/Polymarket/Metaculus/Manifold (numbers 4 weeks ago in brackets):

Trump: 53/56/55/51 (47/45/49/46)

Biden: 36/34/48/46 (40/45/50/52)

RFK: 2/2/1/0 (3/3/1/1)

(Michelle) Obama: 3/5/1/0 (2/5/1/0)

Newsom: 2/2/1/0 (1/0/1/0)

Harris: 1/2/1/1 (1/1/1/0)

Whichever way you look at it, there’s been a clear shift from Biden to Trump over this period. The real money markets continue to put Biden’s chances well below those in the play markets, which appears to be mostly because they give some significant weight to the possibility that Biden will drop out (24% chance, per https://polymarket.com/event/will-biden-drop-out-of-presidential-race/will-biden-drop-out-of-presidential-race).

Metaculus has a basic problem: they give the chance that either Trump or Biden will win at 103%, while also giving 15 other people each a 1% chance of winning. I’m not sure what to make of this.

Expand full comment

Let's suppose I'm a rich person willing to spend some hundreds of thousands of dollars to help my preferred candidate get elected in whatever the most efficient way is. Should I consider using some of that money to manipulate prediction markets? I think the best way to do it would be to deploy it at strategic moments, to make a bad news cycle look less bad, or to make a good news cycle look better than it is.

Cons: not that many people openly pay attention to prediction markets (but those who do may well be upstream of people with a larger audience)

Pros: it's relatively cheap to move most major prediction markets. You have a real chance to influence the narrative at a crucial point, which you don't get from donating another incremental $100K to an already-wealthy election campaign for yet more ads.

Expand full comment

As it happens, I know that somebody dropped $450k on Trump on Polymarket on 22 January. That had the effect of moving the market from 49% to 53%.

Expand full comment
Jun 10·edited Jun 10

Do I read this right? The Biden drop out market has $4,429,996 riding on it, while the corresponding Trump bet has only $397 and basically no activity? What's going on there?

https://polymarket.com/event/will-trump-drop-out-of-presidential-race?tid=1718045266120

Expand full comment

> I’m not sure what to make of this.

You should make money and/or fake internet points?

Expand full comment

I have two new Posts:

“Incentives” where I discuss crafting the correct incentive f public investments/subsidies for industrial policy, supply chain security, and climate change;

https://thomaslhutcheson.substack.com/p/incentives

And

“Energy Policy,” where I use the mistakes of the 1975 energy policy to express skepticism about Roger Pelkie’s ideas for a 2026 energy policy.

https://thomaslhutcheson.substack.com/p/energy-policy

Comments either here or on the Substack or better, both are very welcome.

Expand full comment

I'm growing extremely confused why "calculator notation"(2+3*4=20) is basically nonexistent in computer science with an insane preference for inverse polish notion and the 2ndary option being stack machines. Conceptually the 3 machines are roughly the same complexity to implement, and inverse polish notion is obviously terrible to understand.

Expand full comment

"Calculator notation" is basically nonexistent everywhere outside of cheap calculators that can't store expressions and are forced to just evaluate as you type. Scientific calculators and phone calculators don't use it.

Expand full comment

I use kdb a lot. It has right to left notation.

A = 4*3+2

A is now 20.

Because assignments are right to left, this reverse order feels natural to me. If the assignment was reversed it would make sense to flip the operators back

Expand full comment

RPN is the same fundamental idea as stack machines, you're thinking of Polish Notation when you think of its polar opposite, i.e. S-Expressions, f(a, g(b, ....)). S-Expression take the extra step of making this look like (f a (g b ...)), but it's a very minor variation on the theme.

RPN and PN make parsing trivial. They're basically just a flat instruction stream with extra steps, especially RPN. So that's why they're used in Bytecode machines and other binary formats so much. As for why they aren't used in source languages, that's the bitter lesson that Lisp advocates keep discovering over and over again: Concrete Syntax Matters. Nobody likes to write flat instruction streams, or indicate nesting very painfully with a soup of undifferentiated parenthesis, or simulate the stack effect in their mind and carefully learn to think alien thoughts to program. Most people simply hate it.

Infix syntax is natural, beautiful and light on the eye. Any particular precedence hierarchy used will do, PEMDAS just so happen to be a very common one. It's not used without modification, for instance primary school algebra doesn't teach the Left Shift or String Concatenation operators, but those are somewhat common in various languages, and so those languages typically modify PEMDAS to fit those. The fundamentally good thing about the idea is both the precedence hierarchy and Infix-ness: being infix means the operators/control commands disappear between their arguments, and being a precedence hierarchy means the common case won't have parenthesis all over, that there is a default ordering that does "the right thing" most of the time, and parenthesis are occasionally only needed to override this usually-sensible ordering.

2 languages that use infix but deviate from the precedence hierarchy idea are Smalltalk and APL. Smalltalk does strict left-to-right evaluation, because it thinks everything is a method call: 2 + 3 * 4 is really 2.plus(3).multiply(4). The meaning of "+" and "*" isn't even built-in. The mirror image of this is APL, which does strict right-to-left.

Smalltalk gains impressive extensibility, one disadvantage of non-uniform precedence hierarchies is that they're non-extensible, there's a part of the language that is fixed-in-stone and very difficult to change, Haskell and Raku has an infix syntax but still allow you to define operator-like symbolic functions, which proves that it can be done but must have been hell to support in the languages' parsers.

I'm not necessarily saying that the ad-hoc system we inherited from mathematics is the best syntax, but Lisp/Json/Yaml is the single worst replacement and people will never love it over infix. I love the likes of Raku and Mathematica because they massively innovate in their Syntax, showing us that other roads hitherto not taken are still possible.

Expand full comment

It’s not all that hard to parse user-defined operators.

A lexical analyzer can typically be represented as a finite state machine. If you represent this finite state machine as a data structure, it’s easy to add character sequences representing user defined operators to the lexical analyzer. (In an actual implementation, you would probably make the lexical analyzer use this data structure when it encounters a character that can begin an operator.)

One approach to handling precedence rules is to define a separate nonterminal for each precedence level:

expression ::= expression1 | expression "or" expression1

expression1 ::= expression2 | expression1 "and" expression2

expression2 ::= expression3 | expression2 plus_operator expression3

...

expression6 ::= integer_constant | identifier | "(" expression ")"

Then, when a trivial expression like the constant "0" is encountered, the parser has to reduce the integer_constant to an expression6, reduce the expression6 to an expression5, and so on until it finally reduces an expression1 to an expression. That’s seven steps to parse a one token expression.

If we use an ambiguous grammar, those seven steps are reduced to one:

expression ::= operator expression | expression operator expression

| integer_constant | identifier | "(" expression ")"

The parser has to use an operator precedence table to resolve the ambiguity. If you are writing a parser by hand, it’s easier to use an operators precedence table than to code a more complex grammer. If you are using a parser generator tool, it may be a bit of a challenge to get it to use an operator precedence table. In any case, once you have a parser using a precedence table, letting users define new operators with arbitrary precedences is easy.

The biggest difficulty with user-defined operators is that they complicate language tools that have to be able to parse a program, but, unlike a compiler or interpreter, don’t need to determine whether the program is valid.

Expand full comment

Isn't pedmas just the standard notation used in algebra, and thus pretty much all math? If that's what people trained in math use, why not use it for programming too?

Expand full comment

Id go further in simplifying "the stack" then most, but even right now its a given that only full languages should have pedmas; byte code and virtual machines shouldnt.

Expand full comment

PEDMAS is taught in school and used in many/most mainstream programming languages for pretty much that reason.

Reverse Polish is easier to implement, but mostly used by Forth (and Postscript). For all practical purposes compilers convert PEDMAS into Reverse Polish "under the hood."

Smalltalk implements math with what you are calling "calculator notation." But outside of Smalltalk there really isn't much reason to choose this method over PEDMAS which everyone who made it through 8th grade is supposed to know.

Expand full comment

full languages need a full ast and at that point you can implement PEDMAS sure

(tho, honestly at this point Id like to see compile warnings if you actually use it instead of having the extra ()'s)

but you need smaller machines to build up to that complexity

> For all practical purposes compilers convert PEDMAS into Reverse Polish "under the hood."

Are you sure? I hear that claimed, but bitcoin, jvm, llvm, asm, etc etc etc; stack and/or register machines

Expand full comment

Compilers translate into whatever the machine uses. Often there's an intermediate stage in assembly language, where things are often of the form "instruction arg1 arg2", which is basically Polish Notation. It's not hard to do the translation, so there's no reason for a high-level programming language to prefer one style of notation to another, except for usability.

Expand full comment

What do you mean 'basically nonexistent in computer science'? Do you mean it doesn't get taught? That's because everybody knows about it. Do you mean it doesn't get used in practice? All most used programming languages use it. I only know about Forth that uses reverse polish notation. Stack machines are not a notation form of calculations but a way to calculate them.

Expand full comment

> All most used programming languages use it

thats "pemdas" and it thinks 2+3*4=12, not 20

Expand full comment

I've never seen a calculator that doesn't abide by PEMDAS rules. My phone's default calculator app evaluates 2+3*4 as 14, as I'd expect.

The reason for why this is good is the distributive property, and that having a uniform convention is good for everyone here. Any other questions?

Expand full comment

try it on one of the cheap "4 function" calculators

I get its mathematically incorrect, but compare it to inverse polish notation

Expand full comment

In my initial comment, I failed to fully understand what you meant. Now I think I get it. You're curious why a rolling-result style 2 + 4 = 8 * 2 = 16 / 2 = 8, isn't more common over an equally easy to implement polish notation. If my understanding is correct, then the reality is that those notations are basically useless and never used in real programming. Everything just uses PEMDAS, so it's a bit pointless to focus on which unused notation is more commonly explored in academia. The only time you would use the polish notations is for uncommon languages like LISP.

Expand full comment
Jun 10·edited Jun 10

The only language that comes to mind that uses infix notation (operators between the operands) but not operator precedence is Smalltalk. And pretty much no one uses Smalltalk today.

I think you're probably right about operator precedence being implemented because of user preferences. Strict left-to-right evaluation would be easier to implement, but it would continually clash with the expectations of users who have had PEMDAS drilled into them in elementary school.

Expand full comment

> If my understanding is correct, then the reality is that those notations are basically useless and never used in real programming.

Bitcoin is based a very simple stack machine. The "shunting yard" algorithm is considered very important to learn for parsing languages. I think the jvm is a stack machine as well etc.

As much as I like the "academics just masturbating" reasoning, you kinda need simple machines for interrupting either tiny programs or bytecode, and when making more complex data formats(data compression)

Usually given two options of stacks and inverse polish, people go with stack even tho inverse polish notion is presented as the main one, but there's still 100000 videos about inverse polish notation

Expand full comment

It's easier to program.

Expand full comment

I just disagree

Expand full comment

I regularly read about how in medical testing mice are, whilst convenient, often not great analogues for human reactions for drugs, and hence a lot of drugs seem to work in mice but not in humans.

Do we have reason to believe the inverse is also true, that is, that many drugs that *don't* work in mice *do* work in humans, but safety controls (i.e. you need to test on the mice first) make that broadly impossible to quantify?

Expand full comment

Mouse studies should mainly be for safety. Ineffectiveness in mice should not rule out effectiveness trial with humans although they should have some weight in deciding which human trials are worth doing.

Expand full comment
Jun 10·edited Jun 10

There are loads of large lavishly illustrated plant encyclopedias. I see them in thrift shops, stacked by the shelf full, and as downloadable ebooks. Probably with a bit of effort, one could accumulate enough to cover practically every known plant in the world. But I have yet to see one of a type which would be very useful for gardeners, both amateur and professional.

The other week, I planted chives and wild garlic seeds in a couple of dozen large pots with fresh soil and fertiliser. But now loads of different plants have started sprouting from these. The majority are obviously weeds, But the problem is I don't know which is which. There is no reliable way to separate the sheep from the goats! Actually, I can tell chives from grass by feel: baby chives stalks feel soft and round, whereas grass stalks have a definite edge and a rougher texture. But I have no way of telling which, if any, are the wild garlic from among the several different looking sprouting seedlings.

So it would be useful to have a pictorial reference of common seedlings and young plants, with several pictures of each at, say, weekly intervals from one week to several weeks. That would allow them to be distiguished from weeds, and the latter could be discarded before they start choking the good guys!

Expand full comment

I kinda understand your frustration, but your method has a bit of a "print out the form, take a photo of the form laid on a table, embed photo in a pdf" vibe.

The GENERAL problem of plant id is difficult. The SPECIFIC problem of plant id in your own garden is much easier. Label stuff and take notes. I have 5 pepper varieties growing on my balcony right now, and the chance to ID them without knowing what I planted where is essentially nil until the fruits are pretty far along. Markers like "how fast and how tall does the plant grow" vary between cultivars, but vary waay more with weather and other external factors.

Actually, I can kinda distinguish the capsicum baccatum from the capsicum annuum, that's as far as it goes. So I would be pretty surprised if an ML based app can do anything more specific than tell you the family or occasionally the species. But between different types of garlic? probably not.

Expand full comment

Google Lens can do this for you.

Expand full comment

A good idea in theory, but without flowers and/or mature leaves, plant ID is very difficult - easier to use lollipop sticks to label your containers/rows.

Expand full comment
Jun 10·edited Jun 10

True, although I know what is in each pot (either chives or garlic). I just don't know where in each pot!

I could have placed the garlic seeds in a systematic order, as they are each the size of a match stick head. But the chives seeds are so small one would literally need a microscope to separate them. So I had to sprinkle them at random from a height. Maybe I should have germinated them on damp blotting paper, and then transferred each seedling to an egg-cup sized pot to grow on its own for a while, as I do with my baccy seeds.

Thanks for the other replies everyone! (I guess this discussion is rather lowbrow for this blog, but I knew people would come up with some good ideas!)

Expand full comment
Jun 11·edited Jun 11

Do any of the grass-like seedlings have a stronger garlic scent when you rub them between your fingers than others? I suspect they would have scent even at this stage and that I would be able to distinguish it from chives, though it might be subtle and more challenging for some people. If you can figure it out from this, you may be able to find a very slight visual cue that will help you differentiate. As others have said, from a botany stand-point there may not be any sure way to differentiate. Then it becomes a matter of observation and recognition. If there were two women standing with their backs to us, same hair color and length, same size, most of the time we can still distinguish the one who is our friend even if it would be challenging to describe what it was that let us recognize them (the shape of the shoulders, the way they stand, etc). I have found the skill of learning to ID plants or to ID birds in flight to be very similar to this and a skill that humans are capable of strengthening.

I don't think observational skills are low-brow.

Expand full comment

A weed is often referred to as "a plant, out of place". Next time, plant your chives/garlic in a pattern, in the pot, and then you can freely pluck what comes up out of order.

Expand full comment

> A weed is often referred to as "a plant, out of place".

I like the term ‘volunteer'.

Expand full comment

'Volunteer' is usually used for a plant that is otherwise useful and has decided to grow itself in a location of its choosing.

It's one of those 'all volunteers are weeds, but not all weeds are volunteers' situations.

I garden a lot and love volunteer plants as they are usually self-optimised for local growing conditions, making them quite useful.

Expand full comment

I picked the term up from a professional gardener using it in the sense you mean. After years of doing battle with the creeping charlie in my lawn I started to use it for any weed that I was sick of pulling up.

Expand full comment

Young men of old used to get some living done before they stared on their careers. Young men of today go straight from school to university to a career. Before Steve McQueen was an actor, he worked on a farm, in a circus, hitchhiked across the country, went to jail, served in the merchant navy and became a marine. Sean Connery was in the Navy, a milkman, lifeguard, labourer and model.

Many people who grew up in the 70s have a story like this but it all stopped in the 80s. These days, young folk go to school, go to university, get a job, get married and buy a house. What changed? Where did the spirit of adventure go?

Expand full comment

I suspect you only know a very narrow slice of "young folk". These steps are routine for the middle class - but the "too poor for university" class and the "too rich to bother" class still exist. Even within the middle class, I used to follow an influencer who worked at the mall during high school, sold jewellery on Etsy, started a YouTube channel and became an influencer while attending UCLA (simultaneously freelancing as a videographer and editor), and is now a model/videographer/director shooting ads of high end fashion labels. She's still in her 20s.

This isn't an unusual story among creatives who got their degree ~ 2008 or slightly after, mostly because of the unavailability of stable full-time jobs.

I also suspect a lot of people are rushing to the "buy a house" step and delaying any adventures to after the house is secured. It's a lot easier to drop everything and join a circus or whatever if you're not worried about making rent - for an equivalent house, rent has been consistently more expensive for at least the last decade, and foreclosure takes about 2 months to eviction's 2 days. As someone else mentioned, young people are also subject to more debt, and also, a highly sophisticated global surveillance state that makes it all but impossible to escape said debts by doing dramatic identity reinvention ala the Great Gatsby or Mad Men.

Expand full comment

I may have a bit of selection bias going on but here’s one little anecdote.

In my twenties, an agency called and asked, “Do you want to work in America?” I went for an interview in London and they hired about twenty of us. We got to choose from a selection of American states and corporations. Six weeks later ten of us arrived at JP Morgan on Wall Street and we all had stories to tell about similar adventures in our pasts. Two years later, I was working for a startup in Palo Alto.

The young people I talk to - mostly friends of my twenty-something kids - say that it’s impossible that you could get a job these days at a Wall Street bank or in a Silicon Valley startup without a degree.

Expand full comment

That kind of hiring environment sounds unreal to me.

Over here, in the 2010s, apparently recruiters would go to backpacker hostels and ask "who wants a job in the outback?" But the jobs are like, weeding, cooking, cleaning toilets, or making beds in the mines. Nothing requiring hauling (it's all automated) or docks (you need maritime or air security checks which random backpackers obviously can't pass). The majority of the other stuff requires certs.

Expand full comment

You have a point about young creatives. My daughter is a young creative with a Master’s. She is having trouble getting started with Real Life but she has a fun part-time job while she searches and she is off to China in a couple of weeks.

I also agree about the house ownership thing. Yes, buying a house has become a major focus for young people starting out in a way that it wasn’t 40 years.

Expand full comment
founding

I think your generalization of "young men of old" is inaccurate, and I suspect that's because it is based on famous actors (or famous people generally).

I don't think any of my family who came of age in the half century from 1920-1970, did anything but go to school, get a job (unless female), get married, and settle down to a career that follows from their schooling and first job. The biggest class of "get some living done" was a bit of non-career military service, but the stars aligned for all the Schillings et al to be the wrong age or gender for WWII, Korea, and Vietnam and none of them went for a peacetime enlistment.

But if you're looking at famous people, then there basically is no career path that goes "go to school, get a job, become famous" except by incredible luck, and doing more different things gives you more chances to get lucky. Going to acting school and becoming an actor, makes you a C-lister at best. Going to acting school, getting frustrated with acting, then working as Francis Ford Coppola's carpenter, makes you Harrison Ford.

So, "famous" strongly selects for unconventional life experiences. Can you find some relevant data on non-famous young men of old? And modern young men as controls, of course.

Expand full comment

I'm not sure whether this trend is real, but I'd suggest that movie stars have horrendously atypical careers and are probably not a great place to be looking. Actors either turn full-time in their youth or they spend years working a series of crappy jobs while waiting for their big break. Picking a couple of random examples of actors who I know didn't make it big until well into adulthood:

George Clooney "earned money selling women's shoes, selling insurance door to door, stocking shelves, working in construction, and cutting tobacco"

Jon Hamm worked as a waiter and as a set designer for softcore pornography (it had never occurred to me that this was a job category...)

Samuel L Jackson was a security guard, social worker, heroin addict and kidnapper (wew, that last one never seems to come up in promo circuit interviews)

Expand full comment

I chose them because they are well known. There are plenty of people in other careers who had adventure in their early life. Me, for example.

Expand full comment

Seems cherry-picked. I have no actual data, either, but my own experience is my dad was born in the 50s and married when he was 20, got a job with the county, had me immediately, never changed jobs, and never moved. I was born in 1980 and did more or less exactly what you describe. I lived in 10 different states within 7 years of turning 18. I joined the Army. I worked as a janitor, Disneyland performer, non-profit fundraising director, and bar manager before joining the Army. All before my real "permanent" career that I'm in now.

Where's this anecdata even coming from? You seem like you're probably mid-50s or so by now. How many 20 year-olds do you know? I know 0 and have no idea what young folks today are doing, but I at least know myself and quite a few peers born in the 80s who don't fit the description you're giving.

Expand full comment

Cherry-picked, and useful only insofar as it suggests "outlier people have outlier life stories," which isn't really news.

Expand full comment

> Before Steve McQueen was an actor, he worked on a farm, in a circus, hitchhiked across the country, went to jail, served in the merchant navy and became a marine. Sean Connery was in the Navy, a milkman, lifeguard, labourer and model.

The economy is now a lot better so you don't need to take shitty low paying jobs if you're a smart, capable person.

Expand full comment

Most social changes like that come from relentless optimization. Same reason many kids don’t have childhoods any more. My boss at my old company dropped out of school at 15 and still ended up successful, I don’t think that’s common at all any more.

Expand full comment

Right. That was my situation too. Lots of my friends quit school at 16 and 18 and we all did very well. It's a shame it's not an option any more.

Expand full comment

I don't have an answer but I think this is mostly a US/Canada problem. Source: I've been living the pre-career adventurous lifestyle you describe and most of the people I meet who are on the same track are European, sometimes Asian or Latin American, but almost never Anglo American.

Expand full comment

And uncivilizedengineer just below gives one of the reasons:

> You graduate with a mountain of debt

Expand full comment

I appreciate your point and mainly agree, though I don’t think looking at actors is the most representative sample. I think it’s pretty obvious what changed, mainly the cost of everything going through the roof. You graduate with a mountain of debt, so you start off already in the hole, then if you want any hope of ever getting ahead, you have to get a 30 year mortgage to be able to afford property. Put simply, the boomers were the most privileged generation to ever live, and now they continue to reap the benefits as they sit on the properties they bought for a steal, demanding ridiculous prices to fund their cushy retirement.

Expand full comment

I chose the actors because they are well known. There are more examples in my Substack. https://raggedclown.substack.com/p/the-spirit-of-adventure

Most of my friends left school at 18 in the 80s to work in finance or nursing or to join the army. I left school at 16 and was a sonar engineer on a nuclear submarine at 21. By 23, I was a software engineer.

It's true that you graduate with a mountain of debt, but why did you need to graduate? That's what I want to know. It seems like a con to me.

Expand full comment

By going to university I missed out on a bunch of life experiences I could have had in the Navy. But by joining the Navy I'd have missed out on a bunch of life experiences I could have had at university.

I've done a lot of things in my life despite following a reasonably straightforward career path. I always had the option of taking a year off and working on a farm, but I don't regret the things I did instead. Some things sound romantic but in practice they just reduce either to "working hard on something repetitive" or "hanging around at a backpacker hostel getting drunk".

Expand full comment

>>It's true that you graduate with a mountain of debt, but why did you need to graduate?

Yes. Business hiring practices are fundamentally different. My grandfather was a steel mill worker and HS graduate who took night classes in computers and was hired by IBM in the 60s with just the HS education and night school. I met one other IBMer with a similar story who was hired in the 70s, but he self described as the last of his kind. The culture of getting decent-paying work without a college degree has been basically been dead since the 80s. Trades exist as one option, but just because your training isn’t coming at a school doesn’t mean you aren’t doing years of it.

There are various movements and nonprofits trying to break the trend and get businesses less invested in using degrees as a filtering mechanism, but none I’m aware of has made much headway.

Expand full comment

That's right. I think the bias that someone without a degree has no training is harmful and expensive.

I hope the movement away from everyone getting a degree is successful. I think there is a change coming. I hope it comes soon.

Expand full comment

Pretty sure this is just false? In the early 20th century it was very common for people to enter a job and work for the same company their whole life. Now people move between jobs every couple years.

Expand full comment

I think you're overly sure in the absence of data. For example, there hasn't been much change over the last four decades:

https://www.pewresearch.org/short-reads/2022/12/02/for-todays-young-workers-in-the-u-s-job-tenure-is-similar-to-that-of-young-workers-in-the-past/

Expand full comment

A quick google gives me dozens of articles that say people are switching careers more than ever before. But not idea how they quantify it. I'd imagine its something you could dig into census data for. Your link seems to be comparing now to the 1980s but think OP is talking about further back

Expand full comment

I grew up in the 70s and came of age in the 80s. Most of my friends got their first jobs at 18 — in sales, in finance, in nursing, in software — but they didn't stay in those jobs forever. I got my first job at 16 and was on my third career by the time I was 23.

None of those jobs would be possible at 18 now.

Expand full comment

Does your quick google search turn up any actual data, or again just people giving their impressions?

Expand full comment

They might change jobs, but they pick a career and stick with that forever.

I think young people had a freedom until their early twenties back then that is missing now. Not everyone had this freedom of course — only those who chose to take it.

And it's not just young adults with careers. Children as young as eight were allowed to roam free in the 70s in ways that would get their parents arrested in modern times.

Expand full comment

I think this comes down a bit to how you are defining "career". Like, is a farm worker, a factory laborer, and a barman all different, or all subsets of low skill manual labor, which people used to do casually with no qualifications. Vs. different subsets of white collar work, or work vaguely related to computers. Is someone who trains in coding, then does project management, then sales, while still working in tech changing careers? Or a young person who works in a bar in college, does an internship at a charity, then becomes a junior corporate lawyer.

The point about greater freedom for children is true, but I think unrelated to career dynamics. If you are talking about a broad notion of, like, freedom, people change locations a lot more, and are less constrained by strict social norms in how they live their lives.

Expand full comment

You are focused on Steve McQueen's manual labour but I am also talking about other jobs that require degrees these days. I was an electronics engineer at 20 and a software engineer at 23.

Expand full comment

From your other comments, you did this in the Navy. I'm sure there's still a lot of 21 year olds in the Navy doing very technical and specialised jobs without a college degree, so that's definitely still an option.

On the other hand you're also talking about having freedom in your early 20s, and joining the Navy, while it may be rewarding in many other ways, isn't a great way to maximise your freedom.

Expand full comment

I can comment on the EE part: the job has drastically changed since the 80's, there's no way to become an EE without a rigorous course of study. The company I work for doesn't hire designers without a minimum of MS in electrical engineering. I have challenged my manager about this policy, and his response was, look, a BS degree just doesn't include the minimum course load required for the job, a 4-year graduate simply doesn't have the knowledge necessary to get going.

Expand full comment

If you have a chance to discuss this again with him, try to press for specifics. What courses have MS EEs consistenly taken that BSc EEs haven't? And why are those courses must-haves for work at your company? Graduate-level courses are routinely cross-listed as fourth-year undergraduate courses, so advanced undergraduates can take them if they want to.

I suspect your manager is engaging in snobbery and making excuses, or perhaps using a graduate degree for intellectual quality control, but I suppose it's possible there is something more going on.

Expand full comment

I did a four-year apprenticeship with one of those years doing on the job training on a warship. I expect I learned more than someone doing a four year degree. I expect the masters is required precisely BECAUSE the bachelor's degree does not cover the necessary materials. It doesn't have to be that way.

Expand full comment
Jun 10·edited Jun 10

The economy shifted away from manufacturing and towards services, where services require increased specialization compared with more manual labor?

What if, now, young men instead turn to creating and selling Fortnite skins or other digital goods?

It's an interesting question you pose. I don't know young men so I don't have anecdotal data. But I too am curious how the transition from child to adult is structured today.

Edit: Also just watched The Graduate (1967?) and it effectively portrays the school-to-career pipeline as a cage. So we're not the first ones to question this phenomenon.

Expand full comment

One word: plastics.

Expand full comment

The Graduate provides a good counter-example. Not everyone enjoyed their adventures before settling down, even in the 1960s.

I would suggest that most jobs in services don't require any specific knowledge that is taught in university except maybe on the science & technology side of the house. Financial services, software engineering, management, sales — these are all jobs where the necessary learning is acquired as you go along.

Expand full comment

I agree with you in principle. I entered the market way later than you, in 2013, without a degree, just a decent foundation of building websites.

One other thing that comes to mind is the increasing legibility of social life, ala Seeing Like a State. So if you're a kid and you don't get in the right path at the start, it will be increasing difficult for you to get a non-manual labor job down the road because your papers are not in order. Even tech is getting less porous than it was in 2013.

Now, what's driving that is a mystery to me. I suspect it's a few big factors and many small ones, which makes it more difficult to debug and figure out the main drivers.

My gut reaction is they I don't like it because it's a worse world for people like me. But I'm also wary of my perception being skewed. I don't hang out with younger people. I see a very narrow slice of the world, so I can't really judge if what I'm seeing is just "back on my day" stuff.

Expand full comment

I suspect that employers have no incentive to hire someone without a degree. For them it costs just the same to hire the person with the degree as to hire the one without. Why take the risk?

On the other side, maybe there has been an arms race between students that costs them a lot of money but benefits no one.

It was quite common to be hired without a degree “back in my day” in 30 years, I only got asked for it once and the hiring manager told HR, “we are gonna hire him whatever you say. “

Expand full comment

I will concur that for software specifically most learning happened on the job rather than in school.

Expand full comment

Well that’s a general statement without proof and two examples. People married young back then. So, for most, the living was leaving school, working in the local factory, and marrying in their early twenties. Of course for people who go on to be actors the trajectory is going to be different, but it speaks to no general trend.

Expand full comment

It certainly wasn't a universal practice to live a little before you settled down to get a job but plenty of people did it though. Navy at sixteen. Submariner at 21. Round the world at 22. Repairing satellite communication on oil tankers at 23. Director of Research in Silicon Valley at 30. I posted about it the other day. Lots of other examples there too.

https://raggedclown.substack.com/p/the-spirit-of-adventure

I've received lots of comments from folks my age saying that they had similar adventures. Back in my day, most people left school at sixteen and less than 10% went to university.

Something changed around the 80s/90s to require, for example, supermarket managers and nurses and bank managers to go to university when those jobs didn't previously require a degree. They just needed someone smart who could learn on the job.

I find it a mystery.

Expand full comment
Jun 10·edited Jun 10

I haven't read the Case against Education by Bryan Caplan, but I've seen his talks in which he summarizes the book, and that book has also been reviewed by someone on the slatestarcodex subreddit (slatestarcodex being the previous incarnation of this blog before it changed its name).

https://www.reddit.com/r/slatestarcodex/comments/7t59x1/review_bryan_caplans_the_case_against_education/

Bryan Caplan argues that the point of education is not to acquire skills, but to prove your worth to the system. Employers will always prefer those with the best education, regardless of the particular knowledge or skills required for a job. If more people go to university, logically employers will start requiring that people have gone to university.

If you're the only person who doesn't go to university in a world where nearly everyone does, you're telling employers that you're particularly dumb or lazy or rebellious, and employers don't want that, even for a basic job.

Whereas in a world where only a minority goes to university, as was once the case, not going to university will not be interpreted in the same way by employers.

Expand full comment

I'd go along with that. Its also a cheap and easy proxy (for the employer) for an employee's intelligence.

In times past, it was more acceptable to measure an employee's IQ directly.

Expand full comment

Maybe it's because HR departments now prefer people who don't have gaps in resume, and as much years of experience as possible? So you don't have luxury of mucking around unless you want to work low pay jobs for the rest of your life.

Expand full comment

That just begs the question of why HR departments don't allow gaps in resumes. Is it because most people who want to climb the greasy pole don't have gaps in their resumes?

Expand full comment

It doesn't beg the question, that means something else.

Anyway, I think it's an overplayed myth that employers are scared of gaps on resumes. They might _ask_ about it, but as long as you've got a good reason (I was backpacking around Asia) instead of a bad reason (I just couldn't get a job / I was in prison / I had a mental breakdown / I'm just so lazy and flighty that I can't be bothered doing a job for more than six months at a time) then it's not an issue.

Expand full comment

HR is scared of gaps in resumes because employees know that HR is scared of gaps in resumes and so they avoid them and the only people who would have gaps in resumes are the losers who couldn't keep a job — so HR are right to be scared of gaps in resumes.

Expand full comment

Gaps in resumes are seen as "didn't/couldn't get a job", "was fired/let go from job" and other bad things. If you have a lot of temporary work or moved around a lot, then the gaps can look like "isn't capable of getting a full-time job and/or is a bad worker who gets fired a lot" as much as "free spirit having adventures before settling down to the grind".

Why hire on someone you suspect will turn out to be incompetent or a trouble maker and have to fire them later?

Expand full comment

HR departments themselves are much more common than they were prior to the CRA, per Richard Hanania. https://www.astralcodexten.com/p/book-review-the-origins-of-woke

Expand full comment
founding

selection bias mostly.... but yeah, college enrollment is up.

Expand full comment

Suppose the alignment problem is unsolvable, even in theory. Doesn’t that prevent an AGI from making significant changes to itself because it, too, has to worry that the new thing it builds won’t be aligned with its goals?

Expand full comment

This assumes that the future AGI cares about “alignment” or its own survival or other human concepts.

Expand full comment

Alignment is multiple problems, value specification, value stability and also value fragility for specifically human values.

Alignment is currently not solved for any of these three sub problems, but even if an AI solves specification and stability, there's no reason to solve 3, so humans are still dead. On top of that, if an AI ends up having a much simpler goal than humans, the first two problems would be much easier to solve. Finally, even if alignment is unsolvable at human levels of R&D, at human time scales, that does not mean it's unsolvable at all levels of resource usage or cognitive ability.

Expand full comment

The question was specially about the case of “unsolvable even in theory.” Such as, the halting problem.

That might actually be a pretty good basis for a proof.

You can’t using a computer to solve the halting problem. “Will this new Turing complete system in going to make, kill me”, is just as hard as the halting problem and therefore unsolvable even in theory.

Expand full comment

What I hear in the parent comment is that it's plausible that, for example, value specification and value fragility turn out to be unsolvable but perhaps one can solve value stability, which is the only thing that matters for a self-modifying system; the other two aspects matter to us a lot, but wouldn't prevent an AGI to rewrite itself.

Expand full comment

My experience is that most people in this space are heavily biased by a naive assumption that the orthogonality thesis is true, and a deep aversion to having this questioned.

Yet even as it’s stated, it says that it’s clearly false in limited cases. Yet nobody is interested in fleshing out the details as to where it is and isn’t valid. Personally I think that survival and evolution over long time frames is an impossible problem to solve due to the entropic and chaotic nature of the universe: things fall apart and you can’t predict, eg the weather more than a few days out.

So think any AGI that considers infinite time frames would have to be extremely cautious because every change it makes to itself adds some small probability of spawning a child agent that decides to kill it, and this can’t be ruled out for certain - only probabilistically bound.

Expand full comment

Sorry, saying things like "have a deep aversion to having it questioned" without providing a reason makes me think the reason people are aversive is that your reasons are not great.

And also trying to relate this to the halting problem in the first place is not a great sign that you do have good reasons, considering that you can use that exact argument to say that game theory is completely pointless, because game theory will obviously be computed and this Turing complete. Or that aircraft safety is pointless because... and so on. The internet still exists even though the Byzantine generals problem, in its full generality is unsolved, we still use large software programs with lots of complexity and can expect, by default they won't hang most of the time.

A generalized form of a problem demanding optimality does not mean that there are no specialized solutions, or solutions that work given substantially more restrictions. That we don't have such an account of agents or goal specification should be worrying, in the same way that not having calculus should be worrying to people trying to build space ships.

Expand full comment

I gave you a reason, but you ignored it. I’ll give it again.

No one, not even AI researchers, believe in the orthogonality thesis without a list of caveats. An arbitrarily intelligent agent can’t have the goal of making itself dumber. Nor can it have the goal of destroying itself. Nor can it have a goal that is beyond its capacity to compute a utility function for. On top of this, every intelligent agent is believed to be likely to use convergent instrumental subgoals.

None of that is in dispute by AI safety researchers.

I claim the restrictions are much more important than is generally realized, and that convergent instrumental subgoals are ALSO a constraint on the orthogonality thesis. Based on this, I think the space of intelligent agents, navigating our entropic chaotic universe, is far more constrained than academics tend to think, because very few academics are responsible for keeping technical systems alive in said hostile environment, so they haven’t internalized how wildly entropic the physical universe is.

Your analogy to calculus fails because this entire enterprise is about reasoning in the limit of a process that goes exponentially. So of course theoretical limits matter there. Why wouldn’t they? General relativity doesn’t matter when designing roller coasters, but it sure does matter when building clocks for GPS satellites. The halting problem doesn’t matter when building a web app, but it certainly does if you’re a QA team lead and you’d like to drive defects down to zero. The halting problem was the reason bitcoin’s scripting language was designed, intentionally, not to be Turing complete. Language security researchers consider Turing complete input formats to be incredibly dangerous. So this thing you are saying doesn’t matter for AGI clearly matters in a number of cases, just not the academic domain of game theory.

To make this discussion more positive: would you meant telling me what evidence would change your mind here? The thing that would change my mind, and make me think an AGI were a much bigger danger, was if someone managed to build a machine that contains a general purpose digital computer and can also heal itself and reproduce itself just like other living organisms can. That’s the crux, for me: I think “keeping yourself alive in the physical universe ” requires much of what people imagine to be a weakness of biological life. We imagine machines to be more sturdy and capable than they are because most people aren’t responsible for keeping machines and technical systems from dying.

Expand full comment
Jun 12·edited Jun 12

> I gave you a reason, but you ignored it. I’ll give it again.

You gave a sketch of a reason, that doesn't have any details to them, nor any reasons for believing it nor answering why the former reason leads to the conclusion (the fact that the world is chaotic and this prevents X, without saying exactly what about X that makes it different from say, building a bridge or launching a rocket or inventing networking protocols, is not a great sign.) If you thought that the reason, as presented in your previous post constituted a sufficient argument (which I don't think you do), then I have no surprise why there's an aversion. This is not to say your later reasons are also unsatisfactory, but I felt that if I responded to your existing reasons, that is too much like attacking a straw man, or a position you would not endorse.

> Your analogy to calculus fails because this entire enterprise is about reasoning in the limit of a process that goes exponentially

How does this follow? Alignment is not just about doom, I have mentioned nothing about doom and do not consider doom a key and necessary component in my alignment-by-default worldview. So how can the existence of an unrelated to my point and stated sentence (I have not mentioned foom anywhere!)

Which is why your list of examples is even more bizarre to me. You haven't actually given a theorem on the level of Cantor's diagonialization argument, or Einstein's gravitational/acceleration reference frame equivalence, only expressed a hope that such a theorem exists elsewhere in the thread, yet you are confidentially talking as if such a theorem exists and has the exact details required to make those bounds. Yes, obviously if you had such a theorem you can make such generalizations and wouldnmt need to prove it to make your point but you in fact live in a world with no theorem of that level of power, which is my point about calculus. If I am wrong, link me to something on the level of the fundamental theorem of calculus that you think supports your point. It does not have to be proven, just exact enough to constrain without room for hiding miracles in the ambiguity of language.

> No one, not even AI researchers, believe in the orthogonality thesis without a list of caveats

The existence of caveats does not mean that a particular undiscovered caveat would be to your benefit.

> To make this discussion more positive: would you meant telling me what evidence would change your mind here?

1. Disproof of essentially everything in https://arxiv.org/pdf/1803.03453v1 but especially "Learning to play Dumb on the test" section on page 8. If it turns out extremely simple evolutionary systems can't stumble upon deception by accident, this would raise my threshold on how """natural""" deception is as a strategy. And lower my perceived difficulty of alignment. Dislroof here meaning something like "oh yeah it turns out running 10k RL algorithms, you only get those examples once. Or it turns out these specification gaming examples only affect these specific areas of RL, here are the following sub fields where no one does anything to defend against it and it's not a problem. And so on.

2. Known instances of genetic cooperation, as a ratio increase vs known instances of genetic defection. I.e. DNA parasites turn out to not be the supermajority of all genes, things like an "invention gene" which increases the fitness of all genes, but not itself turn out to be common. Group selection is gains a bunch more evidence relative to the selfish gene, and becomes the default paradigm by which evolutionary omits use to analyze selection. If you were correct that the chaotic and uncertain nature of the universe leads to something like cooperation and alignment, then we should obviously see traces of this in our genes. That we don't suggests that line of thought is not true on the merits.

3. Actual evidence that demonstrates healing and reproduction causes alignment in some way, I don't know what this would look like physically, because it sounds like positive sounding abstract concepts smooshed together to sound good. But if you can provide non-post hoc evidence or make a prediction with this theory that can cash out on relatively short timeframes (or try and postdict something about the past, in an area you're not familiar with) and succeed, then I'll pay attention. Alignment as a framework makes specification gaming entirely unsurprising, and a world in which it's a functional concept is a world where things like the selfish gene, institutional capture and so on pop up with some regularity.

Edit: changed the word proof to theorem.

Expand full comment

I want to acknowledge that I misread your question, and provided some irrelevant answers.

However, even if it is unsolvable in theory, details matter!

The halting problem is unsolvable, but that doesn't mean distributed computing is literally impossible to do, because you don't know if a particular job would terminate. You can rephrase all infinite Turing machine problems to ones with bounded ones, which, yeah it doesn't answer things in terms of the most-easily-analyzed-via-proof formulation, but does have practical utility to us!

So details of the bounds on alignment would be relevant to what concrete actions would be limited.

I don't know why it would be useful to speculate on this, since most speculation on alignment is hopelessly polluted by wishful thinking about how human morality is the most objectively correct one.

Expand full comment

Isn't this a broader problem with learning from experience?

I might learn something that altered my opinion about something important. I might have an insight that leads me to think about a problem in an entirely different way. From the standpoint of my old self, are those new selves aligned with the old self? Maybe they still adhere to some sort of meta-value, but what if that meta-value changes?

Have you ever had an experience that you believe made you into a person that your old self would view as "worse"?

Expand full comment

Yes, definitely. I think learning is a form of evolution. The old self has to die for the new knowledge to take hold. When we aren’t attached to “the me that cannot ride a bicycle”, it’s easy. This is much harder if it’s, eg, “the me that knows the outgroup is bad and the ingroup is good.”

Expand full comment

One of the reasons alignment is hard is because it's so broad. Even if you could make an AI do what you tell it, it's hard to tell it what you really want. Not all AI's that could doom us have this problem, because their goals could be much simpler, and easier to control.

Expand full comment

I wouldn't say it prevents it, because it can still do those changes, but it just has no guarantee that the resulting AI will be aligned with it's goals. It would be akin to creating a child rather then a self-improvement.

Expand full comment

Whos unsolvable? n=np and cyptro? "we dont actaully know in 3n+1 halts"? 3 body problem when you can just do math with 6 vectors with 64 bits? or actual practical approximations?

a smart gun is an aligned machine, your immune system is often enough aligned with your bodies goals, etc.

Expand full comment

Could pinball be a "solved game," meaning a machine could play a particular pinball game forever without the ball ever going into one of the drains, while also scoring the maximum amount of points over N period of time without a risk that any play would send the ball into a drain?

Expand full comment

In machine learning deterministic single-player games are not used because they are too trivial. They are deliberately distorted by noise. For example, for the famous Atari games benchmark, each input is randomly replaced with some other input with some small probability. (10% or so?) That means that sooner or later it's game over due to bad luck.

If you don't add noise, then it's easy to keep playing forever. Whether the optimal strategy for getting the maximum amount of points can be found, I assume that this depends a lot on the exact pinball version that you use. For the very basic ones, I would guess yes, but it gets hard pretty quickly as you add long-term effects.

Expand full comment

Mechanical pinball machines have solenoid activated bumpers that don’t behave in perfectly predictable ways, so playing forever seems unlikely.

There is currently a system that can solve a Rubik Cube in 1/3 of a second though.

https://arstechnica.com/culture/2024/06/robot-solves-rubiks-cube-in-literal-blink-of-an-eye-for-new-world-record/

Expand full comment

The machine would have the ability to create very detailed computer simulations of each pinball machine it played with, including of the bumpers. As the game progressed and it saw how the bumpers reacted to ball strikes, its model of how the solenoids worked would get more detailed.

Expand full comment

They could play better than a human if they were sophisticated enough but I don’t think they could go on indefinitely never losing a ball. The solenoids themselves are crude devices that don’t activate with exactly the same force each time and the spring loaded actuators would stretch and wear so they would behave differently between individual rebounds.

Expand full comment

Assuming there's no wear and tear:

Assuming you can play perfectly and that the unpredictability of physics are not a factor:

Yes, at any one time there is an ideal path to maximize your score within an N period of time. I don't see how that could not be the case.

But the problem is that it's not really possible (for a human) to play perfectly all the time, nor can the physics be predicted perfectly (by a human).

PS:

Even if you made a robot with perfect aim, that could always make ramp shots, and always regain control of the ball from easier-to-predict returns like orbit shots, it still wouldn't score as highly (within N time) as a robot who can make unsafe shots on targets and then regain control afterwards. On most machines (or tables, as they are sometimes called). But that would be a lot more difficult, I think.

But it would still be worth 10,000 dollars of ad revenue or something to build a computer that can play a physical pinball machine very well. You could get a long ways with a computer that's perfected aim + basic ball control skills, even if it couldn't nudge.

But then you get to multiball and the computer would need a more advanced set of skills for that...

Expand full comment

The player would make the critical difference, as success hinges on one's ability to bump the table with one's hip at just the right moments -- to change the trajectory of the ball. Someone more skilled at bumping the machine scores higher, while a programmed 'bot doesn't.

Expand full comment

It might do okay if it were deaf, dumb and blind and played by sense of smell. ;)

Expand full comment

Thank you for that earworm! :-)

Expand full comment

In a trivial sense, if you are using a physical machine you eventually have parts wear out and break. More generally, physical systems are inherently somewhat chaotic, eventually something weird will happen with the way the ball moves that is unexpected

Expand full comment

I would imagine this depends on the specific machine. This might be doable for some machines but not others.

Expand full comment

In Russia, there is perception that modern West society is built on "snitching culture", in that ordinary citizen report each other to their bosses (e.g. for harassment, or breaking safety rules a bit), or to government (e.g. for improper parking) constantly. People who espouse this view contrast it with "no cooperation with any authority, we sort stuff out among ourselves" approach that was widespread in late Soviet period and in 90's, born out of complete distrust toward any official institutions, prison culture and "us against them" mentality in a lot of workplaces.

On the other hand, a lot of people from later generations view at least some kinds of "snitching" as positive. For example, any social media post about reporting improper parking to police usually generates a wave of outraged and supportive comments (the distribution depends on what corner of internet you find yourself in), with supporters of the clamoring for better tools to clean up streets, and opponents writing things like "so, like you never break the law?" and "snitches get stitches". In apartment buildings, if you get a neighbor car towed for e.g. blocking a pedestrian path (a common occurrence in Russian cities, where cars far outnumber parking spaces), it's not unusual to find outraged threats from the owner of the car posted on communal board, with calls to other tenants to "help find the rat".

So, my question is - what is the actual general attitude toward such reporting in various Western countries? I guess in reality it varies a lot between different regions (Southern Europe is closer to Eastern Europe in a lot of ways, like disregard for traffic laws - yes, Sicilians, I mean you, - and I would guess this would be one of them, too).

Expand full comment
founding

"Snitching" is still generally regarded as a derogatory term in American English, so I think you and/or your Russian sources are overstating things with "snitching culture". It is, around here, the exception rather than the norm (and, being exceptional, overemphasized in the reporting when it does occur).

But it's probably less exceptional than in Russia.

The United States is, for the most part, still a high-trust society. Russia clearly isn't. Since it really really sucks being a lone wolf in a low-trust society, there's going to be a strong ethic of fitting in and getting along with whatever tribe you wind up in, in a way that snitching to outsiders very much isn't. People divided up into small mutually-distrustful groups, are less likely to "snitch" than they otherwise would. But in a high-trust society, reporting offenses to a trusted authority is more likely to achieve useful results, and it's relatively safe to live as an atomic individual while you find a new tribe if you wind up being ostracized from the old one.

More importantly, in a high-trust society, the sort of offenses that result in "snitching" are not positively regarded by others of that culture They mostly involve dishonest and untrustworthy behavior. And the barrier to snitching is reduced by the contempt one feels for someone dishonest and untrustworthy.

Expand full comment

It's very interesting to hear the stereotypes that Russians have of the West, I never would have guessed that one.

Australian attitudes towards "dobbing" are complicated. The school playground is very different to adult life. The median person's attitude would probably be that they'd have no hesitation in calling the authorities on someone who harms or seriously inconveniences them personally, or is behaving in a way that is clearly terrible or dangerous to someone else, but someone who goes around reporting minor rule-breakers that don't concern them personally is probably an annoying busybody. In actual practice I've never reported anybody who didn't do anything to me personally. If I see someone, say, fare-evading on the train then I'll feel aggrieved and hope that an inspector jumps out of the bushes to catch them, but I'm not going to take time out of my own day to try to do something about it on my own.

Dobbing became controversial during covid, because opinions differed on whether a covid rule breaker was doing "something dangerous that concerns me personally" or not. (Recall that we mostly had an elimination strategy so a single super-spreader could fuck things up for a long time)

Expand full comment

I think Aussies like to think we're not snitches but if someone asks about some kind of behaviour on a lot of the r/city Reddit subs, the top voted comment is almost always "report it" (unless it's genuinely trivial).

Compared internationally, Aussies have really high trust in public institutions (not a bad thing). It's a notorious nanny state, but a popular one (Covid suppression strategy wouldn't have worked if we didn't have a popular nanny state - the laws without the culture would have done basically nothing)

Expand full comment

In many latin american countries, there's a general vibe that the government, institutions and companies are "out to get us" and thus rule following is for suckers. However, personal generosity and helping each other out is seen as good. This inevitably leads to a low-trust society, as the equilibrium ends up being stuff like 30%+ fare evasion of public transport while everyone complains about how shitty the public transport is and how companies are ripping us all off. This is of course a self-fulfilling prophecy.

Expand full comment

Cui bono? To whose benefit?

This is a culture optimized for criminals, abusers, gangs, and mafia. People supporting it are either in those groups, want to be in those groups, think they benefit from those groups, or live in active fear of those groups.

Consider for a moment that the entire "MeToo" movement is composed of snitching rats.

I bet within 5 minutes any of us could convince any of 90% of the members of "anti-snitching" culture that, no, they do in fact have an interest in enforcing laws.

Expand full comment

I’m thinking the case of the ‘stick up his ass’ dorm RA who shuts down any sign of students having a bit of fun. In a case like that I would be anti snitch.

Expand full comment

Yeah, there's an implicit judgment about the value of the rules, which is simply assumed as a silent part of the American system. Many parts break down and become miserable if the rules are always enforced or never enforced.

But when stated as a general principle, hell no. It's the mark of a villain or a sucker.

Expand full comment

Ruminating about omerta led to thinking about my mother’s father an Ellis Island Italian immigrant. His countrymen made a significant minority in my home town. The guy kept a snub nosed, nickel plated, 38 revolver in his dresser for ‘hunting rabbits’.

As a little boy he bought me around to the Italian barber, shoe repair guy and sports ticket takers. “This is my grandson. He doesn’t have to pay.” Yikes.

Expand full comment

Wow. Also...

> 'hunting rabbits’

I'm ... used to that as a medieval euphemism for cunnilingus. o_O

Expand full comment
Jun 10·edited Jun 10

This kind of reminds me of watching "American Factory," where they show Chinese management talking to Chinese factory workers about how lazy their American counterparts are. You don't tell your in-group that the out-group is getting 5 day work-weeks while you're stuck with one day off per month, you tell your in-group that the out-group is lazy, and lacks the in-group's rough, manly determination and tolerance for hard work.

Similarly, you don't tell your in-group that the out-group has better and more trustworthy institutions that are less likely to demand bribes or otherwise exploit their authority when called for assistance, you tell your in-group that the out-group is a bunch of cowardly snitches, and lacks the in-group's tough, manly ability to handle matters between themselves.

Expand full comment

In this case, my feeling is that anti-snitching is more of grassroot movement than something government-imposed, and indeed, government constantly creates new tools for reporting stuff (although the long-awaited app to report dangerous driving still isn't released to public, as far as I'm aware).

Expand full comment

But that's the thing - it doesn't need to be government-imposed propaganda to arise as a narrative.

When one group of children go to an elite private academy and have access to all sorts of resources, while another group of children go to a poorly funded public school, there's no authority figure *explaining* to the kids at the poor school that they should start to believe that the rich kids are all soft and spoiled and could never beat the average poor kid in a fist-fight, but the belief will arise all the same. They don't need an adult or teacher to push that narrative onto them - we all love our communities, warts and all, and find ways to make our vices into virtues accordingly.

Expand full comment

Take a look at this stackexchange academic post: https://academia.stackexchange.com/questions/195031/how-is-cheating-viewed-in-western-universities-and-why-is-it-seen-as-such-a-big

The title is "How is cheating viewed in Western universities and why is it seen as such a big deal?" Perhaps it speaks to my (western) culture that it took me a bit of time to realize this wasn't a troll.

> I spend a lot of my time in the American infosphere and one thing that always baffled me is the apparent attitude towards cheating, plagiarism etc. It seems that everyone (including the students) see it as immoral or the like. Here in Russia cheating is a normal part of student life. It is seen as a way of bolstering your grade/passing a class and is only bad in the sense that you might get caught. And you are not a thief until you are caught...

> My question is why it seems to be such a big deal in the West (and in America in particular)? I have seen movies where students buying course papers from each other is a major plot point and is presented as something horrible and not as something totally mundane. Would people in the US be comfortable telling stories about how they managed to cheat on exams to their bosses during small talk?

etc.

Now quoting the second answer:

> There is a profound cultural and factual difference in understanding the society between the rule-of-law democratic countries ("the West" in the Russian parlance) on one side and the ex-USSR space on the other side. In "the West" law, order and various rules are seen to be: 1. beneficial for the society as a whole 2. beneficial for the society members (people breaking the rules may see short term 3. advantage, but long term resistance as well) 3. subject to change, should a general consensus of changing them is established 4. livable in the first place

5. generally enforced 6. usually take precedence over what the boss says [...] "In Soviet Russia" everything is the other way round. 1. Rules are from the above. 2. Even if they change, the average Ivan has no business thereof. 3. It is generally hard to follow the rules. They are hardly ever evaluated for efficiency or acceptability (no feedback so no one cares). 4. Rules are enforced sporadically and selectively. 5. The best-known social elevator is breaking the rules in one way or another (corruption, stealing, etc...). People doing this precisely are seen as successful and see approval instead of resistance. 6.The society well-being is a ruler's business, not everyone's.

And later:

> In regard to the integrity and "keeping your word": In "the West", keeping your word is keeping your word. There is no profound difference between cheating your wife, cheating on exam, cheating in your tax form or stealing an item from the neighbor. On the other side of the iron curtain, cheating "the system" (be it your boss, the government or the grocery store) is really, really morally different from doing the same to people equal to you (friends, family, direct contractual partners or even an unknown person on the street). The former is acceptable and sometimes approved, the latter is not.

Expand full comment

This post basically expands on my points about "why" of anti-reporting sentiment in Russia, but I feel it goes a bit too far in direction of painting Western countries as citadel of law - from what I read on Internet, cheating on taxes is a national sport, at least in America (I would guess Red states with their "fuck big government" mentality have wider spread of low-level cheating? I mean, companies and billionaires "optimize" their taxes no matter what's their political views - or nationality - so it's more meaningful to look at what a poorer person tries to get away with).

However, this is a bit tangential to what I ask: cheating at anything is one thing, but reporting other people is another. For example, personally, I never cheated on exams, but I also never ever considered reporting another student who I saw using a cheat sheet, or who asked to copy my work. On the other hand, I feel if I had a way to report every damn bastard with a loud muffler or a bass booster which makes surrounding buildings rumble, I think I would do it.

Expand full comment

Because ~no one *cheats* on their taxes. Everyone is playing by the rules - and trying to optimize them towards their own ends. Hence finding "loopholes" is seen as neutral, even good - treating the rules as holy means exploits are fair game.

Expand full comment

Huh? Sure they do.

Two very common examples of low-level cheating are not claiming cash income and claiming regular personal purchases as business expenses. There are many others.

Expand full comment

Yeah, those seem like things that happen sometimes but not things that "everyone" is doing.

In particular the practice of "cash only" businesses that try to cheat on taxes seems to be much more common among immigrant communities (especially Chinese) than native Westerners.

Expand full comment

Huh, I always assumed cash only stores were due to people not wanting to pay interchange fees and being too financially illiterate to realize they're losing a lot more due to lost sales from friction and from the labor required to handle petty change.

But the one cash-only store I know of near me was a chinese bakery, so maybe it's what you say.

Expand full comment

I know plenty of native Westerners who don't claim cash income and claim ordinal expenses as business. And the fact that I know this about them is in itself a testament that they don't consider it shameful and believe their social circle doesn't strongly disapprove of such behaviour.

Indeed, the prevalent sentiment in my social circle is that we're being taxed to death with little to show for it in services that are getting increasingly more sparse and difficult to access. I live in Canada, btw, if it matters.

Expand full comment

We've established that you don't live in Philadelphia, where the ethnic mix for cash-only restaurants seems to be pretty even.

I'm actually fascinated that there are cash-only businesses, card-only businesses-- sometimes a specific electronic method of payment, and businesses which take both cash and cards, some of which have a small fee for paying by card. I would have expected a consensus for payment methods, but no, or at least not yet.

Expand full comment

I can speak more to the moral question than the factual one. While I don't have a strong opinion on snitching either way, I think one of the main motivations or justifications for it is the sheer unfairness of:

"9 out of 10 people who break the law, even to the maximum extent and with obnoxious self-awareness, face no punishment

1 in 10 people who break it, even accidentally in the most tiny technical way, get punished because there was an inspector/officer/etc around"

that you get when it's only the authorities doing the enforcement. Authority figures are much more constrained to act according to the letter of the law, while ordinary "snitchers" have much more discretion to tailor their snitching to things like how deliberate the law-breaking was, how far the law is being broken (e.g. the five minutes vs whole day parking issue Michael Roe mentioned), and above all how much of an unrepentent obnoxious asshole the law-breaker is being about it.

In this sense snitching looks like a clear positive: the result is much greater likihood of the worst law-breakers being punished, plus much greater difference between the fate of obnoxious law-breakers and careful or accidental ones. Another positive about snitching (for everyone) s that by increasing the enforcement of existing laws you make the enactment of more and tougher laws (which is the main way the state responds when existing laws are being often ignored) less likely. It seems pretty clear to me that "enforce existing laws as much as possible" is strictly better than "punish caught infringers as harshly as possible" which in turn is strictly better than "make as many things illegal as possible". I'd be interested to know if anyone disagrees with that.

Expand full comment

>while ordinary "snitchers" have much more discretion to tailor their snitching to things like how deliberate the law-breaking was, how far the law is being broken (e.g. the five minutes vs whole day parking issue Michael Roe mentioned), and above all how much of an unrepentent obnoxious asshole the law-breaker is being about it.

A lot of what I've observed in the US about favor/disfavor of snitching seems to hinge on this part in particular. There are social norms around what is and isn't appropriate to report to the authorities, and those who violate those norms in either direction are seen as doing a bad thing. Reporting offenses is generally more likely to be seen as tolerable or even obligatory when the offense is more serious, for clear cut and deliberate rather than technical or accidental, when the rule being violated is seen as reasonable, when the authorities being reported to are seen as likely to handle the report appropriately, when the offense has a clear and innocent victim, and when the reporter is the victim or at least seems to be motivated more by sympathy for the victim than by malice towards the perpetrator.

Expand full comment

As a British person ... generally, you will get reported if you are actually causing a problem for someone, but probably not if you're just technically breaking a rule.

In the case of parking violations: if you're illegally parked in a way that blocks traffic, or prevents someone getting their car into their driveway, then yes, you're getting reported and towed.

On the other hand ... in the street where I live, if i get a contractor (plumber, electrician, builder etc) to come and do work on my house, then I am supposed to give them a permit to put in their vehicles's windshield so they can park it. I do in fact do this, at least if their vehicle is going to be parked for more than about 10 minutes.

It is very unlikely anyone would report a contractor who just unloaded a vehicle for 5 minutes without having a permit attached, or indeed if they spent a day working on a property without putting a permit of their vehicle There are, however, government parking enforcement officers around, so you would probably get ticketed if you were there all day/

There is a certain amount of "if its only going take 5 minutes to unload this, and I cant actually see any parking enforcement officers around, then just do it and try and make some kind of excuse if i'm unlucky and an enforcement officer shows up".

Expand full comment

Yes, I feel that similar to "jury nullification", there's a form of "reporting nullification", where most people don't report violation of stupid rules.

Expand full comment

I think this is fairly accurate division. In my experience people draw the bar fairly high for causing a problem - they take into account whether the likely punishment is proportionate. For example, I have neighbours who probably comprise an unlicenced HMO (house of multiple occupation, which needs a different licence to a standard rental). They are a bit noisy and sometimes put overflow rubbish in my bin. But I don't consider that evicting them would be proportionate (and it's also technically the landlord who is at fault). I have friends whose neighbours have an unlicenced "dark kitchen" (food delivery business with no storefront). They haven't reported them even though there's smoke going into their home. For me I would seriously consider doing so because I don't think it's healthy or safe. But again, taking away someone's home and livelihood is a pretty severe punishment.

Expand full comment

Living in an unlicensed HMO isn't illegal, only letting it is. If a landlord is found to be running an unlicensed HMO, they even lose the right to a no-fault eviction (presumably to prevent them from retaliating against the tenants for reporting it).

A while ago my housemates and I (in England) had reason to suspect that our house's HMO license was fake. I was surprised that one of my housemates was strongly opposed to reporting this to the council. I would understand not wanting to report an unlicensed HMO where the tenants were happy to live there anyway, but in this case it seemed that the agents were deliberately lying to us about it which seemed to me a whole lot more malicious and deserving of punishment. (It later turned out there was a more reasonable explanation, but had we gone ahead with reporting the issue, the council would presumably have just told us this.)

Expand full comment
Jun 10·edited Jun 10

"I was surprised that one of my housemates was strongly opposed to reporting this to the council."

Often times it's because "better a shitty landlord than no landlord"; if it was indeed fake and the letting was stopped, then the tenants mightn't have anywhere else to go. Particularly if you're a student in the middle of the academic year and it's difficult enough to find accommodation willing to take students.

In Ireland, anyway, there are (allegedly) an awful lot of landlords who flout rules about such things as "you are supposed to issue a rent book to the tenant and record payments" and tenants are worried that if they complain, they will get kicked out and the market is sufficiently tight that it's nearly impossible to find another place to rent.

https://districtmagazine.ie/news/irish-twitters-most-cursed-landlord-stories/

There are terrible tenants as well, of course, but there's a lot of landlords who aren't declaring their rental income for tax purposes, don't maintain properties, and more or less tell potential tenants "take it or leave it" because they know desperation to find any kind of accommodation will make some people put up with it.

https://www.rte.ie/news/business/2023/0304/1360043-explainer-why-are-private-landlords-selling-up/

"But why are landlords selling, when both demand and rents are so high?

Increased regulation, high taxes and tricky tenants are top of the list.

High property prices have also made the idea of selling more appealing for some.

Mr McDonald pointed out that many home owners are 'accidental landlords'.

"The property crash brought about a situation whereby the value of the house was so low vis-à-vis the mortgage on the property, owners had to hold on to the property and rent it out.

"Now that we have seen house values increase over the past five years, the value of the house has gotten to a level that the owner can dispose of the property and pay off their mortgage," he explained."

Expand full comment

Living in an unlicensed HMO is indeed not illegal. But often councils will refuse to licence an unlicenced HMO, leaving the landlord with no option to avoid fines other than by terminating the tenancies.

Expand full comment

Hm. What about people who don't obstruct anything, but at a cost to public property? E.g. when someone parks on grass because otherwise he would have to park a few blocks away. I guess it might be somewhat uniquely Russian problem in some ways, though, because we live in huge apartment buildings with somewhat spacious green spaces around them, and not enough parking spaces, so there is a lot of temptation (a particularly bad example: https://teletype.in/files/71/b9/71b98d6e-f914-4e8b-b001-a18e198d7175.jpeg)

Expand full comment

Here in Texas, we don't have much grass; but the general attitude would be "fuck you, you goddamn snitch! Man, if these cops weren't here right now...", heh.

It's considered a shitty thing to do and kind of unmanly/weak/slimy. Obviously, this doesn't apply if someone is doing something endangering others or materially harming /your own or your neighbor's/ property.

Else, mind your business, basically.

When I traveled to Germany I had a fairly lackadaisical attitude to a lot of rule-breaking; my girlfriend would get quite nervous about it, and insist "no, people here are different, they WILL phone it in!" (e.g. about parking in an empty but reserved spot for five minutes so I could run in and grab our food, say; or cutting across, on foot, an area that had been cordoned off; or smoking a vape in a no-vaping area; etc. etc.)

...no one ever did, AFAIK, though. Maybe it was just her, heh.

Expand full comment

> ...no one ever did, AFAIK, though. Maybe it was just her, heh.

Germans have lots of respect for authorities (also trust and fear). e.g. "if I am not the perfect citizen, the allmighty state will come and crush me (and justly so)".

Source: I am german, who started to notice cultural differences after moving abroad

Expand full comment

Thank you for confirming one Russian's stereotypes about Texans :)

I've been to Germany two times, and I was always amazed that it never seemed as lawful and clean it should have been according to my mental picture formed by jokes and anecdotal evidence. People crossed roads on red light. People littered. People drew ugly graffiti everywhere. Just like home, you know. When I asked why the reality is so different from the legend of German law-abiding, people usually told me I was in the "wrong" part of the country. "Munich? That's not REALLY Germany!". "Schwerin? Oh, it's not THAT Germany". So far, the "true" Germany where they all march to the law's beat remains elusive.

Expand full comment

Not in my street, but I got to hear this story from the person who reported the guy...

So, this guy has commercial waste from his business in a dumpster (a) in residential area; *(b) on a garage driveway that he is illegally subletting from a guy who is renting it from the council; (c) improperly secured, so that the building waste is blowing all over the street, and on to peoples cars.

People are concerned that this might damage their cars, and ask the guy to at least secure the dumpster full of commercial waste so that it isnt blowing about. He declines to do this.

Ok, time to take some photographs documenting this situation and send a complaint to the council. It turns, out, the council was already aware of this situation but has a policy of not taking enforcement action unless someone actually complains. Well, now someone has complained and that guy is in a ton of trouble for breaking a bunch of rules.

Expand full comment

And on the subject of safety regulations sometimes becoming a problem...

Painter is about to repaint my office at work

In order to avoid build up of solvent fumes, it would be good to leave the door open.

But ... the office door is a fire door.

Me: "There is actually a solution to this problem that building management is trying to tell me is intractable. You can get automatic devices that hold the door open and release it if the fire alarm is triggered. New proposal for building refurbishment: fit automated door closing devices to fire doors, then the painter can work, and if we;re so unlucky that there's a fire while the building is being repainted, its still ok. Justify the additional cost on the basis its needed for compliance with fire regulations."

Expand full comment

My friend (joking) "Or the painter could just prop open the fire door with a container of white spirit."

Expand full comment

He really should be smoking a cig too to round the humorous example out.

Expand full comment

Indeed.

And, sometimes, it does matter.

1. Building fire alarm trips

2. I evacuate to the assembly point

3. Fire crew shows up (yep. this is not a drill)

4. Fire crew asks me, would you happen to know if this is a real fire?

5. Me: the alarm panel says the alarm trip.was in the lab where we have a cutting laser. If there was going to be a real fire anywhere, it would probably be there. But no: I have no definite info on whether this is a real fire.

(You can get false alarms from cutting lasers, of course. if e.g. the specimen being cut generates much smoke, but its a controlled and contained sort of fire.

Also: if you're cutting lots of acrylic, remember to regularly clean out all the little bits of offcut acrylic from the laser cutter bed. Otherwise, there's a thing that can happe where a;;l the little bits of acrylic catch fire/

Expand full comment

You may want to tease at least the first couple paragraphs of some subscriber posts. Teasers like that are what got me to finally actually subscribe to other writers here.

Expand full comment

I assume it's deliberate, given Scott said he made efforts to avoid the subscriber-only posts being intrusive.

Expand full comment

Posted this in the paid open thread just now, might as well go here:

This post on monopolies (and the entire substack) feels relevant to the old SCC discussion on cost disease:

https://open.substack.com/pub/mattstoller/p/economic-termites-are-everywhere

Expand full comment
Jun 10·edited Jun 10

Refugees are people and not victims, and this is a weird job ad not a normal one.

In Second Tree, people have always grown in the team, building skills within the organisation. It was like this when we started as four volunteers, and it is now that we are a team of 20-25 – many having been here for years – managing several projects in refugee camps in Greece and training institutions and organisations in more than 20 countries.

BUT we have been awarded the project that we’ve always dreamed of. In it we have put all our ideas on how to change the humanitarian world and refugee reception systems (yes: a very unambitious project!). So, we thought: we will need more people, with a large array of skills; people who share our objective and ethical commitment and, maybe, feel unhappy with what they are doing now?

We can offer an enormous amount of work (but for something really meaningful) for a very small salary (but the same as the CEO and all of top management).

So, if you are a disillusioned humanitarian, who is looking for a place that really cares; or you work in another sector and want to start helping, this might be for you.

To read more, follow this link:

docs.google.com/document/d/1aav_rMyTyCatrdAMwPlkviBk2_73fuGP5cRMc2nWd0o/edit?usp=sharing

Expand full comment

If I were redesigning the world's refugee reception systems then I'd do it in a way that fairly distributes refugees to countries of a similar level of economic development, to ensure that people can find refuge if needed but that they can't abuse the refugee system to move to a richer country.

Expand full comment

So you'd socialise the cost across the poorest countries, and deny the benefits of migration to the wealthier countries (with declining populations)? Seems like a lose-lose.

Why is taking refuge in a wealthy country 'unfair'?

Expand full comment

It's unfair not to the wealthy country, but to the genuine refugees. Actual refugees fleeing religious persecution or attempted ethnic genocide have to face a grueling process to prove their claims are genuine (and a good chance they'll be judged unconvincing and be deported). Why? Because huge, huge numbers of economic migrants brazenly lie and fabricate experiences of persecution in order to get into a wealthy country.

This situation is the case in Australia, at least, and is uncontroversial among anyone who knows someone who has worked in the Home Affairs Department. To take just one example, Chinese "refugees" claim to have converted to Christianity and been arrested or threatened by the state, but when asked to describe the beliefs of Christianity they can't name a single one. These of course are the bad liars; the better ones are harder to discover, and bring enormous scrutiny on the truth tellers.

I don't know the solution, but the first step is to be very, very clear that economic migrants (however sympathetic they may be) are not refugees, and if they claim to be are deliberately exploiting a system designed for people much worse off than themselves.

Expand full comment

Under this system the USA wouldn't exist

Expand full comment

How so?

Expand full comment

I say you need a token team-member who is always saying things like "pfft, bunch of criminals if you ask me" and "I bet they're gonna just take all the money and buy drugs" and "hey, do you think they actually do know where to get drugs?", etc. .

Keeps your feet on the ground, y'know? (Now, I AM available — but I warn you: hateful cynicism of this caliber doesn't come cheap.)

Expand full comment

We already have someone like that: “high trust models will inevitably be exploited…”

In the end, everyone succumbs to incentives. If people are given enough reason to assume the responsibilities of a member of the community, they will.

Expand full comment

If any of you guys live in NYC and are interested in learning about computational biology, I'm teaching a free, in-person seminar in Brooklyn the weekend of July 13-14. The only prerequisite is basic Python (no biology background needed). This is part of Fractal University, a very cool community skill-sharing group that offers free/cheap classes on a lot of different subjects.

Here's the course page, with more info:

https://docs.google.com/document/d/1rAbwUbOVYINCVnOd8WlAI7e3Qp5X0ZmPjBy7O7zHXec/edit?pli=1

Here's the link to sign up (don't worry about the May 17 deadline, that's for the other classes):

https://airtable.com/appqj7FQhKgCdLnWM/pagJsr7xx8XjWkcLp/form

There are only like 5 slots still open (might be more if some people cancel), so technically you need to "apply" and admission will be on a first-come-first-serve basis.

Expand full comment
author

1DaySooner on contacting your representative (see item #1 above):

Bill SB 1308, currently in the California General Assembly, might inadvertently ban far-UV as a side effect of trying to ban ozone-producing electronic air cleaners like ionizers. The bill limits ozone production of an air cleaner to 5 parts per billion—opening a window would fail this standard, and current far-UV lamps produce a bit more than that limit. The technical report that the bill is based on explicitly states that the chosen ozone limit was designed for worst-case ventilation scenarios, not the spaces where we spend the vast majority of our time. Given California's size and influence, this bill risks destroying a burgeoning market and preventing further far-UV research and development, but a simple amendment exempting far-UV from the bill could address the problem.

If you are a California voter, you may be able to help by contacting your elected state representative. The bill is currently in the Committee on Natural Resources in the California General Assembly, due for a committee hearing on Monday June 17th (in one week). Here is the committee membership list; among this blog's readership, the most likely relevant committee member is Buffy Wicks of Berkeley. You can call her Capitol office at (916) 319-2014 or her district office at (510) 286-1400, or submit a written comment.

(If your Assembly member is not in the Committee on Natural Resources, you can still call/write to them, but it would be better to wait until the bill is out of committee, ie after June 17th—most of the members won't even be aware that this bill exists yet.)

The key things to concisely convey when contacting your representative are:

1) you are an informed and engaged voter,

2) you respect the representative and share the committee's ultimate goal of improving public health,

3) ensuring that far-UV devices can be sold in CA supports this goal, and

4) ensuring that far-UV devices can be sold in CA supports and is in keeping with the representative's own policies and platform.

Suggested call/email script:

[If calling, begin with: Hi, I'm a constituent in Assemblymember Wicks' district, and I'd like to comment in opposition on bill SB 1308, currently in the Committee on Natural Resources.]

It's great that this bill aims to support Californians' respiratory health, but I'm concerned that by limiting ozone emissions so strictly, it risks accidentally banning a technology called far-UV light. This technology shows immense promise for fighting diseases like COVID and flu, and its potential benefits are much higher than those of the electronic air filters that this bill primarily targets, while its typical ozone emissions are just about 5 ppb, the cutoff specified in the bill. This ozone limit was designed for worst-case ventilation scenarios, not the spaces where we spend the vast majority of our time. It will be better for public health if Californians can access far-UV. For these reasons, I would oppose this bill unless it is amended to exempt far-UV.

[If you generally like Assemblymember Wicks, it's good to conclude with a compliment for her policies/care for her constituents/promotion of public health at this point.]

Expand full comment
founding

Looks like the bill failed. Hooray for far-UV, boohoo for asthma.

Expand full comment
author
Jun 25·edited Jun 25Author

Can you tell me more? I can't find any information about this. I don't even think we wanted the bill to fail, just to be amended - do you know what made it fail?

Expand full comment
founding

I heard from a lobbyist that the bill has been dropped.

https://leginfo.legislature.ca.gov/faces/billHistoryClient.xhtml?bill_id=202320240SB1308

Shows that the author canceled the last 2 hearings, so it seems like they are withdrawing the bill. No idea what happened, 1DaySooner might have more info.

Expand full comment

(I'm the 1Day Sooner point person on this.) The author did withdraw the bill for this session, which doesn't mean it's permanently dead--she could reintroduce it next year, which would give her office more time to understand the arguments for far-UV and more time for far-UV proponents to talk to her and to the regulator. Our coalition was indeed only advocating for an amendment, not failure, since reducing the use of portable ionizers and ozone generators is still a good idea. I think withdrawing the bill for now is a really good move on the author's part; I hope that we get the time to discuss this specific issue in depth and eventually wind up with a favorable amendment and a stronger bill. (Right now, we have no idea what she's thinking about reintroduction; this is just my hope and speculation.)

Expand full comment
founding
Jun 10Liked by Scott Alexander

Thanks for flagging, reaching out to some folks connected to Buffy, and will follow up here

Expand full comment
founding

I wanted to follow up after a little digging:

I started out with the naive assumption that this was a lack of awareness on behalf of law makers, and if I could just get the message to the right people, it would help.

I now think this is more of a It’s a fight between academics — air quality climate focused folks vs. public health folks. The public health argument is not breaking through, despite 1 Day Sooner doing a good job lobbying.

UC Davis did the research around safe levels of ozone, and is pushing the lower limit. I don't know if they think that long-uv can comply with the limit, or if there are alternative technologies that are as good.

The bill is up Monday afternoon, and it will be a testimony battle between a UCDavis prof who supports the bill as is and a UCBerkeley prof on 1 day sooner’s side (school of environmental health sciences) so it should be an interesting hearing.

Expand full comment
author

Amazing, thank you!

Expand full comment

Thanks so much, David!

Expand full comment

Does Far UV actually work? What’s been invented in the past 5 years that made it work now but not during Covid when it was really needed?

Expand full comment

It likely worked then, as well?

See this X/Twitter thread, for starters ...

https://x.com/aronro/status/1550595138690289666

My naive takes:

It wasn't widely deployed because Far-UVC disinfection of indoor air never got the traction it likely deserved during the first several years of the pandemic. And then as pandemic-era restrictions were successively set aside, and also death and hospitalization rates fell (due to less deadly variants, prevalence of vaccination and/or naturally-acquired immunity, and better medical treatment), that was a further impediment to deployment.

Why didn't it get traction? Some speculation here ... during those early pandemic years, frequent air changes, HEPA filtration, etc. were among long standing 'go tos." In contrast, UVC and Far-UVC were either relatively unknown or still seen as exotic, even when they actually made it onto some specifier's or decisionmaker's radar.

It didn't help that UVC - in contrast with Far-UVC - can quickly cause injury to skin or eyes, if anyone is directly exposed to it. And Far-UVC had to overcome such concerns.

Expand full comment